Download as pdf or txt
Download as pdf or txt
You are on page 1of 228

Apunts

Exercicis
Avaluacions

TRIGONOMETRIA
ESO i primer de BATXILLER
Apunts
BTX 1 científic/tecnològic

Coneixements previs necessaris per fer TRIGONOMETRIA

1.- Definició d’angle: sentit positiu de gir dels angles i notació.


2.- Tipus d’angles: agut, recte, obtús, pla.
3.- Definició d’angles complementaris i d’angles suplementaris.
4.- Unitat de mesura d’angles: grau sexagesimal. S’ha convingut que tot el pla sigui
un angle de 360º (?).
5.- Nova unitat de mesura d’angles: radian. Relació aproximada i exacte entre
aquestes dues unitats.
6.- Us d’aquestes unitats en la calculadora: DEG i RAD.
7.- Elements, propietats, notació en un triangle: suma dels angles d’un triangle i
relació entre les mides dels costats.
8.- Classificació dels triangles segons els costats: equilàter, isòsceles i escalè.
9.- Classificació dels triangles segons els angles: acutangle, rectangle i obtusangle.
10.- Triangle rectangle: elements, notació, teorema de Pitàgores, angles aguts, àrea.
11.- Triangles semblants: proporcionalitat dels costats i criteris de semblança.
12.- Paral·lelograms, rombe, diagonals, perímetre i àrea. Les dues diagonals d’un
paral·lelogram es tallen en el seu punt mig.
13.- Trapezis: àrea i perímetre, tipus (isòsceles i rectangle).
14.- Elements d’una circumferència i del cercle. Longitud de la circumferència i àrea
del cercle.
15.- Polígons regulars: triangle equilàter, quadrat, pentàgon, hexàgon, heptàgon,
octàgon, enneàgon, decàgon, dodecàgon,.....
16.- Polígons regulars: apotema, diagonal, angle central, angle interior (fórmula),
perímetre i àrea, circumferències inscrita i circumscrita.
17.- Polígon qualsevol: suma dels angles interiors (fórmula) i nombre de diagonals
(fórmula).
18.- Altura d’un triangle equilàter en funció del costat (fórmula) i de la diagonal del
quadrat en funció del costat (fórmula).
19.- Unitats de longitud i superfície.
MATEMÀTIQUES BTX 1 tecno

Continguts conceptuals i procedimentals


Trigonometria

➢ C1) Definició d’angle, notació i sentit positiu de gir dels angles.

➢ C2) Definició d'angles complementaris i suplementaris, elements d'un triangle


rectangle, teorema de Pitàgores, suma dels tres angles d'un triangle
qualsevol, relació entre la longitud dels costats d'un triangle, classificació dels
triangles segons els seus costats i segons els seus angles.

➢ P1) Unitats de mesura d'angles: concepte de radian com a unitat de mesura


d’angles, fórmula que relaciona el nombre de radians d’un angle, la longitud
d’un arc abraçat per aquest angle i el seu radi.

➢ P2) Equivalències exactes entre graus i radians; pas de graus a radians i a


l’inrevés.

➢ C3) Definició de les raons trigonomètriques sinus, cosinus i tangent d’un


angle agut(0º< 𝜶 <90º) en un triangle rectangle.

➢ P3) Càlcul aproximat, en un triangle rectangle, del sinus, cosinus i tangent


d’un angle agut amb el regle i el transportador.

➢ C4) Valors màxim i mínim del sinus, cosinus i tangent d’un angle agut.

➢ P4) Càlcul del valor exacte de les raons trigonomètriques sinus, cosinus i
tangent dels angles de 30º i 60º en un triangle equilàter qualsevol i de les de
45º en un quadrat qualsevol.

➢ C5) Relació entre les raons trigonomètriques dels angles de 30º i 60º i, en
general, entre dos angles complementaris.

➢ P5) Extensió, per pas al límit, de les raons trigonomètriques dels angles aguts
als angles de 0º i 90º.

➢ C6) Relacions(set) entre les raons trigonomètriques d’un angle agut i per
extensió als angles de 0º i 90º.

➢ P6) Procediments per demostrar aquestes fórmules.

➢ P7) Càlcul, donada una raó i emprant les anteriors relacions, de les altres
raons trigonomètriques d’un angle agut.

➢ P8) Dibuix d'un angle agut amb una raó trigonomètrica determinada i unicitat
de la solució.

➢ P9) Resolució, càlcul de la resta d'elements, d'un triangle rectangle donats:


un angle agut i un catet, un angle agut i la hipotenusa, els dos catets, un catet
i la hipotenusa.

➢ P10) Aplicació de les raons trigonomètriques d’un angle agut al càlcul


d’altures i distàncies inaccessibles.

-Pàgina 1-
➢ C7) Definició de les raons trigonomètriques sinus, cosinus i tangent d’un
angle(0º ≤ 𝜶 ≤ 360º) qualsevol en la circumferència unitat i representació
gràfica(segments sinus, cosinus i tangent).

➢ C8) Signe del sinus, cosinus i tangent en cada quadrant de la circumferència


unitat.

➢ P11) Càlcul aproximat, emprant el calculador trigonomètric, del sinus, cosinus


i tangent d’un angle qualsevol en la circumferència unitat.

➢ C9) Manteniment, amb aquesta nova definició, del valor de les raons
trigonomètriques dels angles aguts i de 0º i 90º.

➢ C10) Valors màxim i mínim del sinus, cosinus i tangent d’un angle qualsevol.

➢ C11) Manteniment, amb aquesta nova definició, de les set relacions entre les
raons trigonomètriques d'un angle.

➢ P12) Donada una raó i el quadrant al que pertany l'angle, calcular, emprant
les anteriors relacions, les altres raons trigonomètriques d'aquest angle.

➢ C12) Sinus, cosinus i tangent dels angles de 0º, 30º, 45º, 60º, 90º, 180º, 270º
i 360º.

➢ P13) Càlcul, en la circumferència unitat, del sinus, cosinus i tangent dels


angles de 120º, 135º, 150º, 210º, 225º, 240º, 300º, 315º i 330º a partir dels
angles 30º, 45º i 60º, del primer quadrant.

➢ P14) Relacionar, en la circumferència unitat, el sinus, cosinus i tangent de


qualsevol angle del segon, tercer i quart quadrants amb les raons
trigonomètriques d'un angle del primer quadrant.

➢ P15) Justificar, en la circumferència unitat, les relacions entre les raons


trigonomètriques de dos angles complementaris, dos angles suplementaris i
dos angles oposats.

➢ P16) Construir, en la circumferència unitat, els dos angles que tenen una raó
trigonomètrica determinada.

➢ P17) Procediment per definir les raons trigonomètriques sinus, cosinus i


tangent d’un nombre real qualsevol.

➢ P18) Resolució, en graus o radians, d'equacions trigonomètriques elementals,


com ara sin x = ½.

Aplicació de les fórmules trigonomètriques en la:

➢ P19) Resolució d'equacions trigonomètriques de qualsevol tipus.

➢ P20) Simplificació d'expressions trigonomètriques i demostració d'identitats.

➢ P21) Resolució, emprant els teoremes del sinus i del cosinus, d'un triangle
qualsevol donats: els tres costats, dos costats i l'angle que formen, dos
costats i un angle oposat a un d'ells o un costat i els dos angles adjacents.

-Pàgina 2-
UNITAT DIDÀCTICA

TRIGONOMETRÍA DELS ANGLES AGUTS.


RAONS TRIGONOMÈTRIQUES:
SINUS,COSINUS I TANGENT.
RESOLUCIÓ DE TRIANGLES
RECTANGLES.
APLICACIONS AL CÀLCUL D’ALTURES I
DISTÀNCIES.
Justificació / Presentació.
És un tema que permet veure una aplicació pràctica immediata: el càlcul
d’altures i distàncies. Podrem desenvolupar activitats fora de l’aula:
construcció d’un goniòmetre i posterior aplicació de la mesura d’angles al càlcul
de dues altures (un arbre i una muntanya).

Avaluació
Per avaluar aquesta unitat didàctica es farà la mitja ponderada de les notes
obtingudes en els següents apartats:
a) Construcció del goniòmetre (implicació i resultat).
b) Realització de les dues pràctiques amb el goniòmetre (treball de camp i
presentació dels resultats).
c) Una prova escrita.

Objectius mínims i terminals.


1. Conèixer la definició d’angle i els convenis escollits per
estudiar-lo.
2. Conèixer el concepte de radian com a unitat de mesura d’angles. Saber
i entendre la relació entre el nombre de radians d’un angle, la longitud
d’un arc abraçat per aquest angle i el seu radi.
3. Saber l’equivalència exacta entre graus i radians.
4. Aplicar aquesta equivalència per a passar de graus a radians i a
l’inrevés.
5. Saber les definicions de les raons trigonomètriques sinus,
cosinus i tangent d’un angle agut en un triangle rectangle.
6. Calcular el sinus, el cosinus i la tangent d’un angle agut amb el
regle i el transportador.
7. Conèixer els valors màxim i mínim del sinus, cosinus i tangent
d’un angle agut.
8. Saber deduir les raons trigonomètriques sinus, cosinus i tangent dels
angles de 30º i 60º en un triangle equilàter i les de 45º en un quadrat.
9. Conèixer les set relacions entre les raons trigonomètriques d’un
angle agut i saber deduir-les.
10. Saber calcular les raons trigonomètriques d’un angle agut a
partir d’una raó donada.
11. Saber construir un angle agut donada una raó trigonomètrica.
12. Saber resoldre un triangle rectangle donats: un angle agut i un
catet, un angle agut i la hipotenusa, els dos catets, un catet i la
hipotenusa.
13. Saber aplicar les raons trigonomètriques d’un angle agut al
càlcul d’altures i distàncies reals.
14. Tenir cura (ser ordenat i fer bona lletra) i posar el màxim
d’atenció per tal de no equivocar-se al resoldre equacions o
sistemes.
15. Saber treballar en grup tot respectant les opinions dels altres.
Observació:
Els objectius mínims són els que es tindran en compte per l’examen de
suficiència.

-Pàgina1-
CONCEPTES i PROCEDIMENTS

C1: Trigonometria.
La paraula trigonometria significa mesurar (METRIA) tres (TRI) costats
(GONO), és a dir “mesurar un triangle”. Aquest polígon serà el protagonista
principal d’ aquest tema.
Recordeu que angle és la regió del pla compresa entre dues semirectes que
tenen el mateix origen.

Convenis:
1. S’agafa com a sentit positiu de gir dels angles el contrari al moviment de
les agulles del rellotge. Assenyala per cada angle la regió que li correspon.

¿Angle OAB?

¿Angle OBA?

2. L’angle més gran, que és tot el pla, tindrà 360 graus sexagesimals
d’amplitud.
Sabries dir per quina raó?

OAB = 0º (?)

OBA = 360º (?)

PROCEDIMENT 1: Construcció d’una nova unitat de mesura d’angles.


EL RADIAN.
Farem tres grups a la classe per construir aquesta nova unitat de mesura
d’angles anomenada radian(en la calculadora RAD). Cada grup utilitzarà un radi
diferent; per exemple 5, 7 i 9 centímetres. El seu nom prové de radi, ja que és
un dels elements que intervé en la seva construcció. Seguiu el procediment
següent:
1. Dibuixeu una semirecta.
2. Amb centre l’origen de la semirecta, i amb l’ ajuda d’ un compàs, dibuixeu
un arc de radi 5, 7 o 9 cm.
3. Amb un tros de fil de cosí de la mida del radi, feu que l’arc mesuri igual.
4. Traceu una altra semirecta amb origen l’extrem esquerra del segment i que
passi pel punt on arriba el fil. Tens davant teu l’ angle d’ un radian.
5. Calcula amb un transportador l’amplitud de l’angle dibuixat i compara el
resultat amb els companys dels altres grups. Què observes?

-Pàgina 1-
C2: Definició d’una nova unitat de mesura d’angles: EL RADIAN.

l=r

un radian  57º

La construcció de l’angle d’un radian


no depèn del radi escollit, ja que la
longitud dels arcs és proporcional
als seus radis.

PROCEDIMENT 2: Descobrim una fórmula.

1. Quants radians té l’angle que amb radi 8 cm abasta un arc de longitud 32


cm? I si l’arc mesura 16 cm? I si mesura 4 cm?
2. Si un angle mesura 4 radians i la longitud de l’arc que abraça és de 36 cm,
quant mesura el radi?
3. Si l’angle fos de 2 radians i el radi de 20 cm, quant mesuraria la longitud de
l’arc?
4. Tot observant els problemes anteriors, descobreix la fórmula que relaciona
la longitud de l’arc que abraça un angle (l), el radi de l’arc (r) i el número de
radians de l’angle ().

PROCEDIMENT 3: Ampliem.

1. Si la longitud d’una circumferència és 6 cm, quant mesura l’arc de


circumferència abraçat per un angle de 2 radians?
R = 6 cm

2. La longitud d’una circumferència és 20 cm. Calcula la longitud exacta de


l’arc que abraça un angle central de 70º. Raona la resposta.
R = 12,2 cm

3. Calcula la longitud d’una circumferència, si sabem que un angle central


d’amplitud 3 radians abraça un arc de longitud 12/ unitats.
R=8u

4. Troba la mida en radians de l’angle corresponent a un arc de longitud 4r/5


sobre una circumferència de radi r.
R = 0,8 radians

5. Calcula el valor exacte, en radians, de l’ angle corresponent a un arc de


longitud 2r en una circumferència de radi r, o sigui tot el pla.

-Pàgina 2-
C3: Relació exacte entre les dues unitats de mesura d’angles: el grau
sexagesimal i el radian. Llistat d’ equivalències exactes.

La relació exacte és: 2 radians = 360º

Equivalències exactes:

30º = /6 radians 180º =  radians

45º = /4 radians 270º = 3/2 radians

60º = /3 radians 360º = 2 radians

90º = /2 radians

PROCEDIMENT 4: Aprenem a canviar d’unitats.


1. Calcula l’amplitud exacte en radians dels següents angles: 120º, 135º, 140º,
150º, 190º, 210º, 225º, 240º, 300º, 315º i 330º.
2. Calcula l’amplitud exacte en graus dels següents angles: 5∏/9, 2∏/15,
11∏/8, 7∏/5 i 4∏/9.

C4: Definició de les raons trigonomètriques d’un angle agut.


Donat un angle agut li podem assignar, de moment, dos nombres reals: un
nombre real que representa la seva amplitud en graus i un altre que la
representa en radians. A aquests dos nombres reals, hi afegirem sis nombres
reals més que anomenarem raons trigonomètriques; venen a ser com un DNI
de l’angle, amb una sola excepció que veurem més endavant.
Aquests sis nombres reals els calcularem de la següent manera:

sin  = catet oposat (Sinus)


hipotenusa

cos  = catet contigu (Cosinus)


hipotenusa

tan  = catet oposat (Tangent)


catet contigu

cotan  = catet contigu (Cotangent)


catet oposat

sec  = hipotenusa (Secant)


catet contigu

cosec  = hipotenusa (Cosecant)


catet oposat

-Pàgina 3-
Les raons trigonomètriques d’un angle agut no depenen del triangle rectangle
escollit, perquè, en tenir els angles iguals, els triangles són semblants, i per tant
tenen els costats proporcionals; en conseqüència els quocients que obtindrem
seran els mateixos.

sin  = BC = MN = PQ
AB AM AP

PROCEDIMENT 5

Per la realització d’ aquest exercici, és imprescindible dibuixar el triangle


rectangle i prendre les mides amb molta cura.
Dibuixa, amb l’ajuda d’un transportador, un triangle rectangle amb un angle de
35º. Mesura els tres costats amb un regle i amb les longituds obtingudes,
completa la taula, que trobaràs en l’ annexa.
Compara els teus resultats amb els dels altres companys de classe; llavors,
comenta i justifica el que observes.
Per millorar aquests resultats (més precisos),calcularem la mitja aritmètica dels
obtinguts per deu alumnes. Abans d’ iniciar els càlculs, has de completar la taula
corresponent de l’ annexa.

PROCEDIMENT 6: Valors reals màxim i mínim del sinus, cosinus i tangent


d’un angle agut.

1.- Dibuixa dos angles molt propers a 90º, i intenta esbrinar els valors de les
raons trigonomètriques sinus, cosinus i tangent. Ara construeix una taula amb
una successió d’ angles cada vegada més propers a 90º, i emprant la calculadora
busca el seu sinus, cosinus i tangent. Què observes?
Annexa: TAULA (90º)
2.- Dibuixa dos angles molt propers a 0º, i intenta esbrinar els valors de les
raons trigonomètriques sinus, cosinus i tangent. Ara construeix una taula amb
una successió d’ angles cada vegada més propers a 0º, i emprant la calculadora
busca el seu sinus, cosinus i tangent. Què observes?
Annexa: TAULA (0º)

C5: Valors reals màxim i mínim del sinus, cosinus i tangent d’un angle
agut.
0º    90º 0  sin   1
0  cos   1
0  tan 

-Pàgina 4-
C6: Raons trigonomètriques (sinus, cosinus i tangent) dels angles de 0º
i 90º.

sin 0º = 0 sin 90º = 1

cos 0º = 1 cos 90º = 0

tan 0º = 0 tan 90º = 

PROCEDIMENT 7: Càlcul de les raons trigonomètriques dels angles de


30º , 45º i 60º.
Esbrina, aplicant la definició, el valor exacte de les raons trigonomètriques dels
angles de 30º i 60º en el triangle equilàter, i les de l’angle de 45º en el quadrat.

x = √l2+l2 = √2l2 = l√2


x = √l2 – (l/2)2 = l√3
2
sin 45º= catet oposat = l = 1 = √2
sin 60º=catet oposat=(l√3)/2=√3
hipotenusa l√2 √2 2
hipotenusa l 2
cos 45º =
cos 60º =
tan 45º =
tan 60º =

sin 30º =

cos 30º =

tan 30º =

-Pàgina 5-
C7: Raons trigonomètriques (sinus, cosinus i tangent) dels angles de
30º , 45º , 60º.

sin 30º = 1 / 2 cos 30º = 3/2 tan 30º = 3/3

sin 45º = 2/2 cos 45º = 2/2 tan 45º = 1

sin 60º = 3/2 cos 60º = 1/2 tan 60º = 3

C8:Relacions entre les raons trigonomètriques (sinus, cosinus i tangent)


de dos angles aguts complementaris.

0º    90º

Sin  = cos(90º–)

cos  = sin(90º-α)

Dividint les dues igualtats anteriors


obtenim:

tan  = 1 = cotan (90º-α)


tan(90º-α)

Sabries explicar-me per què passa això? Ho compleix també la parella


d’ angles complementaris: 0º i 90º?

C9: Relacions entre les raons trigonomètriques d’un angle agut.

1
cosec  = sin 2  + cos 2  = 1
sin 

1
sec  = tan 2  + 1 = sec 2 
cos 

1
cotan  = 1 + cotan 2  = cosec 2 
tan 

sin 
tan  =
cos 

-Pàgina 6-
PROCEDIMENT 8: Comprovem les relacions.
Primer veurem dos exemples:
1. Comprova que amb l’angle de 45º es compleix que: sin 2  + cos 2  = 1
2. Comprova que amb l’angle de 60º es compleix que: tan 2  + 1 = sec 2 
I ara ve el més difícil; hem de demostrar-les en general, o sigui per un angle
qualsevol α. A veure si t’ atreveixes!.

PROCEDIMENT 9: Donada una raó trigonomètrica d’un angle agut,


esbrinar les altres, amb l’ajuda de les relacions entre les raons
trigonomètriques.

Per fer aquest exercici cal tenir present, al escriure, que:


sin 2  = ( sin  )2  sin 2
1. Sabem que l’angle agut a té per sinus 3/4. Busca les altres raons
trigonomètriques de l’angle. Has de donar els resultats simplificats i
racionalitzats.
2. Sabem que l’angle agut  té per cosinus 2/5. Busca les altres raons
trigonomètriques de l’angle. Has de donar els resultats simplificats i
racionalitzats.
3. Sabem que l’angle agut a té per tangent 2. Busca les altres raons
trigonomètriques de l’angle. Has de donar els resultats simplificats i
racionalitzats.
4. Sabem que l’angle agut a té per cotangent 3. Busca les altres raons
trigonomètriques de l’angle. Has de donar els resultats simplificats i
racionalitzats.
5. Sabem que l’angle agut a té per secant 3/2. Busca les altres raons
trigonomètriques de l’angle. Has de donar els resultats simplificats i
racionalitzats.
6. Sabem que l’angle agut a té per cosecant 7. Busca les altres raons
trigonomètriques de l’angle. Has de donar els resultats simplificats i
racionalitzats.
7. Sabem que l’angle agut a té per secant 1/2. Busca les altres raons
trigonomètriques de l’angle. Has de donar els resultats simplificats i
racionalitzats.

PROCEDIMENT 10: Construcció d’un angle agut donada una raó


trigonomètrica.

1. Construeix i senyala clarament, un angle agut que tingui per sinus 3/4.
2. Construeix i senyala clarament, un angle agut que tingui per cosinus 2/3.
3. Construeix i senyala clarament, un angle agut que tingui per tangent 3.
4. Construeix i senyala clarament, un angle agut que tingui per cotangent 5/3.
5. Construeix i senyala clarament, un angle agut que tingui per secant 2.
6. Construeix i senyala clarament, un angle agut que tingui per cosecant 7/5.
7. Construeix i senyala clarament, un angle agut que tingui per secant 1/2.

-Pàgina 7-
PROCEDIMENT 11: Resolució completa d’un triangle rectangle.

Per fer aquests exercicis cal agafar com a model aquest triangle:

i no es pot fer servir el Teorema de Pitàgores.

1. Resol el triangle rectangle del que coneixem un catet (c = 7 cm) i un angle


agut (C = 60º). Has de buscar els elements(angles i costats) que falten del
triangle.

2. Resol el triangle rectangle del que coneixem la hipotenusa (a = 6 cm) i un


angle agut (B = 45º); el mateix.

3. Resol el triangle rectangle del que coneixem la hipotenusa (a = 12 cm) i un


catet ( b = 8 cm ); el mateix.

4. Resol el triangle rectangle del que coneixem dos catets (b=10cm i c=8cm);
el mateix.

PROCEDIMENT 12: Càlcul d’altures i distàncies.


Observació important: com més lluny estem d’ un objecte, més petit és l’ angle
d’ elevació; el mateix passa amb l’ angle de depressió. Advertiments: tots els
exercicis, on intervenen únicament els angles de 30º, 45º o 60º, s’ han de fer
sense emprar la calculadora; és imprescindible, per començar, fer una
representació gràfica del problema, i anotar-hi clarament les dades de les que
disposem i les que busquem.

1. La longitud del fil que subjecta un estel és de 15 metres. Si l’angle d’elevació


de l’estel és de 30º, quina altura ateny l’estel?
2. El pilot d’un avió observa un punt del terreny amb un angle de depressió de
30º. Vint segons més tard, l’angle de depressió obtingut sobre el mateix punt
és de 45º. Sabent que vola horitzontalment i a una velocitat de 900 km/h,
calcula l’altitud del vol.
3. Des d’un cert punt es veu la part més alta de la torre d’un castell sota un
angle de 60º. Reculant 250 m en línia recta amb la torre, l’angle esdevé de
30º. Quina és l’altura d’aquesta torre?
4. Troba l’altura d’un arbre que projecta una ombra de 7m, sabent que la
inclinació dels raigs del sol en aquell moment és de 45º.
5. Situats a una certa distància del peu d’un arbre vertical, l’angle de visió és
de 60º. Quin serà l’angle si ens col·loquem a una distància triple?

-Pàgina 8-
6. Una marquesina forma un angle de 60º amb la vertical. Si la seva longitud és
de 2 m, quina ombra farà si els raigs del sol cauen verticalment?
7. En una final de la Champions, el Madrid i el Barça s’enfronten en un partit molt
tens. En el minut 69, en Xavi llança un córner, i l’Etoo no pot rematar, perquè
en Cannavaro l’ agafa per la samarreta. L’ àrbitre no pita el clar penal.
Ràpidament , en Xavi corre cap al centre del camp a defensar el contraatac,
seguint la diagonal. Calcula el temps que tarda en arribar, si va a una velocitat
de 9 m/s. Les dimensions del camp són: 110 x 60 (m). No val utilitzar Pitàgores.
8. Un observador vol calcular l’ altura d’ un gratacel de NovaYork. Mesura l’ angle
que forma l’ horitzontal amb la visual al punt més alt del gratacel i resulta 32º.
S’ acosta 300 m. al gratacel i torna a mesurar l’ angle des de la nova posició;
ara és de 44º. Quina altura té el gratacel? R=525,1m.
9. Resol un triangle isòsceles, sabent que el costat desigual fa 6 cm. i l’angle
oposat 54º.
10. Volem mesurar l’ amplada d’ un riu. A una distància de 25 m. d’ una de les
ribes hi ha un campanar de 35 m. d’altura. Pugem al campanar i observem
l’angle que formen les visuals que van cap a una riba i cap a l’ altra, que és de
20º. Calcula amb aquestes dades l’ amplada del riu. R=27 m. aprox.
11. Troba l’ àrea d’ un pentàgon regular el costat del qual fa 12 m.
12. Tenim un rombe les diagonals del qual fan 15 cm. i 6 cm. Calcula’n els angles.
13. Per assegurar un pal a terra se’l subjecta en sentits oposats amb dos cables a
dos punts separats 20 m. Els cables formen amb el terra angles de 75º i 60º.
Troba l’ alçària del pal.
14. Els dos braços d’ un compàs mesuren 11 cm. Si l’ obrim un angle de 30º, quant
mesura el radi de la circumferència que podem dibuixar? R=5,68 cm.
15. El circ és a la ciutat i s’ ha d’ instal·lar. L’ especialista a muntar-lo encara no ha
arribat i els altres no saben la quantitat de cable d’ acer que necessiten. El més
espavilat recorda que, un cop tensat el cable des de l’ extrem del pal principal
fins a un punt determinat del terra, amb el qual forma un angle de 60º, calen
dos metres més de cable que si forma amb el terra un angle de 70º. En total
han de posar sis cables tensats formant amb el terra un angle de 60º. Quants
metres de cable necessiten? R=153,06 m. aprox.
16. Volem penjar un llum a una certa distància del sostre d’ una habitació. Per fer-
ho, agafem una corda, hi lliguem el llum i la clavem pels extrems en dos punts
del sostre, separats per una distància de 140 cm., de manera que els angles
entre la corda i el sostre són de 40º i 60º a cada un dels extrems.
a) Quina serà la longitud total de la corda?
b) A quina distància del sostre quedarà el llum?
R: longitud corda=214,48 cm.; distància del sostre= 79,12 cm.
17. Quan l’ altura del sol sobre l’ horitzó baixa de 60º a 50º, la longitud de l’ ombra
d’ una torre s’ allarga 10 m. Calcula l’ alçada de la torre, suposadament vertical.
R=38,11 m.

-Pàgina 9-
Taules
Taula per calcular manualment les raons trigonomètriques d'un angle agut i comparar-les amb les obtingudes
emprant la calculadora

NOM COMPLERT DE
LA RAÓ NOM LES TEVES EL TEU RESULTAT AMB RESULTAT
TRIGONOMÈTRICA ABREUJAT DEFINICIÓ DADES SIS DECIMALS CALCULADORA ERROR<0,05

SINUS DE
L’ANGLE AGUT DE
35º

COSINUS DE
L’ ANGLE AGUT DE
35º

TANGENT DE
L’ANGLE AGUT DE
35º

COTANGENT DE
L’ ANGLE AGUT DE
35º

SECANT DE
L’ ANGLE AGUT DE
35º

COSECANT DE
L’ ANGLE AGUT DE
35º
Taula mitjana raons

Sinus de 35º Cosinus de 35º Tangent de 35º

Suma dels sinus Suma dels cosinus Suma de les tangents

Calcula la mitja aritmètica dels sinus, cosinus i de les tangents, i


compara els resultats amb els de la calculadora; han millorat en
precisió, respecta dels que havies obtingut tu, abans?
Taula per calcular manualment les raons trigonomètriques d'un angle agut i comparar-les amb les obtingudes
emprant la calculadora

NOM COMPLERT DE
LA RAÓ NOM LES TEVES EL TEU RESULTAT AMB RESULTAT
TRIGONOMÈTRICA ABREUJAT DEFINICIÓ DADES SIS DECIMALS CALCULADORA ERROR<0,05

SINUS DE
L’ANGLE AGUT DE
67º

COSINUS DE
L’ ANGLE AGUT DE
67º

TANGENT DE
L’ANGLE AGUT DE
67º

COTANGENT DE
L’ ANGLE AGUT DE
67º

SECANT DE
L’ ANGLE AGUT DE
67º

COSECANT DE
L’ ANGLE AGUT DE
67º
Taula obtenció raons trigonomètriques de l'angle de 0º

ANGLES SINUS COSINUS TANGENT

0,1º

0,01º

0,001º

0,0001º

0,00001º

SIN 0º =

COS 0º =

TAN 0º =
Taula obtenció raons trigonomètriques de l'angle de 90º

ANGLES SINUS COSINUS TANGENT

88º

89º

89,9º

89,99º

89,999º

89,9999º

89,99999º

89,999999º

90º

SIN 90º =

COS 90º =

TAN 90º =
Taula graus-radians-raons angles notables

GRAUS RADIANS SINUS COSINUS TANGENT

0º 0

∏/6

45º

60º

90º

120º

3∏/4

150º

180º

210º

5∏/4

240º

270º

5∏/3

315º

330º

360º
Taula graus-radians-raons angles notables

GRAUS RADIANS SINUS COSINUS TANGENT

0º 0 0 1 0

30º 𝛑/6 1/2 √𝟑/2 √𝟑/3

45º 𝛑/4 √𝟐/2 √𝟐/2 1

60º 𝛑/3 √𝟑/2 1/2 √𝟑

90º 𝛑/2 1 0 ∞
120º 𝟐𝛑/3 √𝟑/2 -1/2 −√𝟑

135º 𝟑𝛑/4 √𝟐/2 −√𝟐/2 -1

150º 𝟓𝛑/6 1/2 −√𝟑/2 −√𝟑/3

180º 𝛑 0 -1 0

210º 𝟕𝛑/6 -1/2 −√𝟑/2 √𝟑/3

225º 𝟓𝛑/4 −√𝟐/2 −√𝟐/2 1

240º 𝟒𝛑/3 −√𝟑/2 -1/2 √𝟑

270º 𝟑𝛑/2 -1 0 ∞
300º 𝟓𝛑/3 −√𝟑/2 1/2 −√𝟑

315º 𝟕𝛑/4 −√𝟐/2 √𝟐/2 -1

330º 𝟏𝟏𝛑/6 -1/2 √𝟑/2 −√𝟑/3

360º 𝟐𝛑 0 1 0
67 resultats + 3 presentació = 70
TAULA DELS SINUS DELS ANGLES NOTABLES

VALORS
−√𝟑 −√𝟐 −𝟏 +𝟏 +√𝟐 +√𝟑
DEL -1 0 +1
SINUS 𝟐 𝟐 𝟐 𝟐 𝟐 𝟐
ANGLE
NOTABLE
EN GRAUS
ANGLE
NOTABLE
EN GRAUS
ANGLE
NOTABLE
EN GRAUS

TAULA DELS COSINUS DELS ANGLES NOTABLES

VALORS
−√𝟑 −√𝟐 −𝟏 +𝟏 +√𝟐 +√𝟑
DEL -1 0 +1
COSINUS 𝟐 𝟐 𝟐 𝟐 𝟐 𝟐
ANGLE
NOTABLE
EN GRAUS
ANGLE
NOTABLE
EN GRAUS
ANGLE
NOTABLE
EN GRAUS

TAULA DE LES TANGENTS DELS ANGLES NOTABLES

VALORS
-√𝟑 −√𝟑 √𝟑 +√𝟑
DEL LA -1 0 + +1
TANGENT 𝟑 𝟑
ANGLE
NOTABLE
EN GRAUS
ANGLE
NOTABLE
EN GRAUS
ANGLE
NOTABLE
EN GRAUS
TAULA DELS SINUS DELS ANGLES NOTABLES

VALORS
−√𝟑 −√𝟐 −𝟏 +𝟏 +√𝟐 +√𝟑
DEL -1 0 +1
SINUS 𝟐 𝟐 𝟐 𝟐 𝟐 𝟐
ANGLE
NOTABLE 270º 240º 225º 210º 0º 30º 45º 60º 90º
EN GRAUS
ANGLE
NOTABLE 300º 315º 330º 180º 150º 135º 120º
EN GRAUS
ANGLE
NOTABLE 360º
EN GRAUS

TAULA DELS COSINUS DELS ANGLES NOTABLES

VALORS
−√𝟑 −√𝟐 −𝟏 +𝟏 +√𝟐 +√𝟑
DEL -1 0 +1
COSINUS 𝟐 𝟐 𝟐 𝟐 𝟐 𝟐
ANGLE
NOTABLE 180º 150º 135º 120º 90º 60º 45º 30º 0º
EN GRAUS
ANGLE
NOTABLE 210º 225º 240º 270º 300º 315º 330º 360º
EN GRAUS
ANGLE
NOTABLE
EN GRAUS

TAULA DE LES TANGENTS DELS ANGLES NOTABLES

VALORS
-√𝟑 −√𝟑 √𝟑 +√𝟑
DEL LA -1 0 + +1
TANGENT 𝟑 𝟑
ANGLE
NOTABLE 135º 120º 150º 0º 30º 60º 45º
EN GRAUS
ANGLE
NOTABLE 315º 300º 330º 180º 210º 240º 225º
EN GRAUS
ANGLE
NOTABLE 360º
EN GRAUS

49 resultats + 1 presentació = 50
PRÀCTICA DEL GONIÒMETRE

Amb aquesta pràctica es pretén experimentar, sobre el terreny, els conceptes


teòrics explicats a classe, i que l’alumne vegi, encara que a un nivell
elemental, algunes aplicacions de la Trigonometria.
Han de formar-se equips de dos o excepcionalment tres alumnes. Cada equip
ha de construir un goniòmetre vertical, que servirà per mesurar angles en el
pla vertical.

Construcció del goniòmetre vertical

Material que es necessita:

 Tub cilíndric rígid (coure, plàstic dur...) de radi petit (màxim ½ centímetre) i
d’uns 40 cm. de llarg, que ens servirà de visor.

 Semicercle de cartró rígid, d’uns 10 cm. de radi, amb dos subdivisions de 0º a


90º (grau a grau), tal com es veu en la figura.
Heu de tenir molta cura a l’hora de graduar el semicercle i que les marques
siguin ben visibles. Les mesures que feu posteriorment seran fiables, sempre i
quan, el semicercle estigui molt ben dissenyat.

 Fil elèctrics resistent d’uns 12 cm.

 Un pes de plom o ferro per tal que el fil estigui tens.


Una vegada construït i revisat el goniòmetre vertical i amb l’ajuda d’una cinta mètrica
(20 metres mínim), cada equip prendrà les mides que s’indiquen en les figures per
posteriorment amb els coneixements de Trigonometria adquirits i càlculs relativament
senzills, determinar:
a) L’alçada d’un arbre
b) Una altura de la base inaccessible, per exemple una muntanya

Pensa i contesta: per què els dos angles α de la figura son iguals?
x = distància de l’observador al peu de l’arbre (cinta mètrica).
Quant més gran sigui aquesta distància, més fiable serà el resultat final t (altura
de l’arbre); per què?
y = alçada de l’observador (cinta mètrica).
α = angle de visió (goniòmetre).
z  (càlculs trigonomètrics).
Finalment: t(altura de l’arbre) = y + z

Recomanacions:
Les observacions es faran el més a prop possible del terra, de manera que l’alçada de
l’observador sigui inconsiderable.
Perquè la pràctica sigui correcta els dos punts d’observació i el punt inaccessible han
d’estar alineats.
És recomanable que els don punt d’observació estiguin el més allunyats possible de la
base, i entre ell hi hagi com a mínim 10 metres.
x = distància entre els dos punts d’observació (cinta mètrica).
α , β = angles de visió (goniòmetre).
y = dada desconeguda i d’impossible càlcul en la pràctica.
z = altura que volem esbrinar (càlculs trigonomètrics).
TRIGONOMETRIA DELS ANGLES AGUTS
1.- Triangles semblants
Direm que dos triangles són semblants si tenen els angles iguals i els costats
corresponents són proporcionals. La raó de proporcionalitat entre costats que
es corresponen en dos triangles semblants s’anomenarà raó de semblança (?).

A) TRIANGLES EN POSICIÓ DE TALES

Un cas particular de triangles semblants són els triangles en posició de Tales.


Els triangles ABC i ADE de la figura tenen l'angle 𝑨 ̂ en comú i els
costats BC i DE són paral·lels. A més es verifiquen les igualtats entre angles
següents: . Quan passa això, es diu que els triangles estan
en posició de Tales. Per tant:
Direm que dos triangles estan en posició de Tales si tenen un angle comú i els
costats oposats a aquest angle són paral·lels. Es demostra que dos triangles
que estan en posició de Tales són semblants.

B) CRITERIS DE SEMBLANÇA DE TRIANGLES


Per veure si dos triangles són semblants, no cal comprovar que els costats són
proporcionals i que els angles són iguals. Només cal que es compleixi un dels
conjunts de condicions següents, anomenats criteris de semblança.
Primer criteri: dos triangles són semblants si tenen dos angles iguals.
Segon criteri: dos triangles són semblants si tenen dos costats proporcionals
i l'angle comprès entre ells és igual.
Tercer criteri: dos triangles són semblants si tenen els tres costats
proporcionals.
És fàcil veure que si es compleix un d'aquests conjunts de condicions els
triangles es poden col·locar en posició de Tales i, per tant, són semblants.

c) Criteris de semblança de triangles rectangles


Si els triangles són rectangles, els criteris de semblança se simplifiquen.
Primer criteri: dos triangles rectangles són semblants si tenen un angle agut
igual.
Segon criteri: dos triangles rectangles són semblants si tenen dos costats
proporcionals.
Vegem un exemple d'aplicació de la semblança de triangles rectangles.

-Pàgina 1-
EXEMPLE: Esbrina l'altura d'un edifici que projecta una ombra de 36 m al
mateix temps que un bastó d'1 m de longitud, col·locat verticalment a terra,
projecta una ombra de 3 m.

Els triangles rectangles del dibuix són semblants perquè tenen igual l'angle
agut format pels raigs del Sol i el terreny. Per tant, els costats corresponents
són proporcionals. Es compleix:

⇒ x = 12 ⇒ L'altura de l'edifici és de 12 m.

2. Mesura d'angles
A) MESURA D'ANGLES EN GRAUS SEXAGESIMALS
Un grau sexagesimal (1°) és la mida de l'angle que s'obté quan es divideix
tot el pla en 360 angles iguals. Cada grau es divideix en seixanta parts iguals
anomenades minuts (') i cada minut es divideix en seixanta parts iguals
anomenades segons ("):

B) MESURA D'ANGLES EN RADIANS


Un radian és la mida d'un angle central d'una circumferència els costats del
qual delimiten un arc de longitud igual al radi. Se simbolitza amb rad.

Angle d'1 radian. La longitud del seu arc és igual al radi

-Pàgina 2-
Així doncs, l'angle complet de la circumferència fa 2π rad, perquè:

C) Relació entre l'angle central i l'arc de circumferència


De la definició de radian s'obté la relació entre un angle central α i la longitud
del seu arc l :

, radi r el radi i α mesurat en radians.

D) PAS DE GRAUS A RADIANS I VICEVERSA


Com que un angle complet fa 360° o 2π rad, per canviar d'unitat de mesura
s'aplica una regla de tres simple directa.
EXEMPLES:
1. Expressa en radians un angle de 60°.
S'ha de verificar:

2. Expressa un angle de 3π/2 radians en graus.


S'ha de verificar:

Relació exacte entre les dues unitats de mesura d’angles: el grau sexagesimal
i el radian. Llistat d’ equivalències exactes.

30º = /6 radians 180º =  radians

45º = /4 radians 270º = 3/2 radians

60º = /3 radians 360º = 2 radians

90º = /2 radians

-Pàgina 3-
3. Raons trigonomètriques d'un angle agut

Considerem un triangle rectangle d'hipotenusa a i catets b i c com el de la


figura. Definim les raons trigonomètriques directes de l'angle agut α de la
manera següent:
 Sinus de α és la raó entre el catet oposat c i la hipotenusa a. Se simbolitza
amb sin α.
 Cosinus de α és la raó entre el catet adjacent b i la hipotenusa a. Se
simbolitza amb cos α.
 Tangent de α és la raó entre el catet oposat c i el catet adjacent b. Se
simbolitza amb tg α.

Si agafem dos triangles rectangles ABC i A'B'C' amb el mateix angle α,


obtindrem els mateixos valors per a les raons trigonomètriques de α, ja
que ABC i A'B'C' són semblants i, per tant, es compleix:

Podem concloure que les raons trigonomètriques només depenen de la mida


de l'angle α i no del triangle rectangle considerat. També podem considerar
les inverses de les raons trigonomètriques directes , denominades raons
trigonomètriques recíproques: cosecant de α (cosec α), secant de α (sec α)
i cotangent de α (cotg α), definides de la manera següent:

-PÀGINA 4-
EXEMPLE: Calcula les raons trigonomètriques de l'angle α del triangle rectangle
d'hipotenusa a = 13 cm, catet adjacent b = 12 cm i catet oposat c = 5 cm; però
primer dibuixa un triangle rectangle qualsevol i anota-hi aquestes dades (?).

4.- Identitats trigonomètriques


A) RELACIÓ ENTRE EL SINUS, EL COSINUS I LA TANGENT DEL MATEIX
ANGLE

Fixa't que entre el sinus, el cosinus i la tangent del mateix angle α es verifica
la relació següent:

Per tant:

-Pàgina 5-
B) RELACIÓ FONAMENTAL DE LA TRIGONOMETRIA
Aplicant el teorema de Pitàgores al triangle rectangle anterior, s'obté:
c2 + b2 = a2
Dividint tots els termes per a2, s'obté:

Aquesta igualtat es coneix com a relació fonamental de la trigonometria. Si


ara dividim ambdós membres de la igualtat anterior per cos2 α i simplifiquem,
obtenim:

De manera anàloga, si dividim tots dos membres de la relació fonamental per


sin2 α i simplifiquem, obtenim:

Aquestes igualtats permeten esbrinar totes les raons trigonomètriques d'un


angle quan se'n coneix una.
EXEMPLE 1: Calcula les raons trigonomètriques directes restants de l'angle
agut α, sabent que sin α = 0,4.
Substituint a l'equació sin2 α + cos2 α = 1, s'obté:

-Pàgina 6-
EXEMPLE 2: Calcula les raons trigonomètriques directes restants de l'angle
agut α, sabent que cos α = 0,6.
Substituint a l'equació sin2 α + cos2 α = 1, s'obté:

EXEMPLE 3: Calcula les raons trigonomètriques directes restants de l'angle


agut α, sabent que tg α = 2.
Substituint a l'equació tg2 α + 1 = sec2 α, s'obté:

Com que tg α = sin α/cos α , es verifica que sin α = tg α · cos α. Substituint els
valors de la tangent i del cosinus, s'obté:

sin α = 2 · 0,45 = 0,90

5.- Raons dels angles de 60°, 30° i 45°


A) RAONS TRIGONOMÈTRIQUES DE L'ANGLE DE 60°

Dibuixem un triangle equilàter. Pot ser qualsevol, ja que els angles interiors
faran sempre 60°. L'altura corresponent a un dels costats divideix el triangle
equilàter en dos triangles rectangles.

-Pàgina 7-
Sense pèrdua de generalitat, considerem que el costat del triangle equilàter
fa 1. Apliquem el teorema de Pitàgores per calcular l'altura h del triangle:

Aplicant la definició de les raons trigonomètriques, obtenim:

En resum:

B) RAONS TRIGONOMÈTRIQUES DE L'ANGLE DE 30°


Com que 30° i 60° són angles complementaris, podem trobar les raons
trigonomètriques de l'angle de 30° a partir de les de l'angle de 60° (?).
D'aquesta manera, obtenim:

En resum:

-PÀGINA 8-
C) RAONS TRIGONOMÈTRIQUES DE L'ANGLE DE 45°

Dibuixem un triangle isòsceles rectangle. Pot ser qualsevol, ja que els angles
iguals faran sempre 45°. Sense pèrdua de generalitat, considerem que els
costats iguals del triangle, és a dir, els catets, mesuren 1. Apliquem el teorema
de Pitàgores per calcular la hipotenusa del triangle rectangle:

Aplicant la definició de les raons trigonomètriques, obtenim:

En resum:

Les raons trigonomètriques dels angles de 30°, 45° i 60° s'utilitzen amb molta
freqüència. És per això que et convé aprendre-les de memòria. Les resumim a
la taula següent:

-Pàgina 9-
6.- Resolució de triangles rectangles
Resoldre un triangle és esbrinar quant fan els tres costats i els tres angles,
sabent la mida de tres d'ells. En el cas particular dels triangles rectangles, com
que ja sabem que un dels angles fa 90°, només cal conèixer dues dades més.
Considerem que el triangle és ABC, rectangle en A, i que la manera de
simbolitzar els costats és l'habitual en els triangles: a, b i c (?).

Primer cas: donats els dos catets, b i c


Calculem la hipotenusa aplicant el teorema de Pitàgores:

Calculem (?) utilitzant la seva tangent: tg = c/b

Calculem tenint en compte que és complementari de : = 90° –


EXEMPLE:

Resol el triangle rectangle que té de catets b = 5 cm i c = 4 cm.

Segon cas: donats la hipotenusa, a, i un angle agut,


Calculem els catets utilitzant el sinus i el cosinus de l'angle :

Calculem tenint en compte que és complementari de : = 90° –


EXEMPLE:
Resol el triangle rectangle que té 15 cm d' hipotenusa i = 36° 52' 12".

-Pàgina 10-
Tercer cas: donats la hipotenusa, a, i un catet, b
Utilitzem el teorema de Pitàgores per calcular l'altre catet:

Apliquem el cosinus de per calcular l'angle :

Com que i són angles complementaris: = 90° –


EXEMPLE: Resol el triangle rectangle que té d'hipotenusa a = 17 cm i un dels
catets b = 15 cm.

Quart cas: donats un catet, b, i l'angle contigu,


Utilitzem cos per calcular la hipotenusa:

Calculem el catet c aplicant el teorema de Pitàgores:

Com que i són angles complementaris: = 90° –

Si l'angle donat és l'oposat al catet b, , calculem = 90° – i procedim


de la mateixa manera per calcular a i c.

EXEMPLE: Resol el triangle rectangle que té un dels catets b = 9 cm i = 43°.

-Pàgina 11-
Tingues en compte que en la resolució de triangles rectangles és preferible fer
sempre els càlculs amb les dades originals en comptes de fer-ho amb resultats
intermedis, perquè les dades originals es consideren exactes, mentre que els
resultats intermedis, quan s'han obtingut per arrodoniment, introdueixen
errors que s'arrosseguen als càlculs següents.

7.- Aplicacions de la trigonometria


La resolució de triangles rectangles té nombroses aplicacions en la resolució
de problemes. Vegem-ne alguns exemples.
EXEMPLE 1:
Calcula l'altura d'un edifici sabent que, des del punt on es troba l'observador,
l'angle d'elevació que forma el terreny amb el cim de l'edifici és de 34°, i que
la distància de l'observador al peu de l'edifici és de 30 m.

EXEMPLE 2:
Quan es recorren 150 m per una carretera, l' altitud sobre el nivell del mar
augmenta en 18 m. Quina és la inclinació de la carretera?

EXEMPLE 3:
Des d'un punt, s'observa el cimall d'un arbre amb una inclinació de 60°. Si es
retrocedeixen 20 m en línia recta, la inclinació és de 30°. Calcula l' altura de
l'arbre.

-Pàgina 12-
Considerem x la distància del primer lloc d'observació al peu de l'arbre.
Utilitzem la tangent dels dos angles per plantejar les dues equacions
següents:

Resolem el sistema per igualació:

Si substituïm el valor obtingut per a x, resulta l'altura:

h = x · tg 60° = 10 · 1,732 = 17,32 m

EXEMPLE 4:
Calcula l'apotema d'un pentàgon regular de 10 m de costat.

Els angles interiors d'un pentàgon regular fan 108° (?). Per tant, l'angle entre
un costat del pentàgon i el segment que uneix un vèrtex d' aquest costat
amb el centre fa 54°. Simbolitzem amb a l'apotema del pentàgon. Per tant:

EXEMPLE 5:
La base d' una casa és un rectangle de 14 m de llarg per 10 m d' ample.
Calcula la superfície de la teulada a dues aigües, sabent que té una inclinació
de 30°.

-Pàgina 13-
Considerem el triangle rectangle ABC:

Per tant, l'àrea de la teulada és:

2 · (14 · 5,77) = 161,56 m2

8.- Teoremes per resoldre qualsevol tipus de


triangles
Sigui ABC un triangle qualsevol, com el de la figura.
Teorema del sinus:
a b c
= =
sin A sin B sin C
Teorema del cosinus:
a2 = b2 + c2 – 2·b·c·cos A
b2 = a2 + c2 – 2·a·c·cos B
c2 = a2 + b2 – 2·a·b·cos C
C

a b

B A
c
-Pàgina 14-
UNITAT DIDÀCTICA

RAONS TRIGONOMÈTRIQUES D’UN


ANGLE QUALSEVOL
CIRCUMFERÈNCIA UNITAT
EQUACIONS TRIGONOMÈTRIQUES
ELEMENTALS.
Justificació / Presentació.

Volem definir les raons trigonomètriques de qualsevol número. Construirem el


calculador trigonomètric per posar en pràctica les definicions. Veurem que les
raons trigonomètriques dels angles aguts continuen sent les mateixes.
Finalment resoldrem equacions trigonomètriques elementals.
Al acabar la unitat, ja estarem preparats per , en un proper curs, estudiar les
funcions trigonomètriques.

Avaluació

Per avaluar aquesta unitat didàctica es farà la mitja ponderada de les notes
obtingudes en els següents apartats:
a) dibuix del calculador trigonomètric(implicació i resultat).
b) pràctiques amb el calculador trigonomètric.
c) una prova escrita.

Objectius mínims i terminals.


1. Saber calcular el sinus, el cosinus i la tangent d’un angle
qualsevol en la circumferència unitat amb el calculador
trigonomètric.
2. Conèixer el signe i la representació gràfica del sinus, el cosinus i
la tangent en cada quadrant de la circumferència unitat.
3. Conèixer els valors màxim i mínim del sinus, cosinus i tangent
d’un angle qualsevol i saber-ho justificar.
4. Conèixer el sinus, el cosinus i la tangent dels angles de 0º, 30º,
45º, 60º, 90º, 180º, 270º i 360º.
5. Saber calcular les raons trigonomètriques d’un angle qualsevol a partir
d’una raó donada.
6. Saber justificar les relacions entre les raons trigonomètriques de dos
angles complementaris, dos angles suplementaris i dos angles oposats.
7. Saber calcular en la circumferència unitat el sinus, el cosinus i la
tangent dels angles de 120º, 135º, 150º, 210º, 225º, 240º, 300º,
315º i 330º a partir dels angles del primer quadrant de 30º, 45º i
60º.
8. Saber construir, en la circumferència unitat, un angle amb una raó
trigonomètrica determinada.
9. Saber definir les raons trigonomètriques sinus, cosinus i tangent d’un
nombre real qualsevol.
10.Resoldre les equacions trigonomètriques elementals, com ara l’equació
sin x = ½.
11.Tenir cura (ser ordenat i fer bona lletra) i posar el màxim
d’atenció per tal de no equivocar-se al resoldre equacions o
sistemes.
12.Saber treballar en grup, tot respectant les opinions dels altres.

Observació:
Els objectius mínims són els que es tindran en compte per l’examen de
suficiència.
-Pàgina1-
C1: Circumferència unitat.
La circumferència unitat, com abans el triangle rectangle, és l’eina que
utilitzarem per definir les raons trigonomètriques de qualsevol angle. Tal com
diu el seu nom, el radi és “1”, i el centre el situarem en l’ origen de
coordenades. La primera semirecta de tots els angles serà la part positiva de
l’ eix d’ abscisses, i el sentit positiu de gir serà el contrari al de les agulles del
rellotge.
Si tenim en compte a on està situada la segona semirecta de l’ angle, direm
que els angles aguts(entre 0º i 90º) són els angles del primer quadrant, els
angles entre 90º i 180º són del segon quadrant, els angles entre 180º i 270º
són del tercer quadrant i els angles entre 270º i 360º són del quart quadrant.

-Pàgina2-
C2: Definició de les raons trigonomètriques d’un angle qualsevol en la
circumferència unitat.

sin  = y 0º    360º

cos  = x 0    2

tan  = y / x

cotan  = x / y

sec  = 1 / x

cosec  = 1 / y

Aquesta nova definició té tres avantatges:


- permet calcular les raons trigonomètriques de qualsevol angle.
- les raons trigonomètriques dels angles aguts, obtingudes emprant un
triangle rectangle, no es modifiquen; comprova-ho amb l’ angle de 35º.
- tampoc varien les raons trigonomètriques dels angles de 0º i 90º;
comprova-ho.
-Pàgina3-
C3: Signe de les raons trigonomètriques sinus, cosinus i tangent en
cada quadrant.

C4: Raons trigonomètriques (sinus, cosinus i tangent) dels angles de


180º, 270º i 360º.

sin 180º = 0 cos 180º = –1 tan 180º = 0

sin 270º = –1 cos 270º = 0 tan 270º = 

sin 360º = 0 cos 360º = 1 tan 360º = 0

Pensa: existeix una parella d’ angles amb les mateixes raons


trigonomètriques?; en cas afirmatiu, digues quins angles són.

-Pàgina4-
C5: Representació gràfica de les raons trigonomètriques (sinus,
cosinus i tangent) en cada quadrant de la circumferència unitat.

PRIMER QUADRANT

0º    90º

0    /2

-Pàgina5-
SEGON QUADRANT

90º    180º

/2    

-Pàgina6-
TERCER QUADRANT

180º    270º

    3/2

-Pàgina7-
QUART QUADRANT

270º    360º

3/2    2

-Pàgina8-
PROCEDIMENT 1: Calculador trigonomètric.

1. Dibuixa en un calculador trigonomètric els angles de 69º,174º,241º i 323º i


calcula el seu sinus, cosinus i tangent; compara els resultats amb els de la
calculadora.
2. Dibuixa en el mateix calculador trigonomètric els angles de 45º, 135º, 225º i
315º, i calcula el sinus, cosinus i tangent de 135º,225º i 315º, emprant les
raons ja conegudes de l’ angle de 45º.
3. Dibuixa en el mateix calculador trigonomètric els angles de 30º, 150º, 210º i
330º, i calcula el sinus, cosinus i tangent de 150º,210º i 330º, emprant les
raons ja conegudes de l’ angle de 30º.
4. Dibuixa en el mateix calculador trigonomètric els angles de 60º, 120º, 240º i
300º, i calcula el sinus, cosinus i tangent de 120º,240º i 300º, emprant les
raons ja conegudes de l’ angle de 60º.

C6: Valors màxim i mínim del sinus, cosinus i tangent d’un angle
qualsevol.

0º    360º –1  sin   1
–1  cos   1
–   tan   +

PROCEDIMENT 2: Donada una raó trigonomètrica d’un angle qualsevol,


esbrina les altres, amb l’ajuda de les set relacions que hi ha entre les
raons trigonomètriques d’un angle.

1. Sabem que l’angle a pertany al segon quadrant i sin a = 3/4 . Calcula les
altres raons trigonomètriques. Has de donar els resultats simplificats i
racionalitzats.
2. Si cos a = –1/3 i 180º  a  270º, calcula les altres raons trigonomètriques.
Has de donar els resultats simplificats i racionalitzats.
3. Si tan  = –8/3 i 3/2    2 , calcula les altres raons trigonomètriques.
Has de donar els resultats simplificats i racionalitzats.
4. Si cotan  = 3 i     3/2 , calcula les altres raons trigonomètriques.
Has de donar els resultats simplificats i racionalitzats.
5. Si sec a = 1/2 i 90º  a  180º , calcula les altres raons trigonomètriques.
Has de donar els resultats simplificats i racionalitzats.
6. Si cosec  = 5 i /2     , calcula les altres raons trigonomètriques. Has
de donar els resultats simplificats i racionalitzats.

-Pàgina9-
C7: Relacions entre les raons trigonomètriques (sinus, cosinus i
tangent) de dos angles aguts complementaris:  i 90º –  .

sin  = cos ( 90º –  )


cos  = sin ( 90º –  )
tan  = cotan ( 90º –  )

-Pàgina10-
C8: Relacions entre les raons trigonomètriques (sinus, cosinus i
tangent) de dos angles suplementaris:  i 180º –  .

sin  = sin ( 180º –  )


cos  = – cos ( 180º –  )
tan  = – tan ( 180º –  )

-Pàgina11-
C9: Relacions entre les raons trigonomètriques (sinus, cosinus i
tangent) de dos angles oposats:  i –  .

sin  = – sin ( –  )
cos  = cos ( –  )
tan  = –tan ( –  )

-Pàgina12-
PROCEDIMENT 2 : Càlcul, en la circumferència unitat, de les raons
trigonomètriques d’un angle del segon, tercer i quart quadrants a
partir d’un angle del primer quadrant.
1. Calcula les raons trigonomètriques sinus, cosinus i tangent de l’angle de
135º, a partir d’un angle del primer quadrant.
2. Calcula les raons trigonomètriques sinus, cosinus i tangent de l’angle de
210º, a partir d’un angle del primer quadrant.
3. Calcula les raons trigonomètriques sinus, cosinus i tangent de l’angle de
330º, a partir d’un angle del primer quadrant.
4. Calcula les raons trigonomètriques sinus, cosinus i tangent de l’angle de
127º, a partir d’un angle del primer quadrant.
5. Calcula les raons trigonomètriques sinus, cosinus i tangent de l’angle de
265º, a partir d’un angle del primer quadrant.
6. Calcula les raons trigonomètriques sinus, cosinus i tangent de l’angle de
315º, a partir d’un angle del primer quadrant.

PROCEDIMENT 3: Construcció, en la circumferència unitat, d’un angle


amb una raó trigonomètrica determinada. Unicitat de la solució.
1. Construeix i assenyala correctament, en la circumferència unitat, un angle
que tingui per sinus 3/4. Quants n’hi ha?. Raona la resposta.
2. Construeix i assenyala correctament, en la circumferència unitat, un angle
que tingui per cosinus –2/5. Quants n’hi ha?. Raona la resposta.
3. Construeix i assenyala correctament, en la circumferència unitat, un angle
que tingui per tangent –5/3. Quants n’hi ha?. Raona la resposta.
4. Construeix i assenyala correctament, en la circumferència unitat, un angle
que tingui per cosecant 3. Quants n’hi ha?. Raona la resposta.

PROCEDIMENT 4: Ampliació de les raons trigonomètriques a un


nombre real qualsevol.
Calcula les raons trigonomètriques sinus, cosinus i tangent dels nombres reals:
1920º , 7/3 radians , (–60º) , (–) radians , 2730º i (–16290º).

PROCEDIMENT 5: Resolució d’equacions trigonomètriques elementals.


Resol les següents equacions trigonomètriques en graus i radians:
a) sin x = 1/2
b) cos x = –1
c) tan x = 3 / 3
d) sin x = sin 27º
e) cos x = 1/2
f) tan x = –1
g) sin x = – 3 / 2

-Pàgina13-
CALCULADOR TRIGONOMÈTRIC
Circumferència unitat (R = 1 (10 cms))

(tres decimals) (dos decimals)


RESULTAT CALCULADORA ERROR( ≤0'02)

sin  =

cos  =

tan  =
Teorema del sinus
Per qualsevol triangle, rectangle o no, la longitud dels costats és proporcional al sinus de l'angle
oposat:
a b c
= =
sin  A sin  B sin C 

Però compte, que hi ha problemes que poden tenir dues solucions, doncs hi ha dos angles diferents
entre 0 i 180 amb el mateix valor per al sinus, un al primer quadrant i l'altre al segon quadrant ( i la
calculadora només ens dóna el resultat del primer quadrant, l'altre l'hem de calcular "a mà"):
Per exemple, si sin(A)=0,6947, l'angle A pot tenir dos valors diferents A=44º o bé A=180-44=136º.
Ja que aquests dos angles tenen sinus
0,6947. Si amb la calculadora fem
sin-1(0,6947) obtenenim només el valor de
44º. Hem de saber que hi ha una altra
solució i que es troba fent 180-44º

Exercici 1:

Resoleu el triangle sabent que a=120m i


A=30º i B=45º

Exercici 2:

Trobeu tots els triangles possibles amb la


condició que a=100m, b=110m i A=30º

Exercici 3:

Resoleu el triangle sabent que a=20m,


b=100m i A=30º
Teorema del cosinus
En un triangle general, NO necessàriament rectangle, NO podem aplicar el teorema de Pitàgores.
Però tenim un teorema que és més general. El teorema del cosinus ens diu la relació que hi ha entre
els quadrats de les longituds dels costats:

Observeu que si l'angle A és de 90, l'expressió que resulta és el mateix teorema de Pitàgores.

Això ens permet resoldre triangles quan només coneixem un angle, però no sabem el valor del
costat oposat del mateix (no serveix de res aplicar el teorema del sinus), però si coneixem el valor
dels altres dos costats..

Quan coneixem dos costats i l'angle comprès entre els dos costats, el teorema del cosinus ens
permet calcular el costat que falta. Raoneu que en aquest cas sempre trobarem una única solució.
Per acabar de resoldre el triangle aplicarem el teorema del sinus, raonant si els angles que falten són
aguts (prime quadrant) o obtusos (segon quadrant).

Cal tenir en compte que podem aplicar el teorema del cosinus a qualsevol dels tres costats, només
hem de fer una "rotació" de les tres lletres:

a2=b2+c2-2bc·cos(A)
b2=a2+c2-2ac·cos(B)
c2=a2+b2-2ab·cos(C)
Exercici 1:

Resoleu el triangle sabent que b=120m, c= 20 i A= 120º

Exercici 2:

Resoleu el triangle sabent que a=120m, b= 20 i C= 75º

Exercici 3:

Resoleu el triangle sabent que a=20m, c=100m i B=30º


FÓRMULES TRIGONOMETRIA

𝐬𝐢𝐧( 𝒂 + 𝒃 ) = 𝐬𝐢𝐧 𝒂 · 𝐜𝐨𝐬 𝒃 + 𝐜𝐨𝐬 𝒂 · 𝐬𝐢𝐧 𝒃 sin 2a = 2 sin a · cos a


𝐜𝐨𝐬( 𝒂 + 𝒃 ) = 𝐜𝐨𝐬 𝒂 · 𝐜𝐨𝐬 𝒃 - 𝐬𝐢𝐧 𝒂 · 𝐬𝐢𝐧 𝒃 cos 2a = cos2 a – sin2 a
tan a + tan b 2·tan a
𝐭𝐚𝐧( 𝒂 + 𝒃 ) = tan 2a =
1 – tan a · tan b 1 – tan2 a

𝐬𝐢𝐧( 𝒂 − 𝒃 ) = 𝐬𝐢𝐧 𝒂 · 𝐜𝐨𝐬 𝒃 - 𝐜𝐨𝐬 𝒂 · 𝐬𝐢𝐧 𝒃


𝐜𝐨𝐬( 𝒂 − 𝒃 ) = 𝐜𝐨𝐬 𝒂 · 𝐜𝐨𝐬 𝒃 + 𝐬𝐢𝐧 𝒂 · 𝐬𝐢𝐧 𝒃
tan a - tan b
𝐭𝐚𝐧( 𝒂 − 𝒃 ) =
1 + tan a · tan b
𝒂 𝒃 𝒄
= =
̂
𝒔𝒊𝒏 𝑨 ̂
𝒔𝒊𝒏 𝑩 ̂
𝒔𝒊𝒏 𝑪
𝒂 𝟏−𝒄𝒐𝒔 𝒂
sin = √ ̂
b2 = a2 + c2 – 2·a·c·cos 𝑩
𝟐 𝟐

𝒂 𝟏+𝒄𝒐𝒔 𝒂
cos = √ ̂
a2 = b2 + c2 - 2·b·c·cos 𝑨
𝟐 𝟐

𝒂 𝟏−𝒄𝒐𝒔 𝒂 𝟏−𝒄𝒐𝒔 𝒂
tan = = √ ̂
c2 = a2 + b2 – 2·a·b·cos 𝑪
𝟐 𝒔𝒊𝒏 𝒂 𝟏+𝒄𝒐𝒔 𝒂

𝟏
sin a · cos b = [ sin ( a + b ) + sin ( a – b ) ]
𝟐
𝟏
sin a · sin b = [ cos ( a – b ) – cos ( a + b ) ]
𝟐
𝟏
cos a · cos b = [ cos ( a – b ) + cos ( a + b ) ]
𝟐

𝑨+𝑩 𝑨−𝑩
sin A + sin B = 2·sin · cos
𝟐 𝟐
𝑨−𝑩 𝑨+𝑩 𝟏
sin A - sin B = 2·sin · cos S= ̂
a·b·sin 𝑪
𝟐 𝟐 𝟐
𝑨+𝑩 𝑨−𝑩 𝟏
cos A + cos B = 2·cos · cos S= ̂
b·c·sin 𝑨
𝟐 𝟐 𝟐
𝑨+𝑩 𝑨−𝑩 𝟏
cos A - cos B = -2·sin · sin S= ̂
a·c·sin 𝑩
𝟐 𝟐 𝟐
Exercicis
2. Expressa en graus: 14. El cos=3/5 i  és un angle del quart
 3 quadrant, calcula la tg.
a) b)
15 10
15. La tg =3 i és un angle del tercer
7 11
c) d) quadrant, calcula el cos.
12 6
3. Troba amb la calculadora les següents 16. L’apotema d’un polígon regular de 9
raons trigonomètriques arrodonint a les costats mesura 15 cm, calcula el costat.
centèsimes:
a) sin 25º b) cos 67º 17. El costat d’un hexàgon regular mesura
c) tg 225º d) tg 150º 30 cm, calcula l’apotema.

4. Un angle d’un triangle rectangle mesura 18. L’apotema d’un octògon regular mesura
47º i el catet oposat 8 cm, troba la 30 cm, calcula l’àrea del polígon.
hipotenusa.

5. La hipotenusa d’un triangle rectangle 19. La longitud del radi d’un pentàgon
mesura 26 cm i un angle 66º. Calcula regular és 15 cm. Calcula l’àrea.
els catets.
20. L’ombra d’un arbre quan els raigs del
6. Un angle d’un triangle rectangle mesura
sol formen amb l’horitzontal un angle de
44º i el catet adjacent 16 cm, calcula
36º, mesura 11 m. Quina és l’altura de
l’altre catet.
l’arbre?.
7. En un triangle rectangle els catets 21. El fil d’un estel mesura 50 m de llarg i
mesuren 15 i 8 cm, troba els angles forma amb l’horitzontal un angle de 37º,
aguts. a quina altura vola l’estel?
h
8. La hipotenusa d’un triangle rectangle 22. Per mesurar l’altura
mesura 45 cm i un catet 27 cm, calcula d’un edifici es mesuren
els angles aguts. els angles d’elevació
des de dos punts
9. En un triangle isòsceles els angles iguals separats per 100 m.
mesuren 78º i l’altura 28 cm, troba el 33º 46º
Quina és l’altura si els
costat desigual angles són 33º i 46º? 100

10. Els costats iguals d’un triangle


23. Dues persones que es
isòsceles mesuren 41 cm i els angles
troben separades per
iguals 72º, calcula l’altre costat. h
840 m, veuen alhora
11. El cosinus d’un angle agut és 3/4, un avió amb angles
60º 47º
calcula el sinus de l’angle. d’elevació respectius
de 60º i 47º, a quina 840
altura vola l’avió?
12. La tangent d’un angle agut és 12/5, 24. Per mesurar l’altura d’una muntanya es
calcula el sinus. mesuren angles d’elevació des de dos
punts separats per 480 m i situats a
1200 m sobre el nivell del mar. Quina és
l’altura si els angles són de 45º i 76º?
Trigonometría 4t ESO
EXERCICIS DE TRIGONOMETRIA .
1. Calcula la mesura de l’angle C i dels costats b i c, coneixent que l’angle A mesura 90°, el angle
B mesura 73° y la longitud del costat a =15 cm.

2. Calcula la mesura dels angles B i C i el costat b d’un triangle rectangle, coneixent que l’angle
A mesura 90°, a = 91 cm i c =35cm.
3. Els catets d’un triangle rectangle mesuren 12 i 6 metres. Calcula l’hipotenusa i l’àrea
del triangle.
4. Quina és la longitud de l’ombra projectada per un edifici de 20m d’altura quant el sol s’eleva
20° sobre l’horitzont.
5. Una escala de ma està recolzada contra la paret d’un edifici, de manera que del peu de
l’escala a l’edifici hi ha 12 cm. A quina altura del sol es troba l’extrem superior de l’escala, i
quina és la longitud d’esta si forma un angle de 70° amb el sol.
6. Una escala de 20 metros està recolzada sobre una torre que també mesura 20 metres. El peu de
l’escala es troba a 12 metres de distància de la base de la torre. Que li falta a l’escala per arribar
a la part més alta de la torre? Quins angles forma l’escala?
7. Un edifici de 100 m. D’altura projecta una ombra de 120 m. de longitud. Quin angle d’elevació
te el sol en eixe moment?
8. Un home recorre 500m. al llarg d’un camí que te per inclinació 20° respecte de l’horitzontal.
¿Quina altura aconsegueix respecte al punt de partida?
9. Un arbre trencat pel vent, forma un triangle rectangle amb el sol. ¿Quina era l’altura de l’arbre, si
la part que ha caigut cap al sol forma amb este un angle de 50°, i la part del tronc que ha quedat
dreta te una altura de 20 cm.
10. Des d’un punt d’una vall observem el cim d’una muntanya amb un angle de 7° 49'. Després de
caminar en línia recta cap a la muntanya 500 m. tornem a mesurar l’angle i ara s de 8° 12 '.
Calcula la altura de la muntanya.
11. La Torre Eiffel es troba en una gran avinguda. Situats en un lloc de l’avinguda observem la
torre amb un angle de 15°. Si ens acostem 500 m. en línia recta cap a la torre la tornem a
observar amb un angle de 24° 40'6". Calcula la altura de la torre.
12. Des d’un punt A a la vorera d’un riu es veu un arbre just enfront. Si caminem 100 m. riu avall,
per la vorera recta del riu, arribem a un punt B des del qual es veu el pi formant un angle de 30º
amb la vorera nostra, calcular l’amplària del riu.
Trigonometría 4t ESO

13. Des d’un punt s’observa un edifici del que la part més alta forma amb el sol un angle de 30º, si
avancem 30 metres, l’angle passa a ser de 45º. Calcula l’altura de l’edifici.

14. Un edifici projecta una ombra de 150 m. quan el sol forma un angle de 20º30’ sobre l’horitzó,
calcular l’altura de l’edifici.

15. Amb les dades que ofereix la figura, calcula l’altura de l’arbre.

16. Calcula h i x.

17. Calcula l’altura i el costat desconegut.

18. Des del lloc on em trobe, la visual al penell de la torre forma un angle de 52º amb l’horitzontal. Si
m’allunye 25 m., l’angle és de 34º. Quina és l’altura de la torre?
RAONS TRIGONOMÈTRIQUES D'UN ANGLE AGUT
Sigui α un angle agut (positiu i més petit de 90º), de vèrtex O.
Qualsevol perpendicular a un dels costats determinarà un
triangle rectangle (tots seran semblants). O α

Definim el SINUS de l'angle α com el quocient entre el costat oposat a l'angle i la


hipotenusa. B
costat oposat AB
sin = =
O
α
hipotenusa OB
A
Si el costat AB medeix 5 cm i la hipotenusa 13 cm, podem determinar el valor del sinus

5 cm
sin = =0.3846
13 cm

El valor del sinus d'un angle no depèn de la longitud dels costats del triangle. Per un
mateix angle, siguin quins siguin els costats del triangle, el sinus sempre tindrà el
mateix valor.

Definim el COSINUS de l'angle α com el quocient entre el costat contigu de l'angle i la


hipotenusa

costat contigu OA
cos = =
hipotenusa OB

Si el costat OA és 12 cm i la hipotenusa 13 cm, podem determinar el valor del cosinus

12 cm
cos = =0.9231
13 cm

El valor del cosinus no depèn de la longitud dels costats del triangle. Per un mateix
angle el cosinus valdrà sempre el mateix.

Definim la TANGENT de l'angle α com el quocient entre el costat oposat a l'angle i el


costat contigu.

costat oposat AB
tg = =
costat contigu OA

També podem definir la TANGENT com el quocient entre el sinus de l'angle i el seu cosinus.
AB
sinus  OB AB
tg = = =
cosinus  OA OA
OB Quan val la tangent de l'angle α del dibuix anterior?

El valor de la tangent d'un angle no depèn de la longitud dels costats del triangle. Per
un mateix angle la tangent val sempre el mateix.

Raona per què els valors del sinus i del cosinus han d'estar necessàriament entre 0 i 1?
1. Sabent que sin α = 0.52, calculeu quan mesura el segment OB? I el segment OB'? I quan val el
cosinus de α? Quina relació hi ha entre el sinus i el cosinus d'un mateix angle?

B'
B
6
3
O α A'
A

2. Per un cert triangle rectangle, et donen la següent informació: sin α = 0.82 i cos β = 1.12. ( α i
β són els angles aguts). Poden ser totes dues correctes? Per què? Podries corregir un dels dos
valors per tal que les dues dades puguin ser correctes?

3. Troba les raons trigonomètriques dels angles Β i C dels triangles següents. I obté el valor
d'aquests angles en graus usant la calculadora.
4. Calcula les raons trigonomètriques de cadascun dels angles del triangle que formen l'Avinguda
Diagonal, el Passeig de Sant Joan i la Gran Via de les Corts catalanes.
Troba aquests angles amb la calculadora.
5. Uns excursionistes volen saber la distància en línia recta que separa el punt B del punt J i la
distància de C a G passant pel punt E. Degut a les dificultats del terreny, només han pogut
mesurar exactament les distàncies (en km) i els angles indicats d'una triangulació que es mostra
en l'esquema següent. Ara que ja sabeu una mica de trigonometria, podeu ajudar els
excursionistes a trobar les distàncies esmentades?

Per exemple, per trobar la distància de AC faríem:

BC
sin =  BC = AC sin =13.93⋅0.36=5.01
AC

Ara us toca a vosaltres calcular les distàncies que falten i fer el total.
Solució
LES MESURES DELS ANGLES: ELS RADIANTS

Usualment hem utilitzat els graus sexagéssimals per mesurar els angles:

angle nul: 0º, angle recte: 90º, angle pla: 180º, tres rectes: 270º, volta completa: 360º

La mesura científica més emprada és el radià

1 radià és un angle que abraça un arc d'igual longitud que el radi que hem utilitzat per traçar-lo

Per mesurar un angle en radiants hauríem de dividir la longitud de l'arc per la longitud del radi. Però a la
pràctica utilitzarem les equivalències i els corresponents factors de conversió.

Equivalències graus-radiants:

Un angle de 360º compren un arc de longitud 2pR, que dividit per R dóna de resultat 2p radiants.
Així doncs 180º equivalen a p radiants i aquesta equivalència s'utilitzarà per construir els factors de
conversió:


Per passar de graus a radiants: factor de conversió
180
180
Per passar de radiants a graus: factor de conversió

Expressa els següents angles en radiants, de manera exacte: 0º, 30º, 45º, 60º, 90º, 180º, 270º,
360º. Per sexemple: 360º=2p rad.
0º 30º 45º 60º 90º 180º 270º 360º
0 rad 2  rad.
Raons trigonomètriques exactes dels angles de 30º, 45º i 60º

Dibuixant un triangle equilàter i la seva altura obtenim un triangle


rectangle (en realitat dos) que te els angles aguts de 30º i 60º
respectivament.

Suposant que el costat d'aquest triangle equilàter mesura 2 cm,


calcula les longituds dels costats del triangle rectangle i aplica les
definicions per obtenir els valor exactes del sinus, el cosinus i la
tangent de 30º i de 60º.

Dibuixant un quadrat i la seva diagonal obtenim dos triangles


rectangles equilàters amb els angles aguts de 45º
Suposant que el costat del quadrat mesura 1 cm, calcula les
longituds dels costats del triangle rectangle i calcula el valor exacte
del sinus, el cosinus i la tangent de 45º

30º 45º 60º

Sinus

Cosinus

Tangent
Relacions entre les raons trigonomètriques

El sinus d'un angle i el cosinus d'un mateix angle estan relacionats, sabent un podem calcular
l'altre. Quina és la relació que guarden?

Imaginem un triangle rectangle amb l'angle agut a i la hipotenusa igual a 1m aleshores la longitud
dels catets oposat i contigu mesuren exactament igual al sinus i al cosinus d'aquest angle (raona
perquè). Aplica el teorema de Pitàgores i tindràs la relació buscada:

co co
sin(a) = h = 1 =co
cc cc
cos(a) = = =cc
h 1
El teorema de Pitàgores sabem
que hi ha la relació:

co2+cc2=h2

i substituint co per sin(a)


cc per cos(a) i h per 1 obtenim la
relació fonamental de la
trigonometria:

sin2(a)+ cos2(a)= 1

co sin
De la definició de tangent obtenim: tan = = , la segona relació entre raons
cc cos
trigonomètriques.

Exercici 1: Sabent que el sinus d'un angle val 0,3451 calculeu el seu cosinus i la seva tangent
sense fer ús de les tecles de trigonometria de la calculadora (heu de fer ús de les relacions entre
raons).

Exercici 2: Sabent que la tangent d'un angle val 2,4561 calculeu el seu sinus i el seu cosinus
sense fer ús de les tecles de trigonometria de la calculadora (heu de fer ús de les relacions entre
raons).
PROBLEMES DE TRIGONOMETRIA

1. La hipotenusa d’ un triangle rectangle mesura 10 m i un dels seus angles aguts,


20º. Quina és la mida real del catet oposat a aquest angle?

2. Un dels angles aguts d’ un triangle rectangle mesura 70º i el catet contigu 15 m.


Quant mesura l’altre catet?

3. La hipotenusa d’ un triangle rectangle mesura 8 cm i un catet, 4 cm. Troba la


longitud de l’altre catet i les mesures dels angles aguts.

4. Un ciclista puja per un pendent que té una inclinació respecte a l’ horitzontal de


25º. Quan arriba al cim del pendent, ha recorregut 1.200 metres. A quina alçada
es troba el cim?

5. Un camí de muntanya puja 35 m en una distància horitzontal de 100 m. Calcula


la tangent de l’ angle que forma el camí amb l’ horitzontal. Si el camí pugés 35 m
en una distància horitzontal de 60 m, quina seria la tangent de l’angle?

6. Un far té una alçada de 20 m i des d’ un vaixell s’ observa el seu punt més alt sota
un angle de 22º. A quina distància es troba el vaixell del far?

7. Podries saber l’ altura d’una torre si la seva ombra fa 5 m i els raigs del sol fan un
angle de 65º amb l’horitzontal?

8. Quina és la longitud de l’ombra que fa un arbre de 4 metres quan el sol es troba


a una altura de 60º sobre l’horitzó?

9. Busca l’ alçada a què arriba una escala de 8 m repenjada en una paret i formant
un angle de 45º amb el terra.

10. Calculeu la distància que separa un observador i el cim d’una paret de 59 m


d’alçada, sabent que l’angle d’elevació és de 46º.

11. Dues torres iguals estan separades per una distància d’1 km. Des del cim d’ una
torre es veu la base de l’altra torre amb un angle de depressió de 5º. Quina alçada
tenen les torres?

12. Troba l’ altura d’ una antena sabent que a una distància de 18m es veu la part
superior de l’antena sota un angle de 30º.

13. La base d’ un triangle isòsceles mesura 64cm, i l’ angle que es forma entre els
costats iguals és de 40º. Calcula el perímetre i l’àrea del triangle.

14. Un tronc de 6,2m està repenjant a una paret i forma amb el terra un angle de
55º. A quina altura de la paret es troba repenjat? Calcula la distància des de l’extrem
inferior del tronc fins a la paret.

15. Un arbre projecte una ombra de 16,75m quan l’ angle d’ elevació del sol és de
32º. Calcula l’altura de l’arbre.

-Pàgina 1-
16. Una persona de 176cm d’ alçada projecte una ombra de 121cm. Calcula l’ angle
del sol respecte l’horitzó.
17. Un estel està unit al terra per un fil de 100m que forma amb l’ horitzontal del
terreny un angle de 60º. Suposant que el fil està tensat, troba l’ altura del cometa.

18. Des d’ un far col·locat a 40m sobre el nivell del mar l’ angle de depressió d’ un
vaixell és de 55º. A quina distància del far es troba el vaixell?

19. En un tros de carretera la inclinació és de 6º. A quina alçada puja la carretera en


42m mesurats sobre la mateixa carretera?

20. Una escala de mà està repenjada a la paret d’ un edifici, de manera que del peu
de l’ escala a l’edifici hi ha 12m. A quina altura del terra es troba l’ extrem superior
de l’escala i quina és la seva longitud si aquesta forma un angle de 70º amb el terra?

21. Un home recorre 500m al llarg d’ un camí que té una inclinació de 20º respecte
l’horitzontal. A quina altura arribarà respecte el punt de partida?

22. Un arbre trencat pel vent, forma un triangle rectangle amb el terra. Quina és
l’alçada de l’arbre, si la part que ha caigut al terra forma amb aquest un angle de 50º,
i si la part del tronc que ha quedat en peu té una altura de 20m?

-Pàgina 2-
Problemes Trigonometria
(1) Calcula els quatre angles interiors d’un trapezi isòsceles, del que saps que
la base més gran fa 60 cm., la més petita 40 cm. i l’ alçària 24 cm.
(2) Determina l’ àrea d’ un triangle isòsceles del qual saps que el costat
desigual fa 10 cm. i l’ angle desigual 30º.
(3) Calcula en graus i en radians l’angle central i l’angle interior d’un polígon
regular de 18 costats.
(4) Quin angle formen els raigs solars amb l’horitzontal del terra si sabem que
a una determinada hora un xiprer de 15 m d’alçària projecta una ombra de 6
m?
(5) Des del terra dos radars, separats una distància de 20 km, observen un
avió, situat en llur mateix pla vertical, sota angles de 36º i 52º. A quina altitud
vola l’avió?
(6) Si la tangent d’un angle agut “a” és 3/2, emprant algunes de les relacions
entre les raons trigonomètriques d’un angle agut, calcula el valor exacte del
seu sinus, cosinus i de la seva secant.
(7) Calcula l’àrea, en dm2, d’un triangle equilàter de 12 cm de costat.
(8) En un triangle tenim un angle de 65º i un de 35º. Quina és la mesura
exacta en radiants del tercer angle? De quin tipus de triangle es tracta?
(9) Des d’un punt veiem un núvol amb un angle d’elevació de 50º. Si tirem 2
km enrere, veiem el núvol sota un angle de 20º.
a) Calcula l’altura del núvol.
b) Suposant que el núvol es manté fix, quant ens haurem de desplaçar per
observar-lo amb un angle que sigui la meitat de l’angle inicial?
c) És possible que trobem dues solucions? Raona la resposta.
𝛑
(10) L’angle d’un arc d’una circumferència de radi 9 cm fa rad. Quina és la
𝟔
longitud d’aquest arc?
𝟑
(11) Un arc d’una circumferència fa 15 cm i comprèn un angle de rad. Quant
𝟓
fa el radi de la circumferència?
(12) El cosinus d'un angle agut d'un triangle rectangle fa 0,8 i el catet oposat
fa 6 cm. Calcula quant fan els altres costats.
(13) Els catets d'un triangle rectangle fan 12 i 35 cm. Calcula el sinus i el
cosinus dels angles aguts d'aquest triangle.
(14) El sinus de l'angle agut  fa 0,6. Determina el cosinus i la tangent d'aquest
angle.
EXERCICIS PER PRACTICAR:
Observació important: com més lluny estem d’ un objecte, més petit és l’ angle
d’ elevació (?); el mateix passa amb l’ angle de depressió (?).
Advertiments: tots els exercicis, on intervenen únicament els angles de 30º,
45º o 60º, s’ han de fer sense emprar la calculadora; en els problemes
geomètrics i de càlcul d’altures i distàncies, que no ho tenen, és obligatori i
recomanable, abans de començar, fer una representació gràfica esquemàtica
d’acord amb el que diu l’enunciat i anotar-hi clarament les dades (nombres)
que disposem i que busquem (lletres).
1. Expressa en graus:

a) 𝝅/15 b) 𝟑𝝅/10
c) 𝟕𝝅/12 d) 𝟏𝟏𝝅/6
2. Un angle d’un triangle rectangle mesura 47º i el catet oposat 8 cm, troba la
hipotenusa.

3. La hipotenusa d’un triangle rectangle mesura 26 cm i un angle 66º. Calcula


els catets.

4. Un angle d’un triangle rectangle mesura 44º i el catet adjacent 16 cm,


calcula l’altre catet.

5. En un triangle rectangle els catets mesuren 15 i 8 cm, troba els angles aguts.

6. La hipotenusa d’un triangle rectangle mesura 45 cm i un catet 27 cm, calcula


els angles aguts.

7. En un triangle isòsceles els angles iguals mesuren 78º i l’altura 28 cm, troba
el costat desigual.

8. Els costats iguals d’un triangle isòsceles mesuren 41 cm i els angles iguals
72º, calcula l’altre costat.

9. El cosinus d’un angle agut és 3/4, calcula el sinus de l’angle.

10. La tangent d’un angle agut és 12/5, calcula el sinus.

11. L’apotema d’un polígon regular de 9 costats mesura 15 cm, calcula el


costat.
12. El costat d’un hexàgon regular mesura 30 cm, calcula l’apotema.

13. L’apotema d’un octògon regular mesura 30 cm, calcula l’àrea del polígon.

14. La longitud del radi d’un pentàgon regular és 15 cm. Calcula l’àrea.

15. L’ombra d’un arbre quan els raigs del sol formen amb l’horitzontal un angle
de 36º, mesura 11 m. Quina és l’altura de l’arbre?

-Pàgina 15-
16. El fil d’un estel mesura 50 m de llarg i forma amb l’horitzontal un angle de
37º, a quina altura vola l’estel?

17. Per calcular l’altura d’una muntanya es mesuren angles d’elevació des de
dos punts separats per 480 m i situats a 1200 m sobre el nivell del mar. Quina
és l’altura si els angles són de 45º i 76º?

18. Per calcular l’altura d’un edifici es mesuren els angles d’elevació des de
dos punts separats per 100 m. Quina és l’altura si els angles són 33º i 46º?

33º 46º
100 m

19. Dues persones que es troben separades per 840 m, veuen alhora un avió
amb angles d’elevació respectius de 60º i 47º, a quina altura vola l’avió?

60º 47º
840 m

20. La longitud del fil que subjecta un estel és de 15 metres. Si l’angle


d’elevació d’aquest és de 30º, a quina altura vola l’estel?

21. El pilot d’un avió observa la torre de control d’un aeroport amb un angle
de depressió de 30º. Vint segons més tard, l’angle de depressió obtingut sobre
el mateix punt és de 45º. Sabent que vola horitzontalment i a una velocitat de
900 km/h, calcula l’altitud del vol.

22. Des d’un cert punt es veu la part més alta de la torre d’un castell sota un
angle de 60º. Reculant 250 m en línia recta amb la torre, l’angle esdevé de
30º. Quina és l’altura d’aquesta torre?

23. Troba l’altura d’un arbre que projecta una ombra de 7m, sabent que la
inclinació dels raigs del sol en aquell moment és de 45º.

24. Situats a una certa distància del peu d’un arbre vertical, l’angle de visió és
de 60º. Quin serà l’angle si ens col·loquem a una distància triple?

25. Una marquesina forma un angle de 60º amb la vertical. Si la seva longitud
és de 2 m, quina ombra farà si els raigs del sol cauen verticalment?

-Pàgina 16-
26. Un observador vol calcular l’ altura d’ un gratacel de NovaYork. Mesura
l’ angle que forma l’ horitzontal amb la visual al punt més alt del gratacel i
resulta 32º. S’ acosta 300 m. al gratacel i torna a mesurar l’ angle des de la
nova posició; ara és de 44º. Quina altura té el gratacel? R=525,1m.

27. Resol un triangle isòsceles, sabent que el costat desigual fa 6 cm. i l’angle
oposat 54º.

28. Volem mesurar l’ amplada d’ un riu. A una distància de 25 m. d’ una de les


ribes hi ha un campanar de 35 m. d’ altura. Pugem al campanar i observem
l’ angle que formen les visuals que van cap a una riba i cap a l’ altra, que és
de 20º. Calcula amb aquestes dades l’ amplada del riu. R=27 m. aprox.

29. Troba l’ àrea d’ un pentàgon regular el costat del qual fa 12 m.

30. Tenim un rombe les diagonals del qual fan 15 cm. i 6 cm. Calcula’n els
angles.

31. Per assegurar un pal a terra se’l subjecta en sentits oposats amb dos
cables a dos punts separats 20 m. Els cables formen amb el terra angles de
75º i 60º. Troba l’ alçària del pal.

32. Els dos braços d’ un compàs mesuren 11 cm. Si l’ obrim un angle de 30º,
quant mesura el radi de la circumferència que podem dibuixar? R=5,68 cm.

33. El circ és a la ciutat i s’ ha d’ instal·lar. L’ especialista a muntar-lo encara


no ha arribat i els altres no saben la quantitat de cable d’ acer que necessiten.
El més espavilat recorda que, un cop tensat el cable des de l’ extrem del pal
principal fins a un punt determinat del terra, amb el qual forma un angle de
60º, calen dos metres més de cable que si forma amb el terra un angle de 70º.
En total han de posar sis cables tensats formant amb el terra un angle de 60º.
Quants metres de cable necessiten? R=153,06 m. aprox.

34. Volem penjar un llum a una certa distància del sostre d’ una habitació. Per
fer-ho, agafem una corda, hi lliguem el llum i la clavem pels extrems en dos
punts del sostre, separats per una distància de 140 cm., de manera que els
angles entre la corda i el sostre són de 40º i 60º a cada un dels extrems.
a) Quina serà la longitud total de la corda?
b) A quina distància del sostre quedarà el llum?
R: longitud corda=214,48 cm.; distància del sostre= 79,12 cm.

35. Quan l’ altura del sol sobre l’ horitzó baixa de 60º a 50º, la longitud de
l’ombra d’una torre s’ allarga 10 m. Calcula l’ alçada de la torre, suposadament
vertical. R=38,11 m.

36. Existeix un angle agut tal que el seu sinus sigui 2/3 i a la vegada la seva
tangent sigui 2? Raona la teva resposta.

37. Calcula raonadament el valor real exacte (sense calculadora) d’aquesta


expressió:
3 . sin250º + 3 . cos250º - 2 – sin220º - sin270º

-Pàgina 17-
38. Els dos angles aguts d’ un rombe mesuren 60º i la diagonal petita té una
longitud de 12 cm. Dibuixa el rombe amb aquestes dades i calcula el seu
perímetre i la seva àrea.

39. Es vol trobar l’ alçada d’ una torre AB de peu inaccessible. Per fer això, des
de dos punts P i Q, alineats amb el peu de la torre, es prenen les mides que
mostra la figura.
B

34º 72º
P 50m Q A

Troba l’ alçada de la torre, o sigui AB.

40. Troba l’ àrea del triangle de costats 8 m i 10 m i l’angle comprés de 27º.

41. Un globus aerostàtic està lligat a terra amb dues cordes tenses, que
formen amb l’ horitzontal del sòl angles de 65º. Si la distància entre els dos
punts on són lligades les cordes és de 5 m., troba l’ alçada del globus a terra i
la longitud de les cordes.

42. Al terrat d’ un edifici hi ha instal·lada una antena de telefonia mòbil. Des


d’ un punt P del carrer, l’ angle entre l’ horitzontal i la línia que va de P cap a
l’ extrem superior de l’ antena és de 34º. Ens apropem fins a un punt Q que és
15 metres més a prop de l’ edifici i ara l’ angle entre l’ horitzontal i la línia que
apunta cap a l’ extrem superior de l’ antena és de 42º, mentre que l’ angle
entre l’ horitzontal i la línia que apunta cap a l’ extrem inferior de la mateixa
antena és de 35º.
a) Feu un esquema de la situació, marcant molt clarament quins són els
angles i la distància que es donen a l’enunciat.
b) Calculeu l’altura de l’antena (x) i l’altura de l’edifici (h).

43. Calcula els quatre angles interiors del trapezi isòsceles de la figura, del
que saps que la base més gran fa 60 cm., la més petita 40 cm. i l’ alçària 24
cm.
A B

C D

-Pàgina 18-
44. Dibuixa un triangle rectangle amb un angle agut de 55º i calcula, aplicant
la definició, el seu sinus, cosinus i la seva tangent.

45. Els dos braços d’un compàs formen un angle de 60º. Si cada un d’ells
mesura 10 cm., troba el radi “r” de la circumferència que podem dibuixar.
Comença fent una representació gràfica del problema, amb totes les dades de
l’ enunciat.

46. L’ombra d’ un pal és de 2m. Els raigs del sol formen un angle de 25º amb
el terra. Dibuixa aquesta situació amb les dades, col·locant-les correctament,
i desprès calcula l’ alçada “h” del pal.

47. Completa mentalment la taula.

RADIANS ∏/2 3∏/2 2∏


GRAUS 30º 120º 60º 180º 45º 150º

48. Calcula el radi d’una circumferència, si sabem que un angle central de 3


radians abraça un arc de circumferència de 15 metres.

49. La longitud d’una circumferència és 11 cm. Calcula la longitud exacta de


l’arc que abraça un angle central de 70º. Raona la teva resposta.

50. Troba l’altura exacta d’un gratacels, sabent que des de la finestra d’un
edifici proper es veu el punt més alt formant un angle de 60º amb la visual
horitzontal (angle d’elevació), i el punt on l’edifici toca a terra sota un angle
de 30º amb la visual horitzontal (angle de depressió). L’altura de la finestra
d’observació és de 50 m sobre el terra.

51. Calcula en radians els angles complementari i suplementari de 2/5 rad.

52. Troba raonadament, sense emprar la calculadora, el valor exacte de


l’ expressió: sin2 19º + sin2 10º + cos2 40º + sin2 80º + cos2 50º + cos2 19º.

53. Si l’ angle agut “a” té per secant 3/2, emprant les relacions entre les raons
trigonomètriques, calcula el valor exacte del seu sinus, cosinus i tangent.

54. Sigui el triangle ABC de la figura:

c b=2cm.

45º 30º
B a C

Calcula el valor exacte (sense decimals) de: l’ angle A , l’ altura AH ,


el costat “c” , el costat “a” , el perímetre del triangle, l’ àrea del triangle.

-Pàgina 19-
NOM: DATA:

TRIGONOMETRIA per angles majors de 90º

Omple els requadres amb els angles en graus i en radiants.

Escriu a dins de cada quadrant els signes + o – segons el signe de les raons
trigonomètriques

sin cos tan


Escriu als requadres els angles corresponents si el menor és α

Si sin α l'anomenem M i cos α l'anomenem N tots els altres sin i cos es poden
expressar com +M, -M, +N o -N. Omple la taula
angle sinus cosinus
α +M +N
TRIGONOMETRIA 5

Activitats complementàries
NOM CURS CLASSE

ACTIVITATS NÚM.

2
Comença fent els exercicis que no tenen *. Els d’ampliació són els 4. El cosinus d’un angle A val . Calcula el sinus i la tangent
que tenen *. Deixa’ls per al final. 5
d’aquest angle, utilitzant les fórmules treballades durant la
unitat de trigonometria.
1. Calcula quant mesura el catet d’un triangle rectangle si sa-
bem que la hipotenusa i l’altre catet mesuren 29 i 21 cm res- 5. * La tangent d’un angle γ val 5. Calcula’n el sinus i el cosinus.
pectivament.
2
Quant val l’àrea d’aquest triangle? 6. Troba cos A i tg A, a partir del sin A, sabent que sin A  i
que l’angle A pertany al segon quadrant. 7
* I l’altura respecte la hipotenusa?
Anomenem per a la hipotenusa i per b i c els dos catets. 7. Resol aquests triangles:
` `
a)
Calcula les raons trigonomètriques dels angles B i C .
Quant mesuren els dos angles aguts? b = 5 cm

2. Els catets d’un triangle rectangle mesuren 18 cm i 40 cm res-


pectivament.
60°
a = 10 cm
Quant mesura la hipotenusa?
b)
Calcula les raons trigonomètriques dels angles aguts d’aquest
triangle. b = 5 cm
c = 2 cm

3. El catet d’un triangle rectangle mesura 15 cm i l’angle oposat


60°. Resol aquest triangle.
a = 6 cm

157
IES El Sui Matemàtiques 4t d'ESO /3

Resolució gràfica de triangles (trigonometria)

RECORDA:
• En un triangle els s’anomenen amb lletres majúscules i els costats amb minúscules de
manera que el costat a ha de quedar just al davant del angle A i mai el pot tocar (igual
amb el b i B, així com amb el c i el C:
B

c
a

A
b C

• Quant es demana resol el triangle cal:


o Dibuixar el triangle (Necessitareu un regle graduat, un compàs, un transportador i
paper mil·limetrat.)
o Mesurar els tres costats amb el regle i els tres angles amb el portaangles.
o Donar la resposta per escrit de manera clara i ordenada.

Recordeu que:
Resoldre un triangle vol dir conèixer els seus tres costats i els seus tres angles.
A continuació analitzarem un per un tots els casos diferents amb els que et pots trobar per
quant necessitis resoldre un triangle.

Si tinc els tres costats


• Escriu les instruccions necessàries de manera molt clara com s’ha de fer per
dibuixar un triangle quan coneixes els tres costats. (pots ajudar-te del dibuix)

c
a
b

b a

A c B

• Resol el triangle format pels costats a = 7 cm, b = 5,3 cm i c =3 cm.


IES El Sui Matemàtiques 4t d'ESO /4

• Creus que podríeu dibuixar més d'un triangle amb aquestes condicions o tots
serien iguals?

Si tinc dos angles i un costat


• Instruccions:

o Primer calcula el tercer angle amb la calculadora, recorda que els tres
angles d’un triangle sumen 180 º

o Dibuixa el segment i amb vèrtex en els extrems situeu els angles

A B

A B
c

• Resol el triangle en que coneixem dos angles A=32º i B=94º, i el costat c = 7.

• Creus que podríes dibuixar altres triangles que complissin les mateixes
condicions? per què?

Si tinc dos costats i l’angle contigu als dos costats

• Escriu les instruccions necessàries de manera molt clara com s’ha de fer per
dibuixar un triangle quan coneixes dos costats i l’angle contigu als dos costats.
(pots ajudar-te del dibuix)

A
c
IES El Sui Matemàtiques 4t d'ESO /5

• Resol el triangle format pels costats a = 6 cm, b = 8,3 cm i C =30º.

• Creus que podríeu dibuixar més d'un triangle amb aquestes condicions o tots
serien iguals?

Si tinc dos costats i l’angle oposat a un dels costats.

• Escriu les instruccions necessàries de manera molt clara com s’ha de fer per dibuixar
un triangle quan coneixes dos costats i l’angle oposat a un dels costats . (pots ajudar-te
del següent dibuix)

• Amb la construcció anterior Resol el triangle a=5,1 cm, b=8,3 cm, A=32º.

• Amb la construcció anterior Resol el triangle a=2 cm, b=8,3 cm, A=53º.

• Amb la construcció anterior Resol el triangle a=10 cm, b=3,6 cm, A=41º.

• Hauràs observat que pot ser que no hi hagi cap solució que n’hi hagi una o dues.
Explica a la teva llibreta com han de ser les dades per a que no hi hagi solució, com
han de ser por a que hi hagi una solució i com han de ser per a que n’hi hagi dues.

1.Resolem alguns triangles


a) Dibuixeu el triangle del qual coneixem:

A
B
IES El Sui Matemàtiques 4t d'ESO /6

b) Dibuixeu el triangle del qual coneixem:

c) Dibuixeu un triangle de forma que un costat amidi 7,2 cm i els angles adjacents
siguin de 70º i 40

2.Recordem els canvis d’escala


Recorda que per passar del dibuix a escala a la realitat cal multiplicar per l’escala, per passar
de la realitat al dibuix cal dividir per l’escala i per trobar l’escala cal dividir la realitat entre el
dibuix a escala.
Completa la taula següent

DIBUIX REALITAT ESCALA


14 cm 700000 cm = 7 Km. 1:50 000
2 cm 1:100
42 m 1:1000
7 cm 1:125
12 cm 480 m
14,7 m 1: 70

3.El vaixell
Des del cap de Creus es veu un vaixell sota un angle de 170º i des de Palamós sota un
angle de 85º. Trobeu gràficament la distància del cap de Creus al vaixell i de Palamós al
vaixell. (Com origen d'angles s'agafa sempre la línia observador-nord i el sentit
positiu de gir el de les agulles del rellotge.)
IES El Sui Matemàtiques 4t d'ESO /7

Nort
Nort Escala 1:3000.000
IES El Sui Matemàtiques 4t d'ESO /8

4.El riu
Un individu vol mesurar l’amplada d’un riu i no vol mullar-se, puja a una torre que fa 40
m d’alçada i amb un aparell mesura l’angle amb el que ell veu l’amplada del riu des de
dalt de la torre. Aquest angle és de 21º. A més sap que la torre està a 30 metres del riu.

• Dibuixa a escala la torre i el riu.

• Mesura en el teu dibuix l’amplada del riu

• Fes un altra vegada el canvi d’escala per calcular l’amplada del riu.

5.Es cotxes
Dos cotxes surten del mateix punt i en el mateix instant per carreteres que formen un
angle de 55º. El primer va a 80 km/h i el segon a 60 Km/h. Calculeu gràficament a
quina distància estaran al cap de 1h 30 m.

6.Un altre vaixell


Un vaixell observa el punt més alt d'un far, situat 20,8 m sobre el nivell del mar, sota un angle
de 40º respecte a l’horitzontal. A quina distància es troba del cim del far?

7.Fem feina per estalviar feina


La gran majoria de problemes en els que s’ha de buscar distàncies cal resoldre triangles
rectangles en els que busquem un dels costats. Per evitar fer un dibuix nou per cada exercici
farem una colecció de dibuixos, els mesurarem i apuntarem el que dona.
• Fes 8 dibuixos amb les dades que es donen a la taula següent. Mesura les dades que
falten al dibuix i apunta-les a la taula.
o A la columna c/a posa el resultat de dividir c entre a.
o A la columna b/a posa el resultat de dividir b entre a.
o A la columna c/b posa el resultat de dividir c entre b.
IES El Sui Matemàtiques 4t d'ESO /9

C A b B c a c/a= b/a= c/b=

10º 90º 10
cm
20º 90º 10
cm
30º 90º 10
cm
40º 90º 10
cm
50º 90º 5
cm
60º 90º 3c
m
70º 90º 2c
m
80º 90º 2c
m

Imagina que per fer la taula utilitzem triangles iguals pel que fa als angles, és a dir, a la
forma, però diferents en grandària. ¿Canviaria el resultat de les columnes c/a , b/a, c/b? ¿Per
què?

8.Aprofitem la feina feta


Utilitzant la taula anterior i sense fer cap dibuix pots resoldre molts problemes. Fixat en
l’exemple i resol els altres:
• Exemple: D’un triangle rectangle coneixem C = 20º i b = 7 cm he de calcular c
solució:
Sense fer el triangle sé que c/b ha de donar el mateix en meu triangle que en el de la
teula anterior. Així doncs

c
≅ 0,36
b
c
Per tant , com que b = 7 tenim que ≅ 0,36
7
i passant el 7 a l’altra banda: c = 7 · 0,36 = 2,53cm.
IES El Sui Matemàtiques 4t d'ESO /10

• D’un triangle rectangle coneixem C = 70º i b = 9 cm he de calcular c i a


• D’un triangle rectangle coneixem C = 40º i c = 12 cm he de calcular b i a
• D’un triangle rectangle coneixem C = 50º i a = 6 cm he de calcular c i b

9.L’arbre
Una prestigiosa biòloga fa un estudi sobre el creixement dels arbres de Cardedeu i ha de
mesurar l’alçada de molts arbres. Com que seria molt incòmode tenir que pujar fins dalt de
cada arbre mesura la distància del tronc a un punt i mesura desprès l’angle en que es veu el
punt més alt.

b metres

Calcula l’alçada dels arbres que ha mesurat si:

• C = 20 i b = 50 m
• C = 40 i b = 32 m

10.El gran descobriment.

Omple ara la taula següent posant el que et surti al posar amb calculadora l’angle C i prémer

les tecles sin , cos , tan o directament prémer les tecles sin , cos , tan
IES El Sui Matemàtiques 4t d'ESO /11

i l’angle C. (Segons el tipus de calculadora)

C Sin C Cos C Tan C

10º

20º

30º

40º

50º

60º

70º

80º

Explica què observes comparant aquests valors amb els de la taula de l’exercici fem feina per
estalviar feina.
¿Qué posaries, a la taula, al costat de c/a = , b/a = c/b = ?
Per tant, quina conclusió podem treure?

11.Resolem triangles amb calculadora.

Utilitzant les tecles sin , cos tan de la calculadora pots saber el valor de c/a , b/a,

c/b de qualsevol angle C d’un triangle rectangle. Amb aquesta informació, i sense dibuix:

• Calcular c d’un triangle rectangle del que coneixem C = 74,26º i b = 4 cm.

• Calcular b d’un triangle rectangle del que coneixem C = 48,33º i c = 13,5 cm

• Calcular a d’un triangle rectangle del que coneixem C = 11,37º i b = 7,2 cm.
IES El Sui Matemàtiques 4t d'ESO /12

• Calcular b d’un triangle rectangle del que coneixem C = 68,2º i a = 19,12 cm

12.El Far

A una distància de 25 m d’un far observem la part més alta d’aquest far sota un angle de 28º.
¿Quina alçada te el far?

28º

25 m

Raons trigonomètriques

Donat un angle agut de mesura C anomenarem sinus de C el valor del quocient:

c longitud del catet oposatat a l' angle C


sin C = =
a hipotenusa

Considerant que un angle agut pot ésser un dels angles d’un triangle rectangle direm:

Sinus d’un angle agut és la raó entre les longituds del catet oposat i de la hipotenusa.

Donat un angle de mesura C anomenarem cosinus de C el valor del quocient:

b longitud del catet contigu a l' angle C


cos C = =
a hipotenusa

De manera semblant al cas del sinus, podem dir:


IES El Sui Matemàtiques 4t d'ESO /13

Cosinus d’un angle agut és la raó entre les longituds del catet contigu i de la hipotenusa.

Donat un angle agut de mesura C anomenarem tangent de C el valor del quocient :

c longitud del catet oposatat a l' angle C


tan C = =
b longitud del catet contigu a l' angle C

Tangent d’un angle agut és la raó entre les longituds del catet oposat i el catet contigu.

El sin C, cos C i tg C s’anomenen raons trigonomètriques de l’angle de mesura C.

13.La rampa del supermercat


En un supermercat de dos pisos hi ha una rampa per poder traslladar els carretons fàcilment;
la rampa té una inclinació de 15º i ocupa una longitud horitzontal de 14 m. Volem saber la
llargada de la rampa

14.El xicot
Un xicot de 1,75 m d'alçada es mira el cim d'un edifici de 33 m des d'un punt situat a 50
m de la base de l'edifici. Quin angle forma la visual amb l'horitzontal.

15.L’estel
Un estel està subjecte a terra per un cordill de 80m de llarg i que forma amb l'horitzontal un
angle de 75º. A quina altura és l'estel?

16.Per concloure
Amb l’ajut del teu professor/a :

a) Dibuixa un triangle ABC rectangle en A i de costats a, b i c:

b) Expresseu en funció dels costats les raons trigonomètriques de l’angle B.

c) Igualment per a l’angle C.

d) Quant sumen els angles B i C ? Sabíeu que per això s’anomenen complementaris?
Doneu una definició d’angles complementaris.
d) Quina relació hi ha entre les raons trigonomètriques dels angles B i C?.
IES El Sui Matemàtiques 4t d'ESO /14

Problemes d’aplicació

1. Determina els angles d’un triangle rectangle si la hipotenusa fa 4 i un dels catets 2.

2.Resol els triangles rectangles següents:

C C

12

a 30º
A B
b
c

60º
A B
3

3.Resol el triangle ABC, rectangle en A, en els casos següents:

a = 12 cm C = 45º

a = 8 cm B = 60º

b = 5 cm C = 40º

c = 8 cm C = 50º

b = 6 cm B = 30º

4.Un estel està lligat a terra amb un fil de 100 m, que forma un angle de 40º amb
l’horitzontal del terreny. Suposant que el fil estigui tibant, calcula l’altura de l’estel.

5.Un globus està lligat a una corda de 200 m. Un cop d’aire el desvia 20 m de la vertical.
Calcula l’angle que forma la corda amb el terra.

6.Des d’un far situat a 40 m sobre el nivell del mar es veu un vaixell amb un angle de
depressió de 55º. A quina distància del peu del far es troba el vaixell?

7.Calcula l’àrea i el perímetre d’un triangle rectangle si un dels seus angles mesura 29,7º i
la hipotenusa 11cm.

8.Els catets d’un triangle rectangle fan 9cm i 12 cm. Calcula la hipotenusa i l’àrea.
IES El Sui Matemàtiques 4t d'ESO /15

9.Calcula l’àrea d’un pentàgon regular de 40 m de costat

10.Calcula l’àrea d’un octàgon regular de 50 m de costat

11.Calcula el costat i l’àrea d’un octàgon regular inscrit en una circumferència de 12 cm


de radi.

12.L’angle oposat a la base d’un triangle isòsceles és de 70º i la base fa 12 cm. Calcula’n
l’àrea.

13.L’angle oposat a la base d’un triangle isòsceles és de 40º i un dels costats iguals fa 16
cm. Calcula’n l’àrea.

14.Una escala de 5m recolza en una paret i l’angle que forma amb la paret és de 20º.
Calcula l’altura que s’assoleix i la separació respecte a la paret.

El mesurador d’angles

Tartàglia (1500-1557) en realitat es deia Nicolo


Fontana. Vivia a Brèscia, al nord d'Itàlia, i quan tenia
12 anys els francesos van atacar la ciutat. Al pobre
Tartàglia li van creuar la cara amb un sabre, li van fer 7
ferides a la cara, una de les quals li va afectar el coll. La
seva mare el va curar a casa com va poder i com a
conseqüència de la feriada del coll es va quedar
tartamut, d'aquí el sobrenom de Tartàglia (el tartamut, a
Itàlia). La vida d'aquest personatge no va ser gens fàcil.
Pot ser amb ganes de venjar-se dels francesos, el 1537
Tartàglia va publicar un llibre sobre l'aplicació de la
matemàtica a l'artilleria. En aquest llibre descrivia un
quadrant de la seva invenció molt semblant al que
construirem, però que, en aquest cas mesurava
directament l’angle. Com que en aquella època no hi
havia calculadores, Tartaglia va fer una taula molt semblant a la que hem fet nosaltres a
l’exercici “fem feina per estalviar feina” però amb
molts més angles. Fabricat en llautó, el quadrant
anava enganxat a la boca dels canons i amb uns
petits càlculs i la taula es podia saber a quina
distància cauria la bala en funció de la lectura de
l'aparell.
Amb una intenció més pacífica construirem
nosaltres, ara un quadrant semblant al del nostre
amic Tartàglia.
IES El Sui Matemàtiques 4t d'ESO /16

Construcció
• Enganxar una fotocòpia ampliada de mig transportador d’angles en un cartró rígid:

• Pega una canyeta en una de les vores


• Lliga un filet amb una plomada a la vora.

A proposta del professorat podeu fer aquest o un altre


treball de camp que haureu de lliurar.
IES El Sui Matemàtiques 4t d'ESO /17

Fem de biòlegs
Al pati del nostre institut tenim la sort de tenir diversos arbres catalogats. Això vol dir que no
podem decidir tallar-los sense consultar a l’ajuntament. Una de les tasques dels biòlegs que
cataloguen els arbres és mesurar-los. Mesureu, amb l’ajut del vostre mesurador d’angles, els
arbres que hi ha al pati.

Com podem saber la mesura dels angles en un triangle rectangle si


coneixem els costats i per tant les raons trigonomètriques.

a) Dibuixeu un angle el sinus del qual sigui 0,4. Trobeu la mesura d’aquest angle amb el
semicercle graduat.

b) Feu el mateix però que el cosinus sigui 0,25.

c) Feu el mateix però que la tangent sigui 3.


IES El Sui Matemàtiques 4t d'ESO /18

La màquina calculadora ens permet també trobar l’angle a partir d’una de les seves
raons trigonomètriques. Això ve donat normalment per la mateixa tecla que ens dona la
raó trigonomètrica però prement abans la tecla INV- SHIFT- 2a FUNCIÓ...

d) Trobeu la mesura dels angles dels apartats a) b) i c) mitjançant la calculadora i


comproveu la vostra habilitat a l’hora de dibuixar i mesurar.

Angle amb el que es veu el Sui des del Sui

A partir de les dades del mapa de la pàgina següent, determineu l’angle amb que es veu el Sui
des del Sui.

La torre de Pisa

La construcció de la famosa torre de Pisa va acabar l’any 1284. Es va comprovar


aleshores que la part més alta de la torre se separava uns 90 cm de la vertical. Avui dia la
separació és aproximadament de 5 m i l’altura de la torre és de 55 m . Calcula l’angle que
forma la torre amb la vertical.

Per practicar
1.Dibuixa els angles aguts que tenen les raons trigonomètriques següents i desprès
calcula el valor de l’angle.

sin B = 3/5

tg C = 1

cos C = 4/5

sin B = 0,5

2. En el triangle rectangle de la figura, un catet amida 4m i l’angle contigu


15º. Resoleu el triangle.

4m
15º
IES El Sui Matemàtiques 4t d'ESO /19
IES El Sui Matemàtiques 4t d'ESO /20

3.Resol el triangle ABC, rectangle en A, en els casos següents:

a) a= 20 cm b = 16 cm

b) a= 5 cm c = 2 cm

c) c= 10 cm b = 5 cm

4.Resoleu analíticament els triangles:

a) a = 16 ; b=3

b) a = 10 ; C = 43º

c) c = 12 ; B = 69º

Valor exacte de les raons trigonomètriques dels angles de 30º 45º i 60º

Als problemes anteriors heu utilitzat les raons trigonomètriques de diversos angles.
Primer les heu calculat mitjançant procediments gràfics i després heu fet servir els
valors donats per la màquina calculadora. Tant en un cas com en un altre eren valors
aproximats. Per als angles de 30º, 45º i 60º es poden trobar les seves raons
trigonomètriques d’una manera analítica i exacta mitjançant càlculs molt senzills,
utilitzant el teorema de Pitàgores.

El vostre professor/a farà l’exercici a la pizarra i vosaltres anotareu les respostes i


conclusions a la vostra llibreta:

a) En un quadrat de costat 1, dibuixa-hi la diagonal i calculeu-ne la longitud. A partir


d’aquest resultat trobeu el valor exacte de sin 45º, cos 45º i tg 45º.

b) En un triangle equilàter de costat la unitat, dibuixa-hi una de les altures i calcula’n la


longitud. Tenint en compte el resultat anterior, determineu sin 60º, cos 60º i tg 60º.

c) A partir de les raons trigonomètriques de 60º trobeu les d’un angle de 30º.

d) Si en comptes d’un quadrat o triangle equilàter de costat 1 haguéssiu considerat un


costat de mesura c, haurien canviat els resultats obtinguts?

Posa la teva calculadora en mode Math i calcula aquestes raons trigonomètriques. Veuràs
que algunes coincideixen amb les que ha fet el professor i altres no. ¿ Qui s’equivoca?
IES El Sui Matemàtiques 4t d'ESO /21

Per practicar
1.La diagonal d’un quadrat amida 5 2 cm. Quant amida el costat?

2.Els angles aguts d’un rombe mesuren cadascun 60º i la diagonal més gran amida
7 3 cm. Quant amida l’altra diagonal? I el costat?

Fòrmules de trigonometria

1.Hi ha una fòrmula molt important que s’anomena Teorema fonamental de


trigonometria. El vostre professor/a us lo posarà a la pissarra i us demostrarà la
seva validesa. Copia tot el que faci a la teva llibreta. ( Pensa que aquesta
demostració pot sortir als exàmens)

2. Donat el triangle rectangle de la figura

a
b

B A
c

a) Expresseu sin B i cos B en funció dels costats.


b) Utilitzant les expressions de sin B i cos B trobades a l’apartat anterior, demostreu que :

sin B
tg B =
cos B

3. Si d’un angle B sabem que sinB=0,6 , trobeu cos B i tg B.

4.. L’exercici anterior l’has resolt, segurament, seguint un dels tres mètodes següents:

Plantejant un sistema d’equacions amb incògnites cosB i tgB, a partir de les


relacions obtingudes a l’exercici 1 i 2.

Imaginant un triangle rectangle on la relació entre el catet oposat i la


hipotenusa fos de 0,6 (per exemple de mesures 6 i 10, o bé 3 i 5 etc.). A partir
d’aquest triangle es pot trobar l’altre catet mitjançant el T. de Pitàgores i les
raons trigonomètriques buscades.
IES El Sui Matemàtiques 4t d'ESO /22

Utilitzant la màquina calculadora, primer per trobar l’angle B i després per


calcular les raons que us demanen.

5.Digueu de quina manera heu resolt l’exercici 3 i torneu-lo a fer pels altres dos
mètodes que no heu utilitzat.

6.

3
a) Si cos B = trobeu sin B i tg B
5

b) Si tg B = 3 trobeu sin B i cos B.

5
c) Si tg B = trobeu sin B i cos B.
12
IES El Sui Matemàtiques 4t d'ESO /23

EXERCICIS COMPLEMENTARIS

1.Dues persones que caminen a tres quilòmetres per hora surten al mateix temps de la
cruïlla de dos camins rectes, que formen entre sí un angle de 15º. Totes dues van en
el mateix sentit. A quina distància es troben l’una de l’altra al cap de dues hores?

2.Les puntes dels braços d’un compàs es troben a 8 cm de distància i cada braç té 14
cm. Calcula l’angle que formen els braços del compàs.

L’angle que forma la visual al punt amb el pla horitzontal que passa per l’ull de
l’observador s’anomena angle d’elevació si el punt està per sobre d’aquest pla i angle de
depressió si hi està per sota.

3.El costat desigual d’un triangle isòsceles mesura 8 cm i l’angle oposat, 24º. Calcula
el perímetre i l’àrea d’aquest triangle.

4.Calcula l’àrea d’un triangle isòsceles si els costats iguals mesuren 8 cm cadascun i
formen un angle de 110,42º.

5.Les diagonals d’un rombe mesuren 66cm i 8 cm. Calcula el seu perímetre i la
mesura dels seus angles.

6.Calcula l’àrea dels polígons següents:

Un hexàgon regular de 5 cm de costat.


Un octàgon regular inscrit en una circumferència de 4 cm de radi.

7.Un quadrimotor sobrevola una pista d’aterratge arrossegant un planador. Si tots dos
estan units per un cable que forma un angle de 30º amb la horitzontal i la força que
actua sobre el planador és de 10 000 N, esbrina el valor de la component
horitzontal d’aquesta força.

F 8.Un submarí de la classe Typhoon


que navega en direcció Nord
Fv localitza en la direcció Oest i a la
mateixa profunditat dos
submergibles de la classe Alfa
separats per una distància de 10
Fh km. Mitja hora després, el seu
sonar situa un dels submarins al
Sud-oest(SO) i l’altre al Sud- sud-
IES El Sui Matemàtiques 4t d'ESO /24

oest (SSO). Esbrina la velocitat del submarí, si sabem que els submergibles no han
canviat de posició.

9.Un F-14 Tomcat sobrevola les costes de Califòrnia amb direcció Nord quan de
sobte es veu immers en una turbulència que es desplaça cap a l’Est a una velocitat
de 120 km/h. Si el reactor desenvolupa una velocitat de 1200 km/h, calcula quina
és la direcció del seu vol, es a dir, l’angle que forma la seva trajectòria amb la
direcció Est.

10.L’angle d’elevació del punt més alt d’un obelisc, observant des d’un punt del terra
situat a 42 m del peu de l’obelisc és de 30º. Calcula l’altura de l’obelisc.

11.Dos radars, A i B, disten entre ells 15 km i detecten un avió, que és en el mateix


pla vertical que ells, sota angles de 42º i 56º. Calcula l’altura a què vola l’avió i la
distància de l’avió a cadascun dels radars.

12.Un globus està subjectat al terra per un cable de 120 m de llargada, que forma amb
el terra horitzontal un angle de 37º. A quina altura es troba el globus si el cable està
tesat?

13.Des d’un far col·locat a 90 m sobre el nivell de mar, s’observa un vaixell sota un
angle de depressió de 35º. Quina distància separa el vaixell del far?

14.Un observador situat a la vora d’un riu veu un arbre situat a la riba oposada sota un
angle de 60º. Si se n’allunya 20 m, el veu sota un angle de 30º. Calcula l’alçaria
d’aquest arbre i l’amplària del riu.

15.Des d’un punt concret de terra es veu el punt més alt d’una torre formant un angle
de 30º amb l’horitzontal. Si ens acostem 75 m al peu de la torre, aquest angle és de
60º. Calcula l’altura de la torre.

16..En un punt de la calçada s’ha fixat una escala de bombers de 10 m de longitud. Si


l’escala recolza sobre una de les façanes forma un angle amb el terra de 45º, i si
recolza sobre l’altra, l’angle és de 30º. Calcula l’amplada del carrer. Quina altura
s’assoleix amb aquesta escala sobre cada una de les façanes?

17.Un observador situat a la vora d’un estany circular veu una estàtua situada a la
vora oposada sota un angle de 45º. Si se n’allunya 10 m , la veu sota un angle de
30º. Esbrina l’alçaria de l’estàtua i l’àrea que ocupa l’estany.

18.Des de la torre d’un far es veu un vaixell sota un angle de depressió de 25º. Si el
vaixell s’apropa 300 m al far, l’angle passa a ser de 39º. Calcula la distància x que
separa el vaixell del far en el moment de la segona observació i l’alçària del punt
més alt del far.

19.Des del punt més alt d’un far de 100 metres d’alçària, s’observa un vaixell que
s’acosta al far sota un angle de depressió de 42º. Al cap de 30 min, l’angle ha
passat a ser de 56º. Si el vaixell s’acosta a velocitat constant, esbrina el valor
d’aquesta velocitat. Expressa el resultat en m/s i en km/h.
IES El Sui Matemàtiques 4t d'ESO /25

20.L’angle d’elevació del punt més alt d’una muntanya des d’un punt del terra és de
45º. Si retrocedim 30m, l’angle passa a ser de 40º. Calcula l’alçària de la muntanya.

21.Un turista observa un monument des d’una certa distància sota un angle de 70º.
Sota quin angle el veurà si se n’allunya quatre vegades aquesta distància?

22.Un home situat a l’Est d’un monument n’observa el punt més alt sota un angle de
35º. S’allunya 15 m cap al Sud i l’angle passa a ser de 30º. Determina l’alçària del
monument.

23.L’angle d’elevació de l’extrem més alt de l’asta d’una bandera de 3 m d’alçària


situada en la part superior d’un edifici, des d’un punt del terra és de 37º i el de la
seva base, de 35º. Calcula l’alçària de l’edifici.

24.En un instant determinat, dos observadors, separats uns distància de 500 m, veuen
una àliga que vola en el mateix pla vertical on ells estan situats, sota angles de 35º i
52º. Esbrina a quina altura vola l’àliga.
IES El Sui Matemàtiques 4t d'ESO /26

ANEX I
EL QUADRANT
El quadrant és un dels primers aparells astronòmics inventats desprès del
senzill gnomon (un bastó clavat al terra), un rellotge de sol és, de fet, un
quadrant. A Messopotàmia coneixien ja aquest aparell. Durant la història ha
anat patint modificacions i
millores vinculades,
fonamentalment pels avanços
tecnològics. Així doncs els
primers quadrants construïts
eren de pedra o fusta. En
arribar l'Edat Medieval al
voltant de l'any 1000 el món
àrab volia millorar la precisió
d'aquests aparells, perquè hi
va haver un gran apogeu de
l'astrologia i es van començar
a construir quadrants grans, amb la qual cosa es podien fer les graduacions amb
molta més precisió.

L'ús del metall i la millora


en aquest art van permetre grans
avanços en la construcció de
quadrants pel que fa a grandària i
perfecció. A la imatge del costat
podem observar el quadrant gegant
del millor observatori astronòmic
sense telescopi de la historia,
IES El Sui Matemàtiques 4t d'ESO /27

l'observatori d'en Tycho Brahe del voltant de l'any 1600.

CONSTRUCCIÓ DEL QUADRANT

En un quadrat de cartró de 16 cm de costat dibuixa un quadrat de 15 cm de


costat:

15 cm 16 cm

15 cm

16 cm

A continuació gradua amb centímetres els 15 centímetres que han quedat


representats a la dreta i a sota el dibuix (pots enganxar la fotocòpia d'un regle).
Gradua el costat DC començant pel punt D i posant el "15" totalment a la vora
del punt C. El costat BC caldrà graduar-la començant pel punt B i posant també
el "15" a la vora del punt C

A B

D C

Des del punts A al B pega una canyeta (de les de xuclar cocacola) que et
serviran com a punts de mira per dirigir la visual.
Finalment lliga una plomada al punt A. La longitud de la corda ha de ser,
almenys, 25 cm.
IES El Sui Matemàtiques 4t d'ESO /28

Acabes de construir un quadrant.

ERRORS MÉS COMUNS EN LA CONSTRUCCIÓ DEL QUADRANT.

Cal enganxar bé la fotocòpia del regle a la fusta de manera que els petits segments que
determinen els mil·límetres estiguin just a la vora de la fusta.

Per graduar la fusta cal començar pel punt C on aniran els valors 25 cm tocant a la vora
(totalment arran de la vora). Això obliga a graduar la cara CD del quadrant al contrari
de com està graduat normalment un regle. Si utilitzes la fotocòpia d'un regle caldrà
esborrar els números i tornar a escriure la graduació.

La canyeta ha d’estar alineada amb els valors zero de les vores graduades

COM S'UTILITZA L'APARELL?:

Cal mirar per la canyeta alineant la visual amb l'objecte a mesurar. En aquest
cas movem l'aparell i deixem que la plomada ens indiqui els valors a mesurar
sobre la graduació.

En aquest cas també tindrem quatre possibilitats dues per mesurar cap a dalt i dues per
mesurar cap avall. Investiga i descriu tu mateix els quatre casos.
IES El Sui Matemàtiques 4t d'ESO /29

L’escala de seguretat
Una arquitecta ha de dissenyar una escala de seguretat per fora de l’edifici que, des de la
nostra finestra de la classe vagi fins el terra. Entra a la nostra clsse i ens demana si sabem
quina alçada hi ha entre la finestra i el terra i nosaltres li contestem que no es preocupi que ho
calcularem a classe de matemàtiques. Mesurem doncs la distància entre la finestra de la classe
i el terra:
• Des del pati busquem un lloc (anomenat A) des del que es vegi bé la finestra i
mesurem la distància d’aquest lloc “A” a la paret just a sota de la finestra.
• Des del punt A mirem la finestra per la canyeta del nostre quadrant i apuntem on
senyala el cordill. (no val utilitzar les dades del company. Cada u de vosaltres ha de
mesurar amb el vostre aparell)
• Dibuixem en un paper el triangle que ens ha sortit al quadrant.
• El dibuix anterior és un dibuix a escala de la realitat. Analitzem quina és l’escala amb
la que està fet el dibuix.
• Utilitzant l’escala i calculem l’alçada que ens demanen.

El rellotge
Imagineu que volem arreglar el rellotge que compartim amb els nostres amics els del IES
Manel Raspall i per a poder arreglar-lo hem de poder accedir al rellotge per la part de fora,
necessitarem llogar una escala o una bastida i necessitem saber amb precisió l’alçada a la que
està el rellotge del terra.
És clar que la biblioteca tant maca que tenim ens impedeix mesurar la distància d’un punt del
pati a la paret just a sota del rellotge, per tant caldrà buscar una estratègia més intel·ligent per
a poder resoldre el problema:
• Buscarem dos llocs A i B des d’on es vegi bé el rellotge i mesurarem la distància entre
aquests dos punts AB.
• Mirarem el rellotge per la canyeta del quadrant des dels dos punts i apuntarem el que
mesura.

D
A
Distància AB
B inaccesible

• Farem un dibuix a escala de la realitat utilitzant les dades obtingudes


o Començarem dibuixant el punt A
o A la distància AB dibuixarem el punt B
o Des d’A i des de B dibuixarem els angles mesurats amb l’aparell
o Allargarem les dues línies anteriors fins que es trobin en un punt. Aquest punt
és el punt C
IES El Sui Matemàtiques 4t d'ESO /30

o Acabem de dibuixar el triangle dibuixant el punt D a sota del C


C
Cal fer un dibuix
que ocupi tota la
plana d’un full

A
B D
• Ara podem mesurar l’alçada del rellotge al dibuix
• Per acabar farem el canvi d’escala
MATEMÁTICAS TIMONMATE
EJERCICIOS RESUELTOS DE TRIGONOMETRÍA Juan Jesús Pascual

TRIGONOMETRÍA

A. Introducción teórica
A.1 Razones trigonométricas de un triángulo rectángulo.
A.2. Valores del seno, coseno y tangente para ciertos ángulos significativos (en grados y radianes).
A.3. Significado geométrico de las razones trigonométricas en la esfera goniométrica.
A.4. Relaciones entre las razones trigonométricas.
A.5. Resolución de triángulos: Teoremas del seno y del coseno.

B. Ejercicios resueltos
B.1. Razones trigonométricas.
B.2. Ecuaciones trigonométricas.
B.3. Problemas.

A. INTRODUCCIÓN TEÓRICA

A.1 Razones trigonométricas de un triángulo rectángulo:

Las razones trigonométricas de un triángulo


rectángulo son las siguientes funciones:

La función seno, coseno, tangente, cosecante,


secante y cotangente.

Todas ellas pueden entenderse como


relaciones entre los lados de un triángulo
rectángulo.

Veamos las expresiones de cada una de ellas referidas a los ángulos α y β


del triángulo rectángulo aquí representado:

a) Para el ángulo α:

función seno función coseno función tangente


a b a
senα = cos α = tgα =
c c b
función cosecante función secante función cotangente
1 c 1 c 1 b
cos ec α = = s ecα = = cotgα = =
senα a cos α b tgα a

1/22
Ejercicios de trigonometría resueltos TIMONMATE

b) Para el ángulo β:

función seno función coseno función tangente


b a b
senβ = cos β = tgβ =
c c a
función cosecante función secante función cotangente
1 c 1 c 1 a
cos ecβ = = s ecβ = = cotgβ = =
senβ b cos β a tgβ b

A.2. Valores del seno, coseno y tangente para ciertos ángulos significativos
(en grados y radianes)

ángulo sen cos tg ángulo sen cos tg


π 3 1
0º 0 rad 0 1 0 60º rad 3
3 2 2
π 1 3 1 π
30º rad 90 rad 1 0 ∞
6 2 2 3 2
π 2 2
45º rad 1 180º π rad 0 –1 0
4 2 2

A.3. Significado geométrico de las razones trigonométricas en la esfera


goniométrica

Se llama circunferencia goniométrica a aquella que tiene por radio la


unidad. Para una circunferencia goniométrica es posible dar un sentido
muy intuitivo a todas las razones trigonométricas. Vamos a verlo
mediante el siguiente dibujo.

2/22
TIMONMATE Ejercicios de trigonometría resueltos

A.4. Relaciones entre las razones trigonométricas

a) Relaciones fundamentales:

El seno, el coseno y la tangente de un ángulo están relacionados


mediante la siguiente igualdad:

senθ
= tgθ
cos θ

Por otro lado, se cumple la siguiente igualdad, estrechamente


vinculada al teorema de Pitágoras:

sen 2 θ + cos 2 θ = 1

b) Relaciones del ángulo suma–diferencia:

sen ( α ± β ) = senα ⋅ cos β ± senβ ⋅ cos α

cos ( α ± β ) = cos α ⋅ cos β ∓ senα ⋅ senβ

tgα ± tgβ
tg ( α ± β ) =
1 ∓ tgα ⋅ tgβ

c) Relaciones del ángulo doble

Es un caso particular del anterior en el que α y β son iguales.

sen ( 2 α ) = 2senα ⋅ cos α

cos ( 2 α ) = cos 2 α − sen 2 α

2tgα
tg ( 2 α ) =
1 − tg 2 α

d) Relaciones del ángulo mitad

α 1 − cos α
sen 2 =
2 2

α 1 + cos α
cos 2 =
2 2

3/22
Ejercicios de trigonometría resueltos TIMONMATE

α 1 − cos α
tg 2 =
2 1 + cos α

A.5. Resolución de triángulos: Teoremas del seno y del coseno

Sea el siguiente triángulo. ¡No hace falta que sea rectángulo! Se


verifican las siguientes dos expresiones, conocidas como teorema del seno
y teorema del coseno.
A

c b

B C
a

a b c
a) Teorema del seno: = =
senA senB senC

b) Teorema del coseno: a 2 = b 2 + c 2 − 2bc cos A

B. EJERCICIOS RESUELTOS

B.1. Cálculo de razones trigonométricas

1. Sabiendo que senα = 0, 86 calcula las demás razones trigonométricas


directas e inversas

Solución:

Las razones trigonométricas directas son el seno, el coseno y la tangente, y


las inversas la cosecante, la secante y la cotangente. Vamos a relacionar
todas ellas con el seno, que es el dato que nos dan:

• senα = 0, 86

4/22
TIMONMATE Ejercicios de trigonometría resueltos

• El coseno se deduce a partir de la ecuación fundamental


sen 2 θ + cos 2 θ = 1 :

sen 2 θ + cos 2 θ = 1 ⇒ cos 2 θ = 1 − sen 2 θ ⇒ cos θ = 1 − sen 2 θ

Sustituyendo datos:
1
cos θ = 1 − sen 2 θ ⇒ cos θ = 1 − 0, 86 2 ⇒ cos θ =
2
• La tangente buscada se deduce de la fórmula fundamental
senθ
= tgθ . Sólo hay que sustituir en ella los valores conocidos:
cos θ

senθ 0, 86
= tgθ ⇒ tgθ = ⇒ tgθ = 1,72
cos θ 0, 5

• La cosecante es la inversa del seno.


1
cos ecα = sen−1α = = 1, 26
0, 86
• La secante es la inversa del coseno.
1
s ecα = cos−1 α = =2
1
2
• La cotangente es la inversa de la tangente.
1
cot gα = tg −1α = = 0, 58
1,72

2. Calcula las relaciones trigonométricas


directas de α y β
Solución:

Las razones trigonométricas directas son el


seno, el coseno y la tangente.

 Para el ángulo α :

40
senα = ⇒ senα = 0, 8 ,
50
30
cos α = ⇒ cos α = 0, 6
50
40
tgα = ⇒ tgα = 1, 33
30
Observa que se cumple que sen 2 α + cos 2 α = 1

5/22
Ejercicios de trigonometría resueltos TIMONMATE

 Para el ángulo β :
30
senβ = ⇒ senβ = 0, 6
50
40
cos β = ⇒ cos β = 0, 8
50
30
tgβ = ⇒ tgβ = 0, 75
40

Observa que también se cumple que sen 2β + cos 2 β = 1 , como no podía


ser de otra manera.

3. Halla las razones trigonométricas de los siguientes ángulos:

 135º
Solución:
El ángulo 135º está en el 2º cuadrante. Será equivalente a un ángulo
de 45º para el que sen45 es positivo y cos45 es negativo, tal como se
indica en la figura.

sen 45 135º
45º
- cos 45

 - 560º
Solución:
Como el ángulo es mayor que 360º lo tratamos del siguiente modo:

560 360 

 ⇒ 1 vuelta ⋅ 360º + 200º
200 1 

El ángulo que tenemos que manejar es -200º. Ello es equivalente a un


ángulo de 20º en el segundo cuadrante, en donde sen20 es positivo y
cos20 es negativo

6/22
TIMONMATE Ejercicios de trigonometría resueltos

sen 20 20º
- cos 45

-200º

3
4. Sabiendo que cos α = y que α está en el 4º cuadrante, halla las
2
demás razones trigonométricas.

Solución:

Si α está en el 4º cuadrante entonces cosα es positivo y senα es


negativo.

El senα lo deducimos usando la relación fundamental de la


trigonometría: sen 2α + cos 2 α = 1
2
 3  3 1
Así: sen α + cos α = 1 ⇒ sen α +   = 1 ⇒ senα = −
2 2 2
1 −  = −
 2  4 2

El resto de razones trigonométricas se obtiene de forma inmediata:


1

senα 1 1
tgα = = 2 =− ; cotgα = =− 3 ;
cos α 3 3 tgα
2
1 3 1
sec α = = ; co sec α = = −2
cos α 2 senα
1
5. Sabiendo que tgα = − y que α está en el 2º cuadrante, halla las
3
demás razones trigonométricas.

Solución:

Si α está en el 2º cuadrante entonces cosα es negativo y senα es


positivo.

1
- Utilizamos la relación tg 2α + 1 = para hallar senα :
sen 2α
2
1  1  1 4 1 3
2
tg α + 1 = 
⇒ −  + 1 = ⇒ = ⇒ senα =
2
sen α   3 2
sen α 3 sen α 2
2

7/22
Ejercicios de trigonometría resueltos TIMONMATE

senα
- Hallamos cosα a partir de tgα = :
cos α
3
senα 3
cos α = = 2 =− .
tgα 1 2

3

- Las obtención de las razones trigonométricas inversas es inmediata:


1 2 1 2 1
sec α = = − ; co sec α = = ; cot gα = =− 3
cos α 3 senα 3 tgα

1
6. Si α está en el tercer cuadrante y senα = − , determina las siguientes
2
razones trigonométricas:

 sen (180º −α )
Solución:
Como α está en el tercer cuadrante el senα es negativo, como bien
indica el enunciado. Pero, en general, senα = sen (180 −α ) , así que
1
sen (180 −α ) = −
2
 sen (180º +α )
Solución:
Como α está en el tercer cuadrante el senα es negativo. Además:
1
senα = −sen (180 −α ) , así que sen (180 −α ) =
2
 cos (180º −α )
Solución:
Como α está en el tercer cuadrante cosα es negativo. Además:
cos α = − cos (180 −α ) .

Deduzcamos cosα :

Usamos la relación fundamental de la trigonometría:


sen 2α + cos 2 α = 1
2
 1  1 3
sen 2α + cos 2 α = 1 ⇒ −  + cos 2 α = 1 ⇒ cos α = −1 −  = −
 2  4 4

8/22
TIMONMATE Ejercicios de trigonometría resueltos

3
Entonces, cos (180 −α ) =
4
 cos (180º +α )
Solución:
Se cumple que cos α = − cos (180 + α ) . Entonces:
3 3
− = − cos (180 +α ) ⇒ cos (180 + α ) =
4 4

 tg (180º −α )
Solución:
1
sen (180º −α ) − 2 2
tg (180º −α ) = = =
cos (180º −α ) − 3 3
4
 tg (180º +α )
Solución:
1
sen (180º +α ) 2 2
tg (180º +α ) = = =
cos (180º +α ) 3 3
4

B.2. Demostración de igualdades trigonométricas:

2sen α + 3
7. = cos α
2tg α + 3 sec α

Solución:
 Vamos a tratar de manipular el lado izquierdo de la igualdad, para
sen α
convertirlo en cos α . Teniendo en cuenta que tg α = y que
cos α
1
sec α = , podemos escribir:
cos α

2sen α + 3 2sen α + 3
=
2tg α + 3 sec α 2 sen α + 3
cos α cos α

9/22
Ejercicios de trigonometría resueltos TIMONMATE

 Operamos esa expresión con el fin de simplificarla:

2sen α + 3 2sen α + 3 cos α (2sen α + 3)


= = = cos α
sen α 3 2sen α + 3 2sen α + 3
2 +
cos α cos α cos α

 Como acabamos de ver, la igualdad se cumple.

sen 2α
8. tg 2α =
1 − sen 2α

Solución:
Vamos a manipular primeramente el miembro de la izquierda, que
llamaremos A:
sen 2α
A = tg 2α =
cos 2 α
Manipulamos el miembro de la derecha, que llamaremos B:
sen 2α
En B = vamos a reescribir el denominador de una forma
1 − sen 2α
más conveniente:
Teniendo en cuenta que sen 2α + cos 2 α = 1 se deduce que
1 − sen 2α = cos 2 α . Entonces:

sen 2α sen 2α
B= =
1 − sen 2α cos 2 α
Observamos que A=B, luego la identidad es verdadera.

2sen (α )  
9. tg (α )⋅ cot g (α ) − =  cos (α ) + sen (α ) ⋅  1 − 1
   sec (α ) cos ec (α )
1 + cot g 2 (α )  
Solución:
Vamos a manipular primeramente el miembro de la izquierda, que
llamaremos A:

10/22
TIMONMATE Ejercicios de trigonometría resueltos

2 ⋅ sen (α ) 1 2 ⋅ sen (α )
A = tg (α )⋅ cot g (α ) − = tg (α )⋅ − =
1 + cot g 2 (α ) tg (α ) 1
1+ 2
t g (α )
2 ⋅ sen (α ) 2 ⋅ sen (α ) 2 ⋅ sen (α )
= 1− = 1− = 1− =
2
cos (α ) 2
sen (α ) + cos (α ) 2
1
1+
sen 2 (α ) sen 2 (α ) sen 2 (α )

= 1 − 2 ⋅ sen 2 (α )

Manipulamos el miembro de la derecha, que llamaremos B:


 1 1 
B =  cos (α ) + sen (α ) ⋅  −  =
 sec (α ) cos ec (α )

=  cos (α ) + sen (α ) ⋅  cos (α ) − sen (α ) =

= cos 2 (α ) − sen 2 (α ) = 1 − sen 2 (α ) − sen 2 (α ) = 1 − 2 ⋅ sen 2 (α )


Observamos que A=B, luego la identidad es cierta.

1
10. 2
= sen 2α ⋅ cos 2 α + cos 4 α
sec α

Solución:
Manipulamos primeramente el miembro de la izquierda, que
llamaremos A:

1
A= = cos 2 α
sec2 α
Manipulamos el miembro de la derecha, que llamaremos B:

B = sen 2α ⋅ cos 2 α + cos 4 α = (sen 2α + cos 2 α )⋅ cos 2 α = cos 2 α

Observamos que A=B, luego la identidad es cierta.

11. cos ec 4α − 1 = 2 cot g 2α + cot g 4α

Solución:
Manipulamos el miembro de la izquierda, que llamaremos A:

11/22
Ejercicios de trigonometría resueltos TIMONMATE

A = cos ec 4α − 1 = (cos ec2α − 1)( cos ec 2α + 1)

Recordamos que cos ec2α = 1 + cot g 2α . Entonces:

(cos ec2α − 1)(cos ec2α + 1) = (1 + cot g 2α − 1)(1 + cot g 2α + 1) =


= cot g 2α ( cot g 2α + 2) = cot g 4α + 2 cot g 2α .

Hemos llegado a obtener el lado B de la expresión dada, luego se ha


demostrado que la igualdad es cierta.

2tg α
12. sen 2α =
1 + tg 2α

Solución:
Partiendo del miembro de la izquierda, que llamaremos A, mediante
manipulaciones adecuadas llegaremos al miembro de la derecha:
sen α ⋅ cos α
sen 2α = 2 ⋅ sen α ⋅ cos α = 2 ⋅ ⋅ cos α = 2 ⋅ tg α ⋅ cos 2 α =
cos α
1 1 tg α
= 2 ⋅ tg α ⋅ = 2 ⋅ tg α ⋅ 2 2
= 2⋅ =
1 sen α + cos α sen α cos 2 α
2
+
cos 2 α cos 2 α cos 2 α cos 2 α
2 ⋅ tg α
= . Queda así demostrado.
1 + tg 2α

2 ⋅ sen x + 3
13. = cos x
2 ⋅ tg x + 3 ⋅ sec x

Solución:
Partiendo del miembro de la izquierda, que llamaremos A, mediante

B.3. Ecuaciones trigonométricas

3
14. Resuelve: sen x =
2
Solución:

12/22
TIMONMATE Ejercicios de trigonometría resueltos

 π
 
 x = 60º =
3   1
3
x = sen−1   ⇒ 
 
2    2π
x 2 = 180º −60º = 120º =
 3
1
15. Resuelve: cos x =
2
Solución:

 π π
 x1 = 45º = 45º ⋅ =
1  180º 4
x = cos−1 ⇒
2 
x = 360º −45º = 315º = 315º ⋅ π = 7 π
 2 180º 4
1
16. tg x =
3
Solución:

 π π
 x = 30º = 30º ⋅ =
1  180º 6
x = tg −1 ⇒
3  π
x = 30º +180º = 210º =
 7

17. Resuelve la ecuación cos 2x = sen x en el intervalo [ 0, 2π ]


Solución:
• Hay que recordar que cos 2x = cos 2 x − sen 2 x . Así:

cos 2x = sen x ⇒ cos 2 x − sen 2 x = sen x

• Por otro lado, hay que tener en cuenta que cos 2 x + sen 2 x = 1 . Por
ello:

cos 2 x − sen 2 x = sen x ⇒ 1 − sen 2 x − sen 2 x = sen x ⇒


2
2
−1 ± (−1) − 4 ⋅ 2 ⋅ (−1)
⇒ 2 ⋅ sen x + senx − 1 = 0 ⇒ senx = =
2⋅2
sen x = −1
=  1
sen x =
 2
• Finalmente estudiamos cada uno de estos dos casos:

13/22
Ejercicios de trigonometría resueltos TIMONMATE


Si sen x = −1 , entonces: x 1 =
2
1 π 5π
Si sen x = , entonces: x 2 = y x 3 =
2 6 6

18. Resuelve la ecuación sen 2x ⋅ cos x = 6sen 3 x en el intervalo [ 0, 2π ]

Solución:
• Hay que recordar que sen 2x = 2sen x ⋅ cos x . Así:
sen 2x ⋅ cos x = 6sen 3 x ⇒ 2 ⋅ sen x ⋅ cos x ⋅ cos x = 6sen 3 x ⇒
⇒ 2 ⋅ sen x ⋅ cos 2 x = 6sen 3 x

• Por otro lado, hay que tener en cuenta que cos 2 x + sen 2 x = 1 . Por
ello:

2 ⋅ sen x ⋅ (1 − sen 2 x) = 6 ⋅ sen 3 x ⇒


⇒ sen x ⋅ (1 − sen 2 x) = 3 ⋅ sen x ⋅ sen 2 x ⇒
sen x = 0
⇒ sen x ⋅ ( 4 ⋅ sen x − 1) = 0 ⇒ 
2
1 1
sen 2 x = ⇒ sen x = ±
 4 2

• Finalmente estudiamos cada uno de estos tres casos:

Si sen x = 0 , entonces: x 1 = 0
1 π 5π
Si sen x = , entonces: x 2 = y x 3 =
2 6 6
1 7π 11π
Si sen x = − , entonces: x 4 = y x5 =
2 6 6

19. Resuelve: cos 2x − cos 6x = sen5x + sen3x


Solución:
 Vamos a utilizar las siguientes relaciones:
A+B A−B
cos A − cos B = −2 ⋅ sen ⋅ sen
2 2
A +B A −B
senA − senB = 2 ⋅ sen ⋅ cos
2 2
 Entonces:

14/22
TIMONMATE Ejercicios de trigonometría resueltos

2x + 6x 2x − 6x
cos 2x − cos 6x = −2 ⋅ sen ⋅ sen
2 2
5x + 3x 5x − 3x
sen5x − sen3x = 2 ⋅ sen ⋅ cos
2 2
 Sustituimos lo obtenido en la ecuación dada y pasamos todo a un
miembro

−2 ⋅ sen ( 4x)⋅ sen (−2x) = 2 ⋅ sen ( 4x)⋅ cos (x)

 Si tenemos en cuenta que sen (−a) = −sen (a) y sacamos factor común,
entonces:
2 ⋅ sen ( 4x) = 0
2 ⋅ sen ( 4x)⋅  sen (2x) − cos (x) = 0 ⇒ 
sen ( 2x) − cos (x) = 0

- Resolvemos la primera ecuación de las dos:

4x = 0 + 2kπ ⇒ x = k π
 2
2 ⋅ sen ( 4x) = 0 ⇒ 
 π π
4x = π + 2kπ ⇒ x = + k
 4 2

- Resolvemos la segunda ecuación:


sen ( 2x) − cos (x) = 0 ⇒ 2 ⋅ sen (x) cos (x) − cos ( x) = 0 ⇒

⇒  2 ⋅ sen (x) − 1 cos (x) = 0 ⇒

  π
 x = + 2kπ
  2
cos (x) = 0 ⇒ 
 x = π + 2kπ
3
  2
⇒ 
  π
  x = + 2kπ
2 ⋅ sen (x) − 1 = 0 ⇒ sen (x) = 1 ⇒  6
 2  5π
 x = + 2kπ
  2
La solución es entonces la unión de todas estas soluciones.

20. Resuelve el siguiente sistema en el intervalo [ 0, 2π ]

sen x + sen y = 1



2x + 2y = π 

Solución:

15/22
Ejercicios de trigonometría resueltos TIMONMATE

• Despejamos x en la segunda ecuación y llevaremos su valor a la


primera ecuación:

π
2x + 2y = π ⇒ x = − y , por lo que:
2
π 
sen x + sen y = 1 ⇒ sen  − y + sen y = 1
2 

• Ahora, para poder simplificar esta expresión usamos la fórmula del


seno de la diferencia de dos ángulos:

π  π π
sen  − y = sen ⋅ cos y − cos ⋅ seny = cos y , es decir:
2  2 2

π 
sen  − y + sen y = 1 ⇒ cos y + seny = 1
2 

• Intentamos expresar el coseno en función del seno, elevando al


cuadrado los dos miembros de la ecuación:

2
(cos y + seny) = 12 ⇒ cos 2 y + sen 2 y + 2 ⋅ seny cos y = 1 ⇒
⇒ 1 + 2 ⋅ sen y cos y = 1 ⇒ sen y cos y = 0

Pero sen y cos y = sen 2y , por lo que sen y cos y = 0 ⇒ sen 2y = 0

• Las soluciones para sen 2y = 0 están dadas por: 2y = 0 y 2y = π ,


π π
esto es: y 1 = 0 ; y 2 = . Teniendo en cuenta que x = − y ,
2 2
entonces:
π
y1 = 0 ⇒ x1 =
2
π
y2 = ⇒ x2 = 0
2

21. Calcula las soluciones del siguiente sistema, en el intervalo [ 0, 2π ] .

sen x = 2 ⋅ sen y



π 
x−y = 
3 

Solución:

16/22
TIMONMATE Ejercicios de trigonometría resueltos

• Despejamos x en la segunda ecuación y llevaremos su valor a la


primera ecuación:

π π
x−y = ⇒ x = + y , por lo que:
3 3
π 
sen x = 2 ⋅ sen y ⇒ sen  + y = 2 ⋅ sen y
3 

• Usamos la fórmula del seno de la suma de dos ángulos en el


miembro izquierdo de la ecuación:

π  π π 3 1
sen  + y = sen ⋅ cos y + cos ⋅ seny = cos y + seny
3  3 3 2 2
Entonces la fórmula a resolver es:

3 1 3 1
cos y + seny = 2seny ⇒ cos y = seny ⇒ 3 = tg y
2 2 2 2

 π
y 1 = 60º =
3
Solución: tg y = 3 ⇒  
 4π
y 2 = 180º +60º =
 3

22. Calcula las soluciones del siguiente sistema.


4y ⋅ sen x ⋅ cos x = 3

2y ⋅ cos 2x = 3 

Solución:

• Dividimos las dos ecuaciones del sistema:

4y ⋅ sen x ⋅ cos x = 3 4y ⋅ sen x ⋅ cos x 3 2 ⋅ sen x ⋅ cos x


 ⇒ = ⇒ = 3
2y ⋅ cos 2x = 3  2y ⋅ cos 2x 3 cos 2x

• Recordamos que 2 ⋅ sen x ⋅ cos x = sen 2x y sustituimos en la


ecuación:

2 ⋅ sen x ⋅ cos x 3 sen 2x


= ⇒ = 3 ⇒ tg 2x = 3
cos 2x 3 cos 2x

17/22
Ejercicios de trigonometría resueltos TIMONMATE

• Despejamos x:
π π
2x = + 2kπ ⇒ x = + kπ
3 6

B.4. Problemas

23. Calcula la altura de un árbol que a una distancia de 10 m se ve bajo un


ángulo de 30º.
Solución:
La altura, y, del árbol la deducimos de
la relación siguiente:
y
tg30 = ⇒ y = 10 ⋅ tg30 ⇒ y = 5,77 m
10

24. Calcula x e y:
Solución:
b
Aplicamos la relación tgθ = a los
a
y
dos triángulos rectángulos,
obteniendo el siguiente sistema de
ecuaciones:

30º 47º
 y
40 m tg47 =
x  x
 Operando:
 y
tg30 =
 40 + x

x ⋅ tg47 = y x ⋅ tg47 = y


 ⇒  ⇒ x ⋅ tg47 = ( 40 + x)⋅ tg30 ⇒
( 40 + x) tg30 = y ( 40 + x) tg30 = y

18/22
TIMONMATE Ejercicios de trigonometría resueltos

⇒ 1, 07x = 23, 09 + 0, 58x ⇒ 0, 49x = 23, 09 ⇒


23, 09
⇒x= ⇒ x = 47, 12 m .
0, 49
Calculemos finalmente el valor de y:
x ⋅ tg47 = y ⇒ 47, 12 ⋅ 1, 07 = y ⇒ y = 50, 42 m

25. Calcula x
Solución:

30º
100 m Tenemos dos triángulos.
60º
De cada uno de ellos
obtendremos una
ecuación trigonométrica.

x y

Resolvemos el sistema:
y
tg30 = 30º
100 100 m
57,7 = y 
 ⇒ 57, 7 = 173, 2 − x ⇒
y 173, 2 − x = y
⇒ x = 115, 5 m
x+y
tg60 =
100 60º
100 m

x+y

26. Calcula el valor de y (las longitudes están expresadas en m)

Solución:
12 Aplicamos el teorema del coseno:
y
a 2 = b 2 + c 2 − 2 ⋅ b ⋅ c ⋅ cos A
40º
Entonces:
10

19/22
Ejercicios de trigonometría resueltos TIMONMATE

y 2 = 10 2 + 12 2 − 2 ⋅ 10 ⋅ 12 ⋅ cos 40 ⇒
y = 100 + 124 − 240 ⋅ cos 40 = 6, 35 m

27. Calcula el valor de los lados x e y, aplicando el Teorema del seno:


a b c
= =
senA senB senC
Solución:
Sustituimos los valores dados en la
z= 3m expresión del teorema del seno:
y

a b c
40º 80º = = ⇒
senA senB senC
x
 3 ⋅ sen40
 y = = 1, 96 m
 sen80
⇒
 3 ⋅ sen60
x = = 2, 64 m
 sen80

28. Halla la altura de la montaña

B
45º

C
4000 m

30º
A

Solución:
Rehacemos el dibujo y de él extraeremos dos ecuaciones, cada una de ellas
 y el ACC´
perteneciente a un triángulo rectángulo (el CBB´ 

20/22
TIMONMATE Ejercicios de trigonometría resueltos

B
:
Triángulo CBB´
45º
4000 − h 4000 − h
tg45 =
x
C 45º
B´ 4000 m
:
Triángulo ACC´
h
h
tg30 =
x
C´ 30º
A
x

Resolvamos éste sistema:

4000 − h  4000 − h 
tg45 = 1=
x 
 x  x = 4000 − h 
⇒ ⇒  ⇒ 4000 − h = h 3 ⇒
h  1
=
h  x=h 3 
tg30 =
x  3 x 
4000
⇒h= m ≈ 1464 m
3 +1

29. Halla la altura de las Torres Petronas, x y también las distancias y, z.

z
x
y

75º 45º B
60º
D 678 m
A

Solución:

21/22
Ejercicios de trigonometría resueltos TIMONMATE

 . De él
 Primeramente vamos a centrarnos en el triángulo ABC
deduciremos las distancias y, z

C y z 678
= = ⇒
60º sen45 sen75 sen60
y z
 y 678
 =
75º 45º  sen45 sen60
⇒ ⇒
A B  z 678
 =
 sen75 sen60

 y 678
 = 
 2 3  2
  y = 678 m
 2 2  3
⇒ ⇒
 z 678 
 =  z = 1356 sen75 m
 sen75 3  3
 
 2
 . De él obtendremos la altura
 Ahora nos fijamos en el triángulo ACD
de las torres, x.
A

2 2 2
2 x= 678 ⋅ sen60 = 678 ⋅ = 452 m
600 m 3 3 3
x 3

60º
D C

22/22
SOLUCIONARI UNITAT 3. TRIGONOMETRIA

3. Trigonometria
EXERCICIS PROPOSATS
1. Exercici resolt.

2. Expressa en radians les mesures angulars següents.


a) 30º c) 200º
b) 60º d) 330º

30º   200º  10


a) 30º   rad c) 200º   rad
180º 6 180º 9

60º   330º  11


b) 60º   rad d) 330º   rad
180º 3 180º 6

3. Troba la mesura en graus dels angles següents.


7
a) rad c) 4 rad
3

3
b) rad d) 4 rad
2

7 7  180º 4  180º
a) rad   420º c) 4 rad   229º 11
3 3 

3 3  180º 4  180º
b) rad   270º d) 4 rad   720º
2 2 


4. L’angle d’un arc d’una circumferència de radi 9 cm fa rad. Quina és la longitud d’aquest arc?
6

2  9 
  4,17 cm
2 6
3
5. Un arc d’una circumferència fa 15 cm i comprèn un angle de rad. Quant fa el radi de la circumferència?
5

3
r  15  r  7, 96 cm
5

6 i 7. Exercicis resolts.

8. El sinus d'un angle agut d'un triangle rectangle fa 0,35 i la hipotenusa fa 10 cm. Calcula la mida dels catets.
b b
sin Bˆ   0,35   b  3,5 cm
a 10

c  102  3,52  87,75  9,37

90 Matemàtiques 1r Batxillerat
SOLUCIONARI UNITAT 3. TRIGONOMETRIA

9. El cosinus d'un angle agut d'un triangle rectangle fa 0,8 i el catet oposat fa 6 cm. Calcula quant fan els al-
tres costats.

cos Bˆ  0,8  sin2 Bˆ  1  cos2 Bˆ  1  0,82  0,36  sin Bˆ  0,6


b 6
sin Bˆ   0,6   a  10 cm
a a
a2  b2  c 2  102  62  c 2  c  8 cm

Hipotenusa = 10 cm, i catet que falta = 8 cm.

10. Determina les raons trigonomètriques dels angles aguts d'un triangle rectangle els costats del qual fan 8,
15 i 17 cm.
8 15
sin    0,4706 cos   0,8824
17 17
15 8
sin    0,8824 cos    0,4706
17 17

11. Els catets d'un triangle rectangle fan 12 i 35 cm. Calcula el sinus i el cosinus dels angles aguts d'aquest
triangle.

a2  b2  c 2  a2  122  352  1369  a  37 cm

12 35
sin    0,3243 cos   0,9459
37 37
35 12
sin    0,9459 cos    0,3243
37 37

12 i 13. Exercicis resolts.

14. Calcula la cosecant, la secant i la cotangent d'aquests angles.


a) 30º b) 45º c) 60º
30º 45º 60º
cosecant 2 1,414 1,155
secant 1,155 1,414 2
cotangent 1,732 1 0,577

15. L'altura d'un triangle equilàter fa 9 cm. Quant fan els costats?
2
c 3c 2 3c 3c 29
h2  c 2     h 9 c   10,39 cm
2 4 2 2 3

91 Matemàtiques 1r Batxillerat
SOLUCIONARI UNITAT 3. TRIGONOMETRIA

16. La hipotenusa d'un triangle rectangle isòsceles fa 15 cm. Quant fan els catets?
a 15
a2  b2  b2  a2  2b2  b    10,61 cm
2 2

17. Calcula els costats que falten d'aquests triangles rectangles.

b
a) sin30   b  sin30  6  0,5  6  3 cm
6

a2  b2  c 2  62  32  c 2  c  36  9  27  5,20 cm

14 14 14
b) sin 60  a   16,17 cm
a sin 60 sin 60

2 2
 14   14 
a2  b2  c 2     14  c  c   sin 60   14  8,08 cm
2 2 2

 sin 60   

18 i 19. Exercicis resolts.

20. El sinus de l'angle agut  fa 0,6. Determina el cosinus i la tangent d'aquest angle.

sin2  cos2   1  cos   1 sin2   1 0,36  0,8

sin  0,6
tg    0,75
cos  0,8

21. La tangent de l'angle agut  fa 0,4. Determina el sinus i el cosinus d'aquest angle.
cos  = 0,9284 i sin 0,3713

1 1 1
1  tg2   cos     0,9285
cos2  1  tg2 1  0,42

1
sin2  cos2   1  sin   1  cos2   1   0,3714
1  0,42

22. La tangent d'un angle agut  fa 0,9. Calcula quant fa el cosinus de .

1 1 1
1  tg2   cos     0,7433
cos 
2
1  tg 
2
1  0,92

23. Demostra aquestes igualtats:

a) cos2   sin2   1 2sin2 

tg2   2 sin2 
b)  1  2cos2 
tg2 

1 1 1
c)  
cos2  sin2  sin2   cos2 

92 Matemàtiques 1r Batxillerat
SOLUCIONARI UNITAT 3. TRIGONOMETRIA

a)  
cos2   sin2   1  2sin2   1 sin2   sin2   1 2sin2 

tg2   2 sin2  tg2  2 sin2  2 sin2 


 1  2 cos 2
    1  
tg2  tg2  tg2  sin2 
b) cos2 
2 sin2  cos2 
 1  1  2 cos2 
sin2 

1 1 1 sin2   cos2  1
c)    
cos  sin  sin   cos 
2 2 2 2
cos2   sin2  sin2   cos2 

24. Exercici resolt.


25. Troba els angles reduïts i les raons trigonomètriques:

 
a) 3990º b) 9c) 25 200º d) e) 765º f)


3990º  25 200º 765º


angle reduït 30º 0 45º

sin 0,5 0 0 0,7071 0,7071


cos 0,8660 –1 1 0,7071 0,7071
tg 0,5773 0 0 1 1

26. Per als angles següents, indica el signe de totes les seves raons trigonomètriques.
3 4 9
a) c) e) 
4 3 4
11 7
b) d) 
3 6

3 11 4 7 9
  
4 3 3 6 4
Quadrant II IV III II IV
sin  i cosec      
cos i sec      
tg i cotg     

27 i 28. Exercicis resolts.

29. Calcula el valor de les raons trigonomètriques següents reduint-les al primer quadrant.
a) sin 150º d) tg 330º g) sin 240º
b) cos 225º e) cosec 135º h) cotg 300º
c) sin 840º f) tg 1800º i) sec 2295º

1 3 3
a) sin 150º  sen 30º  d) tg 330º  tg 30º   g) sin 240º   sin 60º  
2 3 2

93 Matemàtiques 1r Batxillerat
SOLUCIONARI UNITAT 3. TRIGONOMETRIA

2 1 1 3
b) cos 225º  cos 45º   e) cosec 135º   2 h) cotg 300º    
2 sen 45º tg 60º 3

3 1
c) sin 840º  sin 60º  f) tg 1800º  tg 0º  0 i) sec 2295º     2
2 cos 45º

30. Calcula, en funció de h, sin 303º, sabent que cos 33º  h.

sin 303º  sin 57º  sin (90º  33º)  cos 33º  h

31. Calcula el valor exacte de:


3 11 4 5
a) sin b) sin c) sin d) sin
4 6 3 6

3  2 4  3
a) sin  sin  c) sin   sin  
4 4 2 3 3 2

11  1 5  1
b) sin   sin   d) sin  sin 
6 6 2 6 6 2

32. Exercici interactiu.

33 i 34. Exercicis resolts.

5
35. Transforma 15º i rad en una suma o diferència d’angles i calcula’n les raons trigonomètriques.
12

3 2 1 2 6 2
15º  60  45 : sin15º  sin  60º 45º   sin60ºcos45º  cos 60º sin 45º     
2 2 2 2 4

1 2 3 2 6 2
cos15º  cos  60º 45º   cos60ºcos45º  sin 60º sin 45º     
2 2 2 2 4

tg60º  tg 45º 3 1 2 3 4
tg15º  tg  60º 45º      2 3
1  tg60º tg 45º 1  3 2

5   5       2 3 2 1 6 2
  : sin  sin     sin cos  cos sin     
12 4 6 12 4 6 4 6 4 6 2 2 2 2 4

5       2 3 2 1 6 2
cos  cos     cos cos  sin sin     
12 4 6 4 6 4 6 2 2 2 2 4

  3
tg  tg 1
5   4 6 3  2 3
tg  tg     
12  4 6  1  tg  tg  3
1
4 6 3

94 Matemàtiques 1r Batxillerat
SOLUCIONARI UNITAT 3. TRIGONOMETRIA

36. Calcula tg 75º a partir del sinus i del cosinus.

2 3 2 1 6 2
75º  45º 30º : sin75º  sin  45º 30º   sin45ºcos30º+cos 45º sin30º     
2 2 2 2 4

2 3 2 1 6 2
cos75º  cos  45º 30º   cos45ºcos30º- sin 45º sin30º     
2 2 2 2 4

6 2
sin75º 4
tg75º    2 3
cos 75º 6 2
4

 3 
37. Demostra que sin       cos  .
 2 

 3  3 3
sin      sin cos  cos  sin  sin  0  cos    1   cos 
 2  2 2

38. Exercici resolt.

4
39. Determina el valor del sinus, el cosinus i la tangent dels angles de 120º i rad.
3

3 1 3
sin120º  sin(2·60º )  2sin 60º cos 60º  2· · 
2 2 2
2
 1  3 
2
1
cos120º  cos(2·60º )  cos2 60º sin2 60º       
 2   2  2

2tg60º 2 3
tg120º  tg(2·60º )    3
1  tg 60º 1  3
 
2 2

4  2  2 2 3  1 3
sin  sin  2·   2sin cos  2· ·    
3  3  3 3 2  2 2
2
2  1   3 
2
4  2  2 1
cos  cos  2·   cos2  sin2      
3  3  3 3  2   2  2

2
2tg
4  2  3  2 3
tg  tg  2·    3
 3  1  tg2 2 1   3
 
2
3
3

40 El sinus de l’angle de 20° fa 0,34. Calcula el sinus i el cosinus dels angles de 10° i de 40°.

sin2  cos2   1  cos 20  1  sin2 20  1  0,342  0,9404

20 1  cos 20 1  0,9404


sin  sin 10    0,1726
2 2 2

sin2  cos2   1  cos10  1 sin2 10  1 0,17262  0,9850

sin 2  20  sin 40  2 sin 20 cos 20  2  0,34  0,9404  0,6395

sin2  cos2   1  cos 40  1 sin2 40  1 0,63952  0,7688

95 Matemàtiques 1r Batxillerat
SOLUCIONARI UNITAT 3. TRIGONOMETRIA

3 
41. Si  és un angle del segon quadrant i sin   , calcula les raons de .
5 2

L’angle és del primer quadrant, per tant, les seves raons trigonomètriques són positives. Tenim:
2

4
1
3 9 4  1  cos  5  9 3 10
sin    cos    1  sin2   1   sin   
5 25 5 2 2 2 10 10

4 4
1 1
 1  cos  5  1 10  1  cos  5
cos    tg    9 3
2 2 2 10 10 2 1  cos  1
4
5
42. Calcula sin 32º suposant que sin 8º = 0,14.

sin8º  0,14  cos8º  1 sin2 8º  1 0,142  0,99

sin16º  sin(2·8º )  2sin8º cos8º  2·0,14·0,99  0,2772  cos16º  1 sin216º  0,9608

sin32º  sin(2·16º )  2sin16º cos16º  2  0,2772  0,9608 0,5327

43. Exercici resolt.

44. Resol les equacions següents i dona els resultats en graus i en radians.

a) sin x  1 b) 2cos x  1  0 c) 3tg x  1  0

a) El sinus d’un angle val 1 a 90º, 450º, 810º, etc.



Per tant x  90º  360ºk amb k  o, en radians, x   2k amb k  .
2

 2
1  x  120º 360º k  3  2k
b) 2cos x  1  0  cos x     k  .
2  x  240º 360º k  4  2k
 3

1 3 
c) 3tg x  1  0  tg x    x  30º 180º k   k k  .
3 3 6

45. Resol les equacions trigonomètriques indicant les solucions compreses en l’interval [0, 2].
a) sin x  cos x  0 b) sin2x  sin x  0

 3
 x  135º  rad
 4
a) sin x  cos x  0  sin x   cos x  tg x  1  
 x  315º  7 rad

 4

  x  0º  0 rad
sin x  0   x  180º   rad
 
  x  360º  2 rad
 
b) sin 2 x  sin x  0  2 sin x cos x  sin x  0  sin x  2 cos x  1  0    

1  x  60º  3 rad
cos x   
 2  x  300º  5 rad
 
 3

96 Matemàtiques 1r Batxillerat
SOLUCIONARI UNITAT 3. TRIGONOMETRIA

46. Calcula la longitud dels costats b i c d’un triangle ABC sabent que a  10 cm, Â  45º i B̂  100º.

Apliquem el teorema del sinus:

a c a sin Cˆ 10 sin35º
 c    8,11 cm
sin Aˆ sin Cˆ sin Aˆ sin 45º

47. Donat un triangle ABC amb a  12 cm, b  15 cm i Ĉ  35º, calcula la longitud del costat c.

Aplicant el teorema del cosinus:

c 2  a2  b2  2ab cos Cˆ  122  152  2  12  15cos35º  74,105  c  8,61 cm.

48. Determina els elements desconeguts d’aquests triangles.

a) L'angle  val 180º – 50º –35º = 95º


10 b 10  sin35º
 b  5,76 cm
sin95º sin35º sin95º
10 c 10  sin50º
 c   7,69 cm
sin95º sin50º sin95º
7 9 9  sin 25º
b)   sin Aˆ   0,543  Aˆ  147º 05 '12''
sin 25º sin Aˆ 7

Bˆ  180º 147º 05'12''  7º 54' 47''

9 b 9  sin Bˆ
 b  2,28 cm
sin Aˆ sin Bˆ sin Aˆ

c) a2  b2  c 2  2bc cos Aˆ  122  82  102  2  8  10cos Aˆ  cos Aˆ  0,125  Aˆ 82º 49'09''

b2  a2  c 2  2ac cos Bˆ  82  122  102  2  12  10cos Aˆ  cos Aˆ  0,75  Aˆ 41º 24'34''

c 2  a2  b2  2ab cos Cˆ  102  122  82  2  12  8cos Aˆ  cos Aˆ  0,5625  Aˆ 55º 46'16''

97 Matemàtiques 1r Batxillerat
SOLUCIONARI UNITAT 3. TRIGONOMETRIA

d) a2  b2  c 2  2bc cos Aˆ  a2  92  72  2  9  7cos60º  a2  67  a 8,19 cm

8,19 9 9  sin 60º


  sin Bˆ  0,952  Bˆ  72º 06 ' 54 ''
sin 60º sin Bˆ 8,19

Cˆ  180º Aˆ  Bˆ  47º 53'06''

a b a sin Bˆ 8 sin 40º


 b   5,22
sin Aˆ sin Bˆ sin Aˆ sin80º

a b a sin Bˆ 8 sin 40º


d)  b   5,22
sin Aˆ sin Bˆ sin Aˆ sin80º

49. Resol aquests triangles.

a) Â  80º, B̂  40º, a  8 dm c) a  10 cm, b  15 cm, c  20 cm

b) Â  80º, a  10 m, b  5 m d) Â  75º, b  8 mm, c  12 mm

a) Cˆ  180º Aˆ  Bˆ  60º


Apliquem el teorema del sinus dues vegades:

a b a sin Bˆ 8 sin 40º


 b   5,22 dm
sin Aˆ sin Bˆ sin Aˆ sin80º

a c a sin Cˆ 8 sin 60º


 c    7,04 dm
sin Aˆ sin Cˆ sin Aˆ sin80º
b) Apliquem el teorema del sinus:

a b b sin Aˆ 5 sin80º
  sin Bˆ   0,492  Bˆ  29º 29 55,34
sin Aˆ sen Bˆ a 10

(La possibilitat Bˆ  150º 31 40 no és vàlida)

Cˆ  180º Aˆ  Bˆ  70º 30 4,66


Apliquem el teorema del cosinus:

c 2  a2  b2  2ab cosCˆ  102  52  2  10  5cos70º 30 4,66  91,621  c 9,57 m


c) Apliquem el teorema del cosinus dues vegades:

b2  c 2  a2 152  202  102


cos Aˆ    0,875  Aˆ  28º 57 18
2bc 2·15·20

a2  c 2  b2 102  202  152


cos Bˆ    0,6875  Bˆ  46º 34 3
2ac 2·10·20

Cˆ  180º Aˆ  Bˆ  104º 28 39


d) Apliquem el teorema del cosinus dues vegades:

a2  b2  c 2  2bc cos Aˆ  82  122  2  8  12cos75º  158,307  a  12,58 mm

a2  c 2  b2 12,582  122  82
cos Bˆ    0,789  Bˆ  37º 53 42
2ac 2·12,58·12

Cˆ  180º Aˆ  Bˆ  67º 6 18

98 Matemàtiques 1r Batxillerat
SOLUCIONARI UNITAT 3. TRIGONOMETRIA

50. En un triangle, l’angle més gran fa 10° més que l’angle mitjà i 20° més que l’angle més petit. El costat més
llarg del triangle fa 7 cm. Quant fan els altres costats?

L'angle gran fa  . L'angle mitjà fa Bˆ  Aˆ  10º i l'angle petit fa Cˆ  Aˆ  20º .

180º  Aˆ  Aˆ  10º  Aˆ  20º  3Aˆ  30º  Aˆ  70º, Bˆ  60º, Cˆ  50º


Apliquem el teorema del sinus:
a b c 7 b c 7 b c
         b  6,45 cm, c  5,71 cm
sin Aˆ s i n Bˆ s i n Cˆ sin70º sin60º sin50º 0,9397 0,8660 0,7660

51. Exercici interactiu.

52 a 64. Exercicis resolts.

EXERCICIS
Mesura d’angles
65. Expressa aquestes mesures en radians.

Graus 30º 45º 60º 90º 120º 135º 150º 180º


    2 3 5
Radians 
6 4 3 2 3 4 6

Graus 210º 225º 240º 270º 300º 315º 330º 360º


7 5 4 3 5 7 11
Radians 2
6 4 3 2 3 4 6

66. Passa de graus a radians.


a) 585º b) 450º c) 76º 52 30 d) 382º 30

585 13 76,875 41


a) 585   rad c) 76º 52' 30 ''   rad
180 4 180 96

450 5 382,5 17


b) 450   rad d) 382º 30 '   rad
180 2 180 8

67. Els angles següents estan en radians. Passa’ls a graus.


41 11
a) rad b) 13 rad c) rad d) 5 rad
3 12

41 41 ·180 11 11 ·180


a) rad   2460º c) rad   165º
3 3 12 12

13 ·180 5·180


b) 13 rad   2340º d) 5 rad   286º 28 44
 

99 Matemàtiques 1r Batxillerat
SOLUCIONARI UNITAT 3. TRIGONOMETRIA

Raons trigonomètriques
68. Calcula les raons trigonomètriques dels angles aguts dels triangles rectangles següents.

a) Â  90º, a  29 cm, b  20 cm b) B̂  90º, a  65 cm, c  72 cm

a) c  292  202  441  21 cm

b 20 c 21 b 20
sin Bˆ   cos Bˆ   tg Bˆ  
a 29 a 29 c 21

c 21 b 20 c 21
sin Cˆ   cos Cˆ   tgCˆ  
a 29 a 29 b 20

b) b  652  722  9409  97 cm

a 65 c 72 a 65
sin Aˆ   cos Aˆ   tg Aˆ  
b 97 b 97 c 72

c 72 a 65 c 72
sin Cˆ   cos Cˆ   tgCˆ  
b 97 b 97 a 65

69. Indica els angles següents com la suma d’un nombre enter de voltes completes més l’angle restant.
46 52
a) 2345º b) 1500º c) rad d)  rad
3 7

46 4 4
a) 2345º  6 · 360º  185º  6 voltes  185º c) rad  7·2   7 voltes  rad
3 3 3
52 4 4
b) 1500º  5 · 360º  300º  5 voltes  300º d)  rad  4·2   4 voltes  rad
7 7 7

70. Utilitza la calculadora per trobar el valor de les raons trigonomètriques següents. Aproxima els resultats a
les mil·lèsimes.
a) sin 36º c) cotg 111º e) sec 126º 33
b) tg 331º d) sin 25º 40 f) cotg 121º 22 45

a) sin 36º  0,588 c) cotg 111º  0,384 e) sec 126º 33  1,679
b) tg 331º  0,554 d) sin 25º 40  0,433 f) cotg 121º 22 45  0,610

71. Utilitza la calculadora per trobar el valor de les raons trigonomètriques següents. Aproxima els resultats a
les mil·lèsimes. Tingues en compte que els angles estan donats en radians.
 3 21
a) sin c) cos e) tg
12 7 5
b) cosec 2 d) sec 3 f) cotg 2,75

 3 21
a) sin  0,259 c) cos  0,223 e) tg  0,727
12 7 5
b) cosec 2  1,100 d) sec 3  1,010 f) cotg 2,75  2,422

100 Matemàtiques 1r Batxillerat


SOLUCIONARI UNITAT 3. TRIGONOMETRIA

72. Calcula totes les raons trigonomètriques de l’angle α sabent que:


2
a) És un angle del primer quadrant i cos  
3
b) Pertany al segon quadrant i sen   0,25

c) 180º    270º i tg   2

3
d)    2 i sec   2
2
e) 90º    180º i cotg   3

3 5
f)  i cosec   
2 2

a) Com que és un angle del primer quadrant, totes les raons són positives. Tenim:
2 3
cos    sec  
3 2

4 5 3 3 5
sin2  cos2   1  sin   1 cos2   1   cosec   
9 3 5 5

sin  5 2 2 5
tg   =  cotg   
cos  2 5 5
b) Com que és un angle del segon quadrant, el sinus i la cosecant són positius i la resta de raons són negatives.
Tenim:
1
sin   0,25   cosec   4
4

1 15 4 4 15
sin2  cos2   1  cos    1  sin2    1    sec    
16 4 15 15

sin  1 15
tg      cotg    15
cos  15 15

c) Com que és un angle del tercer quadrant, la tangent i la cotangent són positius, i la resta de raons, negatives.
Tenim:

1 2
tg   2  cotg   
2 2

1 3
1  tg2  sec 2   sec    1  tg2   1 2   3  cos    
3 3

sin  6 3 6
tg    sin   cos  tg     cosec    
cos  3 6 2

d) Com que és un angle del quart quadrant, el cosinus i la secant són positius, i la resta de raons, negatives.
Tenim:

1 2
sec   2  cos   
2 2

1 1 2
sei  cos2   1  sin    1  cos2    1     cosec    2
2 2 2

sin 
tg    1  cotg   1
cos 

101 Matemàtiques 1r Batxillerat


SOLUCIONARI UNITAT 3. TRIGONOMETRIA

negatives
e) Com que és un angle del segon quadrant, el sinus i la cosecant són positius i la resta de raons són negatives.
Tenim:
1
cotg   3  tg   
3

1 10 3 3 10
1  tg2  sec 2   sec    1  tg2   1   cos    
9 3 10 10

sin  10 10
tg    sin   cos  tg    cosec    10
cos  10 10

f) Com que és un angle del tercer quadrant, la tangent i la cotangent són positius, i la resta de raons, negatives.
Tenim:
5 2
cosec     sin   
2 5

4 21 5 5 21
sin2  cos2   1  cos    1  sin2    1    sec    
25 5 21 21

sin  2 2 21 21
tg      cotg  
cos  21 21 2

73. Calcula, de forma exacta, el valor de les raons trigonomètriques següents.


7
a) sin 240º d) sin 1215º g) tg j) cotg 225º
3
5 7
b) cos 135º e) cosec 330º h) sec k) sin
3 4
c) cos(600º) f) tg 300º i) sec 120º l) tg( 15 )

3 7 
a) sin 240º  sin 60º   g) tg  tg  3
2 3 3

2 5 
b) cos135º   cos 45º   h) sec  sec  2
2 3 3

1
c) cos  600º   cos 600º   cos 60º   i) sec120º   sec 60º  2
2

2
d) sin1215º  sin135º  sin 45º  j) cotg 225º  cotg45º  1
2

7  2
e) cosec 330º  cosec 30º  2 k) sin   sin  
4 4 2

f) tg300º  tg 60º   3 l) tg  15   tg15   tg   0

102 Matemàtiques 1r Batxillerat


SOLUCIONARI UNITAT 3. TRIGONOMETRIA

74. Calcula, en funció de h, el valor de cadascuna de les raons trigonomètriques següents.


a) sin 123º, sent sin 57º  h. d) cos 250º, sent sin 110º  h. g) tg 290º, sent sin 110º  h.
b) cos 220º, sent tg 40º  h. e) cos 247º, sent sin 113º  h. h) sin 83º, sent cos 7º  h.
c) tg 260º, sent sin 80º  h. f) cosec 701º, sent cotg 199º  h. i) sec 203º, sent cotg 67º  h.

a) sin123º  sen57º  h sen123º  sen57º  h

1 1 1 1
b) 1  tg   sec    cos 220º    
2 2
cos2  1  tg2 220º 1  tg2 40º 1  h2

1 1 1 1 1
c) 1  tg2  sec 2     tg260º  1  1  1 
cos2  1  sin2  1  sin2 260º 1    sin80º  1  h2
2

h2 h
 
1 h 2
1  h2

d) cos250º  cos110º   1  sin2 110º   1  h2

e) cos 247º   1 sin2 247º   1   sin113º    1 h2


2

f) cosec 701º  cosec 341º  cosec19º  cosec199º   1  cotg2199º   1 h2

1 1 h2 h
g) tg290º  tg110º  1  1  
1  sin 110º
2
1 h 2
1  h2 1  h2

h) sin83º  cos7º  h sen83º  cos7º  h

1 1
i) sec 203º   sec 23º     cosec 67º   1  cotg2 67º   1  h2
cos 23º sin67º

75. Determina la raó trigonomètrica que s’indica en cada cas, expressant-la en funció de h.
23 3
a) cosec , sabent que cotg  h 2 . c) tg 348º, sabent que cos 192º  h2 .
5 5
1
b) sec 305º , sabent que cotg55º  .
h

23 3 3
a) cosec  cosec  1  cotg2  1  h4
5 5 5
1 1 1
b) sec 305º     1  tg2 55º  1  h2
cos 305º cos 55º 1
1  tg2 55º

1 1 1 1 1  h4
c) tg348º   1   1   1   1  
2
cos 348º cos2 12º   cos192º 
2
h 4
h2

 3
76. Si  i  i, sent sin   0, 4 i cos   0,5 , calcula:
2 2
a) sin(  ) b) cos(  ) c) tg(  )

cos    1 sin2   0,917 i sin    1  cos2   0,866

103 Matemàtiques 1r Batxillerat


SOLUCIONARI UNITAT 3. TRIGONOMETRIA

a) sen      sen  cos   cos  sen   0,4  0,5  0,917  0,866  0,994

b) sin      sin  cos   cos  sin   0,4  0,5  0,917  0,866  0,994

0,4 0,866
 
tg   tg  0,917 0,5
c) tg(  )    0,738
1  tg  tg  1  0,4  0,866
0,917 0,5

77. Sabent que tg α = 3, calcula les raons trigonomètriques de l’angle 2α si α és un angle:


a) Del primer quadrant b) Del tercer quadrant

a) Com que tg  >1, 45º <  < 90º, per tant, 2 pertany al segon quadrant. Tenim:

1 1 1 10 3 10
cos      sin   tg  cos  
1  tg 
2
1 9 10 10 10

3 10 10 6 3 3 10 10 6 3
sin 2  2 sin  cos   2      0,6 sen 2  2 sen  cos   2      0,6
10 10 10 5 10 10 10 5

1 9 8 4 1 9 8 4
cos 2  cos2   sin2        0,8 cos 2  cos2   sen2        0,8
10 10 10 5 10 10 10 5
sin 2 3
tg2     0,75
cos 2 4
b) Com que tg  >1, 225º <  < 270º, per tant, 2 pertany al segon quadrant i s’obtenen els mateixos valors de
l’apartat interior per a les raons 2.

78. Calcula, de forma exacta, les raons trigonomètriques dels angles següents.
a) 15º b) 7º 30

3
1
 30º  1  cos 30º 2  2 3 2 3
a) sin15º  sin    
 2  2 2 4 2

3
1
 30º  1  cos 30º 2  2 3 2 3
cos15º  cos    
 2  2 2 4 2

2  3   2 
2
sin15º 2 3
tg15º    3
cos15º 2 3  2  3 2  3 
2 3
1
 15º  1  cos15º 2 2 2 3
b) sin7º 30 '  sin    
 2  2 2 2

2 3
1
 15º  1  cos15º 2 2 2 3
cos 7º 30 '  cos    
 2  2 2 2

2
2  2  3 
2 2 3   2 2 3
tg7º 30 ' 
sin7º 15 '
    
cos 7º 15 ' 2 2 3 2  2  3 2  2  3  2 3
 
  

104 Matemàtiques 1r Batxillerat


SOLUCIONARI UNITAT 3. TRIGONOMETRIA

2 
79. Si cos    i 90º <  < 180º, calcula les raons trigonomètriques de .
3 2

2 2
1 1
 1  cos  3  5 30  1  cos  3  1  6
sin    cos  
2 2 2 6 6 2 2 2 6 6

30
 6  5
tg 
2 6
6

Identitats trigonomètriques
80. Demostra les identitats trigonomètriques següents.
sin   cos  1  cotg
a)  cos  b) tg2   sin2   tg2  sin2  c)  cosec  d) sec 2   1  tg2 
tg   1 sin   cos 

sin   cos  sin   cos  sin   cos   sin   cos   cos 


a)     cos 
tg   1 sin  sin   cos  sin   cos 
1
cos  cos 

b) tg2   sin2  
sin2 
 sen2
  

sin2   sin2  cos2  sin  1  cos 
2 2



sin2 
sin2   tg2  sin2 
cos2  cos2  cos2  cos2 

cos  sin   cos 


1
1  cotg  sin   sin  1
c)    cosec 
sin   cos  sin   cos  sin   cos  sin 

1 1  cos2  sin2 
d) sec 2   1   1    tg2 
cos2  cos2  cos2 

81. Demostra les identitats trigonomètriques següents.


tg 
a)  tg2  tg  e) sin2   sin2   sin(  )sin(  )
cos 2

b) tg   cotg   sec  cosec  f)  cos   cos 2   sin   sin 2  4 sin2
2

   
tg      tg    
sin 2 4  4   tg2
c)  tg  g)
1  cos 2 2

1  cos 2 sin2
2
1  cos  
d)  cos2     h)   sin   tg 
tg2   tg   2sin  1  cos 2

sin2  cos2   sin2 


1
2 tg   2  1  tg2  cos   tg 
2
cos2  tg 
a) tg2  tg    tg   tg    1  tg   tg  
1  tg 
2
 1  tg  
2
1  tg 
2
sin 
2
cos   sin  cos 2
2 2
1
cos2  cos2 

sin  cos  sin2   cos2  1


b) tg   cotg       sec  cosec 
cos  sen  sin  cos  sin  cos 

sin2 2sin  cos  2sin  cos  sin 


c)     tg 
1  cos 2 1  cos2   sin2  2cos2  cos 
sen2 2sen  cos  2sen  cos  sen 
    tg 
1  cos 2 1  cos2   sen2  2cos2  cos 

105 Matemàtiques 1r Batxillerat


SOLUCIONARI UNITAT 3. TRIGONOMETRIA

2
 cos   cos2   sin2  cos2  sin2  cos2  sin2   cos2  1
d) cos2        2    2
 tg   sin2  tg2  tg  tg2  tg2  tg 

e) sin(  )sin(  )   sin  cos   cos  sin  sin  cos   cos  sin   sin2  cos2   cos2  sin2  

sin2  cos2   (1 sin2 )sin2   sin2 (cos2   sin2 )  sin2   sin2   sin2 

f) cos   cos 2   sin   sin 2  cos2   cos2   2cos  cos   sin2   sin2   2sin  sin  
    2  2 
 2  2  cos2  sin2   2  2 1  2 sin   4 sin
 2 2   2  2
g) L’expressió és equivalent a la demostrada a l’apartat d.

1  cos 2 sin2 1  cos2   sin2  2sin  cos  2sin2  2sin  cos 


h)       sin   tg 
2sin  1  cos 2 2sin  1  cos   sin 
2 2
2sin  2cos2 

Equacions trigonomètriques
82. Resol les equacions trigonomètriques següents indicant totes les seves solucions en graus.

1 2 3
a) sin x  d) sin x   g) tg x  
2 2 3

3 1
b) cos x  e) cos x   h) sin x  0 sen x  0
2 2
c) tg x  1 f) 1 cos x  0 i) tg x  0

1  x  30º 360º k
a) sin x   k  f) 1 cos x  0  cos x  1  x  180º 360º k k 
2  x  150º 360º k

3  x  30º 360º k 3  x  150º 360º k


b) cos x   k  g) tg x    k   150º 180º k k 
2  x  330º 360º k 3  x  330º 360º k

 x  45º 360º k
c) tg x  1   k   x  45º 180º k k 
 x  225º 360º k

2  x  225º 360º k
d) sin x    k  h) sin x  0  x  180º k k 
2  x  315º 360º k

1  x  120º 360º k
e) cos x    k  i) tg x  0  x  180º k k 
2  x  240º 360º k

83. Resol les equacions trigonomètriques següents indicant totes les seves solucions en radians.

3 2
a) sin 4 x   b) cos 2 x  c) tg3x  1
2 2

x x 1 3x 3
d) sin 0 e) cos  f) tg 
2 3 2 4 3

106 Matemàtiques 1r Batxillerat


SOLUCIONARI UNITAT 3. TRIGONOMETRIA

 4   
 4x   2k  x  k
3  3  3 2 x x
a) sin 4 x     d) sin  0   k  x  2k
2 4 x  5  x  5   2 2
 2k  k

 3 
 12 2
     x 2
2x   2k  x   k   2k
2   8 x 1   x  2  6k
b) cos 2 x   4  e) cos     3 3 
2 2x  7  2k  x  7  k 3 2  x 4 
 2k  x  4  6k
 4  8  3 3
 3   2k  3 x 5  10 8k


3x 
4
 2k x  4  3
 3x 3  4  6  2k  x  9  3
c) tg3 x  1    f) tg   
3 x  7  2k  x  7  2k 4 3  3 x  11  2k  x  22  8k

 4 
 12 3  4 6  9 3

84. Troba totes les solucions de les equacions trigonomètriques següents.

a) sin x  cos x c) sin x  3 cos x  0

b) sin 2x  sin x  0 d) sin x  cos x  2

a) sin x  cos x  tg x  1  x  45º 180º k

sin x  0  x  180º k

b) sin 2x  sin x  0  2 sin x cos x  sin x  0  sin x(2cos x  1)  0   1  x  60º 360º k
cos x  2   x  300º 360º k
 

c) sin x  3 cos x  0  tg x  3  x  60º 180º k

d) sinx  cos x  2   sinx  cos x   2  1  2 sin x cos x  2  2 sin x cos x  1  sin 2 x  1 


2

 2x  90º  360º k  x  45º 180º k

Quan s’eleva al quadrat, apareixen solucions falses amb k imparell. La solució és


x  45º  360º k amb k  0, 1, 2...

85. Troba, per a l’interval [0, 2π], les solucions de les equacions trigonomètriques següents.

a) sin2 x  tg2 x  0 b) 2sin x  3 tg x  0

sin2 x  0  sin x  0  x  0, x   , x  2
 1  
a) sin x  tg x  0  sin x  1 
2 2 2
0 1
 cos x  1   0  Sense solució real
2
 cos2 x

sin x  0  x  0, x  , x  2
sin x  3  
b) 2 sin x  3 tg x  0  2 sin x  3  0  sin x  2   0 5 7
  3
cos x  cos x  cos x   x ,x
 2 6 6

Resolució de triangles
86. Resol els triangles rectangles següents.

a) Â  90º, a  25 mm, c  14 mm c) Ĉ  90º, Â  20º, a  12 dm

b) B̂  90º, a  28 cm, c  45 cm d) B̂  90º, Â  15º, b  15 m

107 Matemàtiques 1r Batxillerat


SOLUCIONARI UNITAT 3. TRIGONOMETRIA

c 14
a) b  252  142  20,71 mm sen Cˆ    Cˆ  34º 3 ' 21'' Bˆ  90º Cˆ  55º 56' 39''
a 25

12 12
c) Bˆ  90º  Aˆ  70º sin Aˆ   c  35,09 dm tg Aˆ   b  32,97 dm
c b

a c
d) Cˆ  90º  Aˆ  75º sin Aˆ   a  3,88 m cos Aˆ   c  14,49 m
15 15

87. Calcula l’àrea de cadascun d’aquests triangles.

a) Â  90º, a  73 mm, c  55 mm
b) c)

55  48
a) b  732  552  48  Àrea: S   1320 mm2
2

5 2 5 2
b) b  c; c  10 sin 45º  5 2 m  Àrea: S   25 m2
2

16 19,07·16
c) c   19,07 dm  Àrea: S   152,56 dm2
tg 40º 2

88. Resol els triangles següents.

a) b  20 cm, c  28 cm, Ĉ  40º d) a  12 cm, b  15 cm, Ĉ  35º

b) a  41 cm, b  9 cm, c  40 cm e) a  30 cm, B̂  30º, Ĉ  50º

c) a  3 cm, B̂  30º, c  5 cm f) b  25 cm, B̂  55º, Ĉ  65º

b c b sin Cˆ 20·sin 40º


a)   sin Bˆ    0,459  Bˆ  27º19 21
sin Bˆ sin Cˆ c 28

Aˆ  180º Bˆ  Cˆ  112º 40 39

a c c sin Aˆ 28·sin112º 40 39


 a   40,2 cm
sin Aˆ sin Cˆ sin Cˆ sin 40º

b2  c 2  a2 92  402  412
b) cos Aˆ    0  Aˆ  90º
2bc 2·9·40

a2  c 2  b2 412  402  92
cos Bˆ    0,9756  Bˆ  12º 40 ' 58 ''
2ac 2·40·41

Cˆ  180º Aˆ  Bˆ  77º 19 2

108 Matemàtiques 1r Batxillerat


SOLUCIONARI UNITAT 3. TRIGONOMETRIA

c) b2  a2  c 2  2ac cos Bˆ  32  52  2·3·5·cos30º  8,0192  b  2,83 cm

b2  c 2  a2 2,832  52  32
cos Aˆ    0,8484  Aˆ  31º 57 ' 43 ''
2bc 2·2,83·5

Cˆ  180º Aˆ  Bˆ  118º 2 17

d) c  a  b  2ab cos Cˆ  12  15  2·12·15·cos35º  74,1053  c  8,61 cm


2 2 2 2 2

b2  c 2  a2 152  8,612  122


cos Aˆ    0,6006  Aˆ  53º 5 14
2bc 2·15·8,61

Bˆ  180º Aˆ  Cˆ  91º 54 46

e) Aˆ  180º Bˆ  Cˆ  100º

a c a sin Cˆ 30 sin50º
 c    23,34 cm
sin Aˆ sin Cˆ sin Aˆ sin100º

a b a sin Bˆ 30 sin30º
 b   15,23 cm
sin Aˆ sin Bˆ sin Aˆ sin100º

f) Aˆ  180º Bˆ  Cˆ  60º

b c b sin Cˆ 25 sin 65º


 c    27,66 cm
sin Bˆ sin Cˆ sin Bˆ sin55º

b a b sin Aˆ 25 sin 60º


 a   26,43 cm
sin Bˆ sin Aˆ sin Bˆ sin55º

89. Troba l’àrea dels dos triangles que compleixen que   45º, a  6 cm i c  7,5 cm.

 1
A ac sin Bˆ  21,5 cm2
a c ˆ c sin Aˆ Cˆ  62º 6 59, Bˆ  72º 53 1
 
 2
  sin C   0,8839   
sin Aˆ sin Cˆ a  ˆ ˆ
C  117º 53 1, B  17º 6 59 A  1
ac sin Bˆ  6,62 cm2

 2

Síntesi
90. a) Demostra que l’àrea del segment circular de la figura es pot calcular per mitjà
de l’expressió:

r2
A  x  sen x 
2

b) Calcula l’àrea de la zona ombrejada.

109 Matemàtiques 1r Batxillerat


SOLUCIONARI UNITAT 3. TRIGONOMETRIA

r 2 x 1 2
a) Àrea del sector circular: A1  Àrea del triangle: A2  r sin x
360º 2

r 2 x 1 2 r 2  x 
Àrea del segment circular: A  A1  A2   r sin x    sin x 
360º 2 2  180º 

r 2  x  r
2
Considerant els angles donats en radians, l’expressió queda: A    sin x    x  sin x 
2    2
b) Apliquem l’expressió anterior per calcular l’àrea dels dos segments circulars que es formen en totes dues cir-
cumferències.
Àrea del segment circular de la circumferència de radi r1  5 cm:

52  5 2  3 2 r2
cos Aˆ1   0,82  Aˆ1  34,92º  1  2Aˆ1  69,84º  1,22 rad  A1  1 (1  sin 1)  3,51 cm2
2·5·5 2

Àrea del segment circular de la circumferència de radi r2  3 cm:

32  5 2  5 2 r2
cos Aˆ 2   0,3  Aˆ 2  72,54º  2  2Aˆ 2  145,08º  2,53 rad  A 2  2 (2  sin 2 )  8,8 cm2
2·3·5 2

Per tant, l’àrea de la zona ombrejada és A  A1  A2  12,31 cm .


2

91. a) Troba una fórmula que permeti calcular l’àrea d’un rombe coneixent les mesures del seu costat i d’un dels seus
angles.
b) Quina és l’àrea d’un rombe de 15 cm de costat si un d’aquests angles mesura 40°
2
c) Calcula els angles d’un rombe sabent que el seu costat mesura 4 cm i la seva àrea 8 cm .

a) Un rombe de costat x i un dels seus angles  es pot dividir en dos triangles isòsceles iguals d’àrea.

1 2
A x sin  , per tant, l’àrea del rombe és AR  2A  x 2 sin  .
2

b) AR  152 sin 40º  144,63 cm2 .

1
c) 8  42 sin   sin    Els angles del rombe són 30º i 150º.
2

92. Sabent que tg t:
2
 
a) Calcula cos2 i sin2 en funció de t.
2 2
b) Amb l’ajuda de les fórmules de l’angle doble, calcula sinα, cosα i tgα en funció de t.
1 sin   cos  tg 
i) ii) iii)
sin   cos  sin   cos  sin   cos 

  1  1 1   1 t2
a) 1  tg2  sec 2   cos2   i sin2  1  cos2  1  
2 2 cos2  2 1  tg2  1  t 2
2 2 1 t 2
1 t 2
2 2

110 Matemàtiques 1r Batxillerat


SOLUCIONARI UNITAT 3. TRIGONOMETRIA

    t 1 2t
b) sin   sin  2    2 sin cos  2 
 2 2 2 1 t 2 1 t 2 1 t 2

    1 t2 1 t 2
cos   cos  2    cos2  sin2   
 2 2 2 1 t 2 1 t 2 1 t 2

sin  2t
tg   
cos  1  t 2

QÜESTIONS
93. Quins són els valors màxim i mínim de sin α, cos α, tg α, cosec α, sec α i cotg α?

El valor mínim de sin  i cos  és 1 i el valor màxim és 1.


El valor mínim i màxim de la resta de valors no està definit, tg  i cotg  poden tenir qualsevol valor, sec  i cosec
 poden tenir qualsevol valor excepte els que pertanyen a l’interval (1, 1).

94. Indica tots els angles positius i més petits que 360° tals que la seva tangent coincideix amb la seva cotan-
gent.

La tangent coincideix amb la cotangent per a aquells angles en què el valor és 1 i 1. Per tant, els angles positius
inferiors a 360º que compleixen aquesta condició són 45º, 135º, 225º i 315º.

PROBLEMES
95. Un globus està subjecte a una corda de 10 m de llargada. A causa de l’acció al vent, el globus s’ha despla-
çat de la vertical del punt de subjecció i es troba a una altura de 8 m. Calcula la inclinació de la corda res-
pecte de la línia de terra.

8
Sigui  la inclinació buscada, tenim: sin      53º 7 ' 48 '' .
10

96. En una ciutat, al migdia del solstici d’estiu, els raigs solars tenen una inclinació de 73° 3′. Calcula la longi-
tud de l’ombra d’un edifici de 52 m d’altura.

52
Sigui x la longitud de l’ombra, tenim: tg73 º 3 '   x  15,85 m.
x

98. Un senyal de trànsit indica que el pendent d’un tram de carretera és del 8 %, cosa que vol dir que en un
desplaçament horitzontal de 100 m es fa un ascens de 8 m d’altura.
a) Quin angle forma la carretera amb l’horitzontal?
b) Quants metres cal recórrer per pujar 125 m?

8
a) Sigui  l’angle buscat, tenim: tg      4º 34'' 26''
100
125
b) Sigui x el recorregut demanat, tenim: sin    x  1567,5 m
x

111 Matemàtiques 1r Batxillerat


SOLUCIONARI UNITAT 3. TRIGONOMETRIA

ambunun
99. Des d’un punt, que és a 20 m del peu d’una torre de 10 m d’altura, veiem el punt més alt d’aquesta sota
dobledede
cert angle. Quina distància hem de recórrer cap a la torre per veure’l amb un angle que sigui el doble
l’anterior?

10 2 tg  1 4 10
tg   ; tg2      4 x  50  x  12,5 m
20 1  tg2  1  1 3 20  x
4

100. Dos cotxes, amb velocitats constants respectives de 90 i 80 km/h, viatgen per una carretera que es bifurca
en dues que formen un angle de 82° i són rectes. Si arriben alhora a la bifurcació i cada cotxe pren una de
les branques, quina distància hi haurà entre ells quan portin 15 minuts de viatge.

Siguin e1 i e2 els espais que han recorregut els dos cotxes en 15 min  0,25 h i sigui x la distància buscada, tenim:

e1  90  0,25  22,5 km


  x  22,5  20  2  22,5  20 cos82º  27,95 km
2 2
e2  80  0,25  20 km 

101. Calcula l’altura dels dos edificis de la figura.

Sigui x l’altura del primer edifici i y la del segon. Tenim:


x
tg33º 42'   x  24 tg33º 42'  16 m
24

y x
tg26º 36'   y  x  24 tg26º 36'  12 m  y  12  16  28 m
24

102. Dues ciutats, A i B, estan situades sobre el mateix meridià de l’esfera terrestre, men-
tre que la ciutat C es troba en el mateix paral·lel que A. La latitud de A és de α = 40°
nord.
B
a) Si la ciutat B és 150 km al nord de A, calcula la seva latitud sabent que el radi de la
C A
Terra és d’uns 6370 km.
b) Si la ciutat C està situada sobre el mateix paral·lel, 30° a l’oest de A, quina distància
separa aquestes dues ciutats?

a) Recordem que la longitud d’un arc d’amplitud  graus i d’una circumferència de radi r
r 
és L  :
180º

   40º 6370 
180º   150 
 
180º L
  180º   41º 21
r 6370
r
b) Es calcula en primer lloc el radi del paral·lel corresponent: sin50º   r  4879,7 km
6370

r 
L  2555 km
180º

112 Matemàtiques 1r Batxillerat


SOLUCIONARI UNITAT 3. TRIGONOMETRIA

103. Calcula l’angle de tir del jugador que està situat en el punt B del camp.

 26,34
 tg CBA   0,5268  CBA  27º 46 49
 50
  DBC  DBA  CBA  6º 10 5
tg DBA  33,66  0,6732  DBA  33º 56 54

 50

104. Les bases d’un trapezi isòsceles mesuren 10 i 5 cm, respectivament. L’angle que forma la base més gran
amb cadascun dels costats no paral·lels és de 35°. Calcula l’altura, el perímetre i l’àrea del trapezi.

h
tg35º   h  1,75 cm
2,5

2,5
cos35º   x  3,05 cm  P  21,1 cm
x
(10  5)·1,75
A  13,13 cm2
2

105. Des d’un satèl·lit GPS s’estableix la posició d’un cotxe respecte d’un punt de referència fix a la Terra. Les
distàncies des del punt fix i el cotxe al satèl·lit són 21 364 i 29 079 km, respectivament. Si la línia que uneix
el punt fix amb el satèl·lit forma un angle amb el terra de 71°, i la que uneix el cotxe amb el satèl·lit forma
un angle de 44°, quina distància separa el cotxe del punt fix? A quina altura és el satèl·lit??

Observem el dibuix i tenim:


h  21 364  sin71  20 200 km

x  y  21364·cos71º 29079·cos 44º  27873,12 km

106. Calcula la distància entre els punts A i B.

AD 7,2
  AD  5,77 m
sin50º sin73º

BD 5,25
  BD  4,13 m
sin 48º sin71º

AB2  5,772  4,132  2, 5,77  4,13cos 180º 73º 71º   11,79  AB  3,43 m

113 Matemàtiques 1r Batxillerat


SOLUCIONARI UNITAT 3. TRIGONOMETRIA

107. Calcula l’angle α que formen la diagonal del cub i la diagonal d’una cara d’aquest.

Sigui a l’aresta del cub, D  a2  a2  a2  3a2  a 3 i d  a2  a2  a 2 , per tant, tenim:

d 2 6
cos        35º 15 52
D 3 3

108. Calcula l’amplitud de l’angle  de la figura.

La figura es pot dividir en dos triangles iguals, ja que tenen els tres costats
iguals, per tant:
 a 1 
tg     26º 33 54    53º 7 48
2 2a 2 2

109. Calcula l’altura, el perímetre i l’àrea del trapezi de la figura.

Altura: h  6tg 40º  5,03 cm


Costat restant: x  6cos 40º  4,6 cm
Perímetre: 23,63 cm
10  4
Àrea: ·5,03  35,21 cm2
2

110. Un home que està situat a l’oest d’una emissora de ràdio observa que el seu angle d’elevació és de 45°. Ca-
mina 50 m cap al sud i observa que l’angle d’elevació és ara de 30º. Troba l’altura de l’antena.

La distància inicial a l’antena és igual a l’altura h, ja que l’angle en el primer


punt és de 45º.
h
des del segon punt, la distància a l’antena és  3h .
tg30º

Com que és un triangle del terra rectangle tenim:

 3h 
2
h2  502   3h2  h2  1250  h  35,36 m

114 Matemàtiques 1r Batxillerat


SOLUCIONARI UNITAT 3. TRIGONOMETRIA

111. Dues persones, que estan separades per 2 km de distància, veuen, sobre el seu pla vertical i en el mateix
moment, un núvol sota angles de 73° 18′ i 84° 17′, respectivament.
Calcula l’altura del núvol i la distància que hi ha entre aquest i cadascun dels observadors.

Hi ha dues possibles interpretacions del problema.


Si el núvol està situat entre els dos observadors, tenim:
b 2
  b  5,02 km
sin73º18' sin22º 25'

a 2
  a  5,22 km
sin84º17' sin22º 25'

h  b sin84º 17'  5 km

Si el núvol està situat a un mateix costat dels dos observadors, tenim


b 2
  b  10,05 km
sin73º 18' sin10º 59'

a 2
  a  10,45 km
sin95º 43' sin10º 59'

h  b sin84º 17'  10 km

PER APROFUNDIR
112. Per al triangle de la figura i la circumferència circumscrita a aquest demostra l’afirmació donada en cada
cas.
a b c
a) Es compleix la relació: r    (Tingues en compte la relació
2 sin Aˆ 2 sin Bˆ 2 sin Cˆ
entre els angles B̂ i B̂ )
abc
b) L’àrea del triangle es pot calcular com: A 
4r

a) Bˆ  Bˆ  , ja que tots dos són angles inscrits a la mateixa circumferència i determinen el mateix arc.

b sin Aˆ b b sin Aˆ b a a b c
sin Bˆ  i sin Bˆ     r  r   
a 2r a 2r 2 sin Aˆ 2 sin Aˆ 2 sin Bˆ 2 sin Cˆ

1 1 c abc
b) A  ab sin Cˆ  ab 
2 2 2r 4r

115 Matemàtiques 1r Batxillerat


SOLUCIONARI UNITAT 3. TRIGONOMETRIA

113. Observa la figura següent.

a) Si les diagonals d’un quadrilàter mesuren d i D unitats lineals, respectivament, i formen un angle α, demostra
1
que l’àrea d’aquest quadrilàter pot calcular-se amb la fórmula: A  dD sin 
2
b) Calcula l’àrea d’un quadrilàter les diagonals del qual formen un angle de 80° si mesuren 4 i 5 cm, respectiva-
ment.

a) El quadrilàter es pot dividir en quatre triangles, com que


sin  180º    sin  es pot escriure:

1 1
A sin   xy  x  D  y   y  d  x    D  y   d  x   dD sin 
2 2

1 1
b) A  Dd sin   4  5 sin80º  9,85 cm2
2 2

114. Considera les dues circumferències coplanàries de la figura.

Calcula la inclinació sobre la recta que uneix els centres de:


a) La tangent comuna exterior. b) La tangent comuna interior.

64 64
a) sin      9º 35 39 b) sin      56º 26 34
12 12

116 Matemàtiques 1r Batxillerat


SOLUCIONARI UNITAT 3. TRIGONOMETRIA

ENTORN MATEMÀTIC

Parada inesperada al pont


Un grup d’estudiants és a Londres aprenent
anglès. Com a activitat complementària fan
una visita a Westminster. Tanmateix, quan
van a creuar el pont es troben que està ta-
llat per un accident. Per entretenir-se men-
tre esperen, el seu professor, Mr. Clever, els
proposa el problema següent:
Can you estimate the height of Big Ben To- h
wer and how far we are from it?
x
Els nois estan una mica desconcertats per- A B
què no saben com aconseguir la informació
que els demana Mr. Clever (Internet via
Smartphone is not allowed).
En aquest moment Bonneidée, una estudiant francesa, recorda les seves classes de trigonometria i proposa un
pla per solucionar el problema. Posa l’Edu al punt A, el Fer, al B, i mesura en passes la distància entre tots dos,
obtenint un valor de 125 passes d’uns 80 cm cadascuna. Després, estima els angles α i β formats pel pont i les
línies que uneixen A i B amb el punt més alt del Big Ben, obtenint que α = 18° i β = 26.
Amb aquesta informació, Bonneidée afirma que pot calcular l’altura de la torre i la distància a aquesta. Creus
que és cert o s’està marcant un bluf davant dels seus amics i de Mr. Clever? Si penses el primer, demostra-ho
calculant tu mateix les quantitats demanades (x i h en la figura).

 h
tg26º  x 0,488 x
  h  x tg26º  0,488 x  0,325   x  199,39 m; h  97,3 m
tg18º  h 100 x
 100  x

El London Eye
Com que el pont no s’obre, Mr. Clever proposa, provocant un
altre ensurt al grup, que ja es veu havent de resoldre un nou
problema, una alternativa: A

Would you like to visit the London Eye?


Encara que n’hi ha alguns que els fa por pujar a la gegantina
roda del mil·lenni, a altres els sembla que pot ser excitant i
que les vistes i les fotografies des d’allà dalt valdran la pena.
Al final decideixen caminar pel costat del Tàmesi fins a la fa-
mosa roda i allí el grup es divideix entre els atrevits que hi
pujaran i els més porucs: en Fer, la Clara i, of course, Mr.
Clever que es queden esperant a baix.
Quan han passat uns minuts, la roda s’atura de sobte per una avaria i comencen a sentir-se protestes des de les
cabines. Els nois observen que la cabina on van els seus companys ha quedat en la posició A. Per tenir ocupats
els nois mentre s’arregla el problema, Mr. Clever sense perdre la seva calma britànica comenta:
It seems like the London Eye is not working very well today! Hey guys, can you tell your friends what height
they are at?
En Fer i la Clara tracten de resoldre el problema. Les úniques dades amb què compten són les que s’indiquen a
l’entrada de l’atracció: altura màxima de 135 m i disposa de 30 cabines.
Què haurien de fer aquests nois per estimar l’altura a la qual estan cridant els seus companys?
Observa que l’angle entre dues cabines, mesurat des del centre O de la roda, és de 12º,
per tant: h  67,5sin24º  27,5 m

Per tant, l’altura a la qual es troba la cabina dels seus companys és 67,5  27,5  95 m.

117 Matemàtiques 1r Batxillerat


SOLUCIONARI UNITAT 3. TRIGONOMETRIA

AUTOAVALUACIÓ

Comprova el que has après


1. Converteix en graus o en radians, segons el cas, els angles següents.

a) 65º b) 138º c) 4 rad d) rad
10

65 13 4·180


a) 65   rad c) 4 rad   229º 11
180 36 

138 23   ·180


b) 138   d) rad   18º
180 30 10 10

2. Escriu en funció d’un angle entre 0 i 45° les raons trigonomètriques següents.
a) sin 120º b) cos 480º c) tg (430º)

a) sin120º  sin 180º 40º   sin 40º

b) cos 480º  cos120º  cos 180º  60º    cos60º   sin30º

c) tg  430º    tg430º   tg70º   cotg20º

3. Calcula la raó demanada en cada cas.


1
a) cos , si tg   4 i   III b) tg , si sin    i   IV
5

1 17
a) 1  tg2  sec 2   sec    1  tg2   1 16   17  cos    
17 17

1 6
b) 1  cotg2   cosec 2   cotg    cosec 2   1   25  1   24  2 6  tg    
2 6 12

4. Si sin   0,6 i sin   0,8, sent tots dos angles del primer quadrant, calcula el valor de:
a) sin (  ) b) cos ( − ) c) tg (  )

3 4 3 4 3 4
sin   0,6   cos   i tg   sin   0,8   cos   i tg  
5 5 4 5 5 3

3 3 4 4
a) sin      sin  cos   cos  sin   ·  ·  1      90º
5 5 5 5

4 3 3 4 24
b) cos       cos  cos   sin  sin   ·  ·   0,96
5 5 5 5 25

1
c)     90º  tg      no existeix perquè cos      0 i tg      
0

118 Matemàtiques 1r Batxillerat


SOLUCIONARI UNITAT 3. TRIGONOMETRIA

3 
5. Si  és un angle del segon quadrant i cos    , calcula les raons de i de 2.
4 2


Com que  és un angle del segon quadrant, el sinus és positiu, la tangent negativa i pertany al primer qua-
2
3 7 7
drant. Tenim cos     sin   i tg    i, per tant:
4 4 3

 1  cos  7 7 14  4 2 14


sin       cosec    
2 2 8 2 2 4 2 14 7

 1  cos  1 1 2  4 4 2
cos     sec     2 2
2 2 8 2 2 4  
2 2 2

 1  cos   1 7
tg   7 cotg    
2 1  cos   
2 7 7

D’altra banda:

3 7 8 8 7
sin 2  2 sin  cos    cosec 2  
8 3 7 21

1
cos 2  cos2   sin2   sec 2  8
8

2 tg  1 7
tg2   3 7 cotg2   
1  tg2  3 7 21

6. Resol les equacions trigonomètriques següents.


sin 
a) sin   4 cos  b) tg   
2

a) sin   4cos   tg   4    75,96º 180º k

sin  sin  sin  sin   0    0º 180º k


b) tg      
2 cos  2 cos   2  No té solució real

7. ˆ  70º , Bˆ  30º i b  12 cm
Resol i calcula l’àrea del triangle amb les mesures: A

Cˆ  180º Aˆ  Bˆ  80º

a b b sin Aˆ 12 sin70º c b b sin Cˆ 12 sin80º


 a   22,55 cm  c    23,63 cm
sin Aˆ sin Bˆ sin Bˆ sin30º sin Cˆ sin Bˆ sin Bˆ sin30º

1
Àrea: A  ac sin Bˆ  133,21 cm2
2

8. Els costats d’un rombe mesuren 6 cm i formen entre ells angles de 60° i 120°. Troba les longituds de les dia-
gonals del rombe.

El rombe es divideix en quatre triangles rectangles com el de la figura.

a  6sin30º  3 cm b  6cos30º  3 3 cm

Per tant, la diagonal més gran del rombe mesura 2b  6 3 cm i la diagonal més petita mesura

119 Matemàtiques 1r Batxillerat


SOLUCIONARI UNITAT 3. TRIGONOMETRIA

2a  6 cm .

3
9. Els angles interiors d’un polígon regular tenen per cosinus  .
2
a) Troba el nombre de costats que té aquest polígon.
b) Hi ha més d’una solució a la pregunta a)?

 3
Existeixen dos angles el cosinus del qual és igual a ,   150º i   210º (>180º) aquest darrer no és inte-
2
rior, ja que els interiors són més petits que 180º. Apliquem la fórmula per calcular la mesura dels angles interiors
d’un polígon regular de n costats:
180(n  2) 180(n  2)
  150º   150º n  180º n  360º  n  12
n n
Per tant hi ha una única solució: n  12 costats.

10. Des d’un punt del terra es veu la capçada d’un pi sota un angle de 42°. Si ens allunyem 2,5 m cap a un altre
punt del terra, alineat amb l’anterior i el peu del pi, veiem la capçada amb un angle de 24°. Calcula l’alçada
del pi.

Sigui h l’altura del pi i x la distància del peu del pi al primer punt. Tenim:

h 
tg 42º  
x  0,9 x
  h  x tg 42º  0,9 x  0,445   1,1125  0,445 x  0,9 x  x  2,45 m  h  2,2 m
h  2,5  x
tg24º 

2,5  x 

RELACIONA I CONTESTA
Tria l’única resposta correcta en cada cas

u2  v 2 u2  v 2
1. Un triangle té per costats: a  uv , b  i c amb u i v dos nombres reals positius tals que
2 2
u > v.
A. Es tracta d’un triangle rectangle a A. C. Es tracta d’un triangle rectangle a C.
B. Es tracta d’un triangle rectangle a B. D. No és un triangle rectangle.

Ex compleix que c 2  a2  b2 , per tant, la resposta correcta és la C.

5
2. Donada la funció , el valor mínim s’obté quan:
5  4 cos 5
A.   2k  rad amb k  C.     2k  rad amb k 

 2 
B.    k  rad amb k  D.    2k  rad amb k 
5 5 5

Com que 5  4cos5  0 el valor mínim de la funció s’assoleix quan cos5 és mínim, és a dir, quan cos5  1,
per tant, la resposta correcta és la B.

120 Matemàtiques 1r Batxillerat


SOLUCIONARI UNITAT 3. TRIGONOMETRIA

3. Un dels casos de resolució de triangles següents té dues solucions diferents. Indica quina és.

A. a  4 cm, b  6 cm, c  15 cm C. a  5 cm, b  10 cm, Cˆ  20º

B. a  5 cm, b  10 cm, Aˆ  20º D. a  3 cm, b  4 cm, c  5 cm

La resposta correcta és la B.

Assenyala, en cada cas, les respostes correctes


4. L’equació trigonomètrica sin2 x  cos2 x té:
   3 
A. Una solució s 0,  C. Tres solucions s 0, 
 2  2 

B. Dues solucions s 0,  D. Quatre solucions a 0, 2

 3
sin2 x  cos2 x  tg2 x  1  tg x  1  x   k, x   k , per tant, totes les respostes són correctes.
4 4

5. A la figura següent:

A. Aˆ  Dˆ  180º B. Aˆ  Cˆ  180º C. Bˆ  Dˆ  180º D. Aˆ  Bˆ  Cˆ  Dˆ  360º

La B és correcta, ja que la mesura d’un angle és igual a la meitat de la mesura angular de l’arc que abraça. Si l’an-
x 360º  x
gle inscrit  abraça un arc x, l’angle inscrit Ĉ abraça un arc de 360º  x, i Aˆ  Cˆ    180º . La C és
2 2
correcta pel mateix raonament i la D és correcta perquè es dedueix dels dos casos anteriors.

Tria la relació correcta entre les dues afirmacions que es fan

6. Tres punts del pla, no alineats, determinen un triangle. Es consideren les afirmacions següents:
1. El triangle és isòsceles 2. El triangle té dos angles els sinus dels quals són iguals.
A. 1  2 C. 2  1 però 1  2

B. 1  2 però 2  1 D. 1 i 2 s’exclouen entre elles.

La B és correcta, si un triangle és isòsceles, tindrà dos angles iguals i per tant amb el mateix sinus, però si un trian-
gle té dos angles amb el mateix sinus, pot ser isòsceles (i per tant també equilàter).

121 Matemàtiques 1r Batxillerat


SOLUCIONARI UNITAT 3. TRIGONOMETRIA

Assenyala la dada innecessària per contestar



7. Per calcular la tangent de l’angle es donen les dades següents:
2
1. tg   0 2. cos   0 3. 1  sin2   1,64

A. Pot eliminar-se la dada 1. C. Pot eliminar-se la dada 3.


B. Pot eliminar-se la dada 2. D. Calen les tres dades.


No n’hi ha prou amb la 1 i la 2 per calcular tg , per la qual cosa la dada 3 és necessària.
2
 1  cos 
1 sin2   1,64  sin2   0,64  cos2   1 sin2   0,36  cos   0,6 i tg 
2 1  cos 
També és necessària la 2 per conèixer el signe de cosinus. A més, si cos   0 ,  pertany al segon o al tercer
 
quadrant, per la qual cosa pot pertànyer al primer o segon quadrant i no queda determinat el signe de tg ,
2 2

així doncs, també necessitem la 1 per concloure que  pertany al tercer quadrant i, per tant , pertany al segon
2
i la seva tangent és negativa. La resposta correcta és la D.

122 Matemàtiques 1r Batxillerat


Activitats de reforç Unitat 3. Trigonometria

DIBUIXANT ANGLES

Amb l’ajuda de la circumferència goniomètrica (radi  1) podem dibuixar angles coneixent el valor d’alguna de les
seves raons trigonomètriques.
3
Vegem com es dibuixa un angle el sinus del qual mesuri . Com que el valor del sinus és positiu, pot ser un angle
5
del primer o del segon quadrant. Observem les dues possibilitats:
Primer quadrant Segon quadrant

3 3 3 3
sin   0,6    arcsin   36,86º sin   0,6    arcsin   180º 36,86º  143,13º
5 5 5 5
Observem altres exemples:
1
Dibuixa l’angle  sabent que cos   0,32 i que Dibuixa l’angle  sabent que tg   
2
l’angle és en el tercer quadrant. i que l’angle és en el quart quadrant.

1. Dibuixa els angles següents en el quadrant indicat.


2
a) sin   , 3r quad. c) tg   3 , 3r quad. e) tg   1, 2n quad. g) tg  1 , 4t quad.
3
1 3 6
b) cos   , 4t quad. d) cos   1r quad. f) sin    3r quad. h) tg   4 , 1r quad.
9 4 7

Si el que coneixem és una de les raons inverses, per exemple, sec   3 i sabem que
l’angle pertany al tercer quadrant, calculem el valor de la raó inversa i dibuixem l’angle
igual que a l’apartat anterior:
1
sec   3  cos  
3

2. Dibuixa els angles les raons dels quals són:


1 2
a) cosec   4 , 3r quad. c) cotg   , 3r quad. e) cotg    , 2n quad. g) sec   2,8 , 3r quad.
5 3
5
b) sec   1,5 , 4t quad. d) sec   6 , 1r quad. f) cosec   2 , 4t quad. h) cotg   , 1r quad.
3

1 Matemàtiques 1r Batxillerat
Activitats de diversitat Unitat 3. Trigonometria
SOLUCIONARI

REFORÇ
Fitxa: Dibuix d’angles
1. a) c) e) g)

b) d) f) h)

2. a) c) e) g)

b) d) f) h)

1 Matemàtiques 1r Batxillerat
Activitats de reforç Unitat 3. Trigonometria

REDUCCIÓ AL PRIMER QUADRANT


tambéles
Si coneixem les raons trigonomètriques d’un angle que és en el primer quadrant podem conèixer també lesraons
raons
trigonomètriques inverses d’altres angles relacionats.
4
Si, per exemple, cos   , aleshores:
5
1 1 4  1 1 3
sec(  )     sec    cotg(  )    tg 
cos(  )  cos  5 2 tg(90  ) cotg 4

1 1 4 3
cosec()    cosec   1  ( )2  
sin() sin 5 5

1. Fixa’t en els exemples i completa la taula.


secant cosecant cotangent
180   sec 
o

 cosec 
90º   tg 
360º  
180º  

2. Expressa la mesura dels angles següents, relaciona cada angle amb un angle del primer quadrant del
qual coneguis les raons, i emplena la taula amb les seves raons trigonomètriques.

Angle 1r
Mesura Radians sinus cosinus tangent secant cosecant cotangent
quadrant
2
120º
3
135º

150º

210º

225º

240º

300º

315º

330º

3. Calcula el valor de les raons trigonomètriques següents:


9
a) sin 540º c) cos 390º e) tg 750º g) sec i) cosec 2190º
4
 
b) cotg 630º d) sin 1380º f) cos    h) cotg (150º) j) sec 1260º
 4

1 Matemàtiques 1r Batxillerat
Activitats de diversitat Unitat 3. Trigonometria
SOLUCIONARI

Fitxa: Reducció al primer quadrant


1. secant cosecant cotangent
180  sec  cosec  cotg 
o

 sec  cosec  cotg 


90º  cosec  sec  tg 
360º   sec  cosec  cotg 
180º   sec  cosec  cotg 

2.
Mesura rad Angle 1r quadrant sinus cosinus tangent secant cosecant cotangent
2 3 1 2 3 3
120º 120º  180º  60º   3 2 
3 2 2 3 3
3 2 2
135º 135º  180º  45º  1  2 2 1
4 2 2
5 1 3 3 2 3
150º 150º  180º  30º    2  3
6 2 2 3 3
7 1 3 3 2 3
210º 210º  180º  30º    2 3
6 2 2 3 3
5 2 2
225º 225º  180º  45º   1  2  2 1
4 2 2
4 3 1 2 3 3
240º 240º  180º  60º   3 2 
3 2 2 3 3
7 2 2 2 2
315º 315º  360º  45º  1  1
4 2 2 2 2
3 3 1 2 3 3
300º 300º  360º  60º   3 2  
2 2 2 3 3
11 1 3 3 2 3
330º 330º  360º  30º   2  3
6 2 2 3 3

3. a) sin 540º  sin (360º  180º)  sin 180º  0 1


f) cotg 600º  cotg (360º  240º)
3 3
b) cos 390º  cos (360º  30º)  cos 30º 
2 3
g) sin 1380º  sin ( 360º · 4  60º) 
3 2
c) tg 750º  tg ( 2 · 360º  30º )  tg 30º 
3  2
h) cos ( ) cos (45º) 
9 2 4 2
d) sec  sec 405º  sec (360º  45º) 
4 2 i) cotg (150º)  cotg ( 360  150)  3
e) cosec 2190º  cosec (360º · 6 30º)  2 j) sec 1260º  sec ( 360º· 3  180º)  1

2 Matemàtiques 1r Batxillerat
Activitats de reforç Unitat 3. Trigonometria

CÀLCUL DE RAONS A PARTIR D’ALTRES DE CONEGUDES


calculartotes
Si un angle mesura el doble o la meitat d’un altre angle les raons del qual coneixem, en podem calcular les seves
les
raons trigonomètriques sense necessitat de conèixer-ne la mesura exacta. Vegem-ne uns quants exemples:
4
 Si sabem que el valor del sinus d’un angle  del segon quadrant és les raons trigonomètriques de l’angle
5

 i de l’angle es poden trobar de la manera següent:
2
2
4 16 9 3 sin 4
   cos   1  cos   1   cos2    cos    tg   
2 2

5 25 25 5 cos  3
3
1
4 (3) 24  1  cos  5  2 2 5
sin 2  2sin cos   2    sin  
5 5 25 2 2 2 5 5
3
1
 3   4   1  cos 
2 2
7 5  1  5
cos 2  cos   sin   
2 2
     cos  
 5  5 5 2 2 2 5 5
 4  3
2  1
 3   24  1  cos  5 2
tg 2  tg  
 4 
2
7 2 1  cos  1
3
1  
 3  5

 Si el que es coneix és una de les raons inverses, per exemple, sec   3, i l’angle pertany al primer quadrant,
aleshores:
1 2 2
cos   sin  tg  2 2
3 3
4 2 7 4 2
sin 2  cos2   tg 2  
9 9 7
 1 3  2 6  1 2
sin   cos   tg  
2 3 3 2 3 3 2 2 2

1 3
1. Sabent que sin  , cosec   2 i tg   i que   I,   IV i   III , calcula:
4 4

a) sin 2 c) tg 2 e) sec g) cos (  )
2
 
b) cos d) sin () f) cos 3 h) cotg
2 2

1 Matemàtiques 1r Batxillerat
Activitats de diversitat Unitat 3. Trigonometria
SOLUCIONARI
Fitxa: Càlcul de raons a partir d’altres de conegudes
15 15
1. cos   tg  
4 15
1 3 3
sin    cos   tg   
2 2 3
4 3
cos    tg  
5 4

1 15 15
a) sin 2  2   
4 4 8

15
 1
b) cos   4   4  15
2 2 8
 3
2  
c) tg 2   3   3
2
 3
1   
 3 
1 3 15  1  3  15
d) sin           
4 2 4  2 8
 1 2 2 2
e) sec       10
2 cos  1  cos  1
4 1
2 5 5

1 3  3  1 
f) cos3  cos  2     cos2 cos   sin 2sin          0
2 2  2  2 
2
 3   1 2 1  1  3  3
cos2        sin 2  2       
 2   2 2  2  2  2

 4  3   3  1 3  4 3
g) cos                 
 5  2   5  2 10

 1 1 4  15
h) cotg   
2 tg  15 4  15
2 1
4
15
1
4

3 Matemàtiques 1r Batxillerat
Activitats de reforç Unitat 3. Trigonometria

EQUACIONS TRIGONOMÈTRIQUES

Resoldre una equació trigonomètrica és buscar el valor o els valors que compleixen l’equació. Per fer-ho, s’utilitzen
diferents tècniques, segons com sigui l’equació. A vegades, n’hi ha prou amb trobar el valor amb la funció arc,
d’altres s’ha d’utilitzar una de les igualtats trigonomètriques que coneixem, fer un canvi de variable o tan sols
treure factor comú i operar.
Un cop s’obté la solució, cal tenir en compte que es pot complir per a angles més grans de 360º.
4cos2x  1  3cos x  4(cos2 x  sin 2 x)  1  3cos x  4cos2 x  4sin 2 x  1  3cos x 
 4cos2 x  4sin 2 x  3cos x  1  0
Ho expressem tot en funció d’una mateixa raó per poder fer un canvi de variable:

4cos2 x  4(1  cos2 x )  3cos x  1  0  8cos2 x  3cos x  5  0


cos x  1  x  180  360k
5  
cos x  t  8t 2  3t  5  0  t  1, t    5  k
8 cos x  
 8  x 51 18 36  360 k

1. Troba la solució de les equacions trigonomètriques següents a l’interval [0,2].


1
a) 2sin x  1 b) 1  cos x  c) cos x  0 d) 3 tg x  1
2

2. Calcula la solució en radians d’aquestes equacions trigonomètriques.


3
a) sin 2 x  1 b) cos2 x  2cos x  1 c) tg 2 x  1  0 d) cos2 x  cos x  1  0
2

3. Calcula en graus la solució de cada equació:


1
a) 3sin 2 x  6cos2 x  1 c) cos x tg x  sin x  1 e) cos x sin2x  cos2x sin x  
4
3sin2x 1 x
b)  d) 3 cos x  sin f) cos2 x  1  sin 2 x
5cos x 10 2

4. Extreu el factor comú per resoldre aquestes equacions trigonomètriques i troba les solucions en el
primer quadrant.
a) sinx cos x  cos x  0 b) sin 2 x  4sin x  0 c) 15sin 3 x  8sin 2 x  sin x

5. Resol aquests sistemes d’equacions trigonomètriques:


x  y  0 cos x  cos y  3 y  2x tg( x  y )  1
a)  b)  c)  d) 
cos x  sin y  1 cos x  cos y  0 sinx  2siny tg x  tg y  1
2 2

1 Matemàtiques 1r Batxillerat
Activitats de diversitat Unitat 3. Trigonometria
SOLUCIONARI
Fitxa: Equacions trigonomètriques
1 1
1. a) 2sin x  1  sin x   x  arcsin  x  30º, x  150º
2 2
1 1 1
b) 1  cos x   cos x   x  arccos  x  60º, x  300º
2 2 2
c) cos x  0  x  arccos0  x  90º, x  270º

1  1 
d) 3tg x  1  tg x   x  arctg    x  150º, x  330º
3  3

 
 x   2k  
sin x  1  2
2. a) sin2 x  1    La solució es pot expressar com x   k , k 
sin x   1 x  3 2
 2k 

 2
b) cos2 x  2cos x  1  cos2 x  2cos x  1  0 cos x  t

t 2  2t  1  0  (t  1)2  0  t  1  cos x  1  x    2k   x  (2k  1), k 

 
 x   k
tgx  1  4
c) tg 2 x  1  0  tg 2 x  1    k
tg x  1  x  3  k 

 4
3
d) cos2 x  cos x  1  0 cos x  t
2
cos x  2 No existeix cap valor de x 
3 1 
t2  t  1  0  t  2, t    1 1  5
2 2 cos x   x  arccos  x   2k  x  2k 
 2 2 3 3

4 Matemàtiques 1r Batxillerat
Activitats de diversitat Unitat 3. Trigonometria
SOLUCIONARI
3. a)
3sin 2 x  6cos2 x  1  3sin 2 x  6(1  sin 2 x)  1  3sin 2 x  5

5 5
 sin2 x   sinx   No té solució a
3 3
3sin2x 1 6sin x cos x 1 6 1 1 x 4º 46' 48'' 360º k
b)     sin x   sin x   k
5cos x 10 5cos x 10 5 10 12 x 175º12'36'' 360º k

sin x
c) cos x tg x  sin x  1  cos x   sin x  1
cos x

1 x  210º 360º k
sin x  sin x  1  sin x   k
2 x  330º 360º k

x 1  cos x 1  cos x
d) 3 cos x  tg  3 cos x   3cos2 x   6cos2 x  1  cos x 
2 2 2

 6cos2 x  cos x  1  0 cos x  t

 1  x  120º 360º k
cos x 
6t 2  t  1  0  t 
1 1 

,t    2 
 x  240º 360º k
k
  
2 3 cos x  1  x 70º 31 12  360º k

 3  x 109º 28 12  360º k

1 1
e) cos x sin2x  2cos2xsin x    2cos2 xsin x  2(cos2 x  sin2 x)sin x   
4 4
1 1 1 1 x  210º 360º k
 2sin3 x    sin3 x    sinx  3      k
4 8 8 2 x  330º 360º k

f) cos2 x  1  sin2 x  cos2 x  sin2 x  1  cos2x  1  2x  360º k  x  180º k, k 

cos x  0
4. a) sinx cos x  cos x  0  cos x(sinx  1)  0   
sinx  1  0

 
x 
 2

sinx  1 No hi ha solució en el primer quadrant

sinx  0  x  0º
b) sin2 x  4sinx  0  sinx(sinx  4)  0  
sinx  4 No té solució

c) 15sin3 x  8sin2 x  sinx  15sin3 x  8sin2 x  sinx  0  sinx(15sin2 x  8sinx  1)  0 

sinx  0  x  0º
  1
  sinx   x 11º 31 48
 1 1  5
15sin x  8sinx  1  0 t  15t 2  8t  1  0  t  , t   
2

5 3 sinx  1  x
sinx
 19º 28 12
  3

5 Matemàtiques 1r Batxillerat
Activitats de diversitat Unitat 3. Trigonometria
SOLUCIONARI

x  y  0 x  y
5. a)    cos2 x  sin2 x  1  cos2x  1  2x  2k   x  k 
cos x  sin y  1 cos x  sin x  1
2 2 2 2

 x  k , y  k  k

 
3  x  6  2k  k 

b) 2cos x  3  cos x  
2  x  11  2k  k 

 6

 5
 y  2k  k
 3  6
2cos y  3  cos y  
2  y  7  2k  k

 6

y  2x
c)   sinx  2sin2x  sinx  2sin2x  0  sinx  4sinx cos x  0 
sinx  2siny

sinx  0  x  180º k k 

 sinx(1  4cos x )  0   1 x  75,52º 360º k k 
1  4cos x  0  cos x  4  x  284,47º 360º k k 

 tgx  tgy
tg( x  y )  1  1 1 tgx  0
d)   1  tgx tgy   1  1  1  tgx tgy  tgx tgy  0   
tgx  tgy  1 tgx  tgy  1 1  tgx tgy tgy  0

  
  y   2k  k 
       4
 tg x 0 x k tg y 1 
 y 5
  2k  k 
  4

  
x   2k  k 
tgy  0  y  k   tgx  1   4
 
x 5
   2k  k 
  4

6 Matemàtiques 1r Batxillerat
Activitats d'ampliació Unitat 3. Àlgebra

EQUACIONS TRIGONOMÈTRIQUES MÉS COMPLEXES

A continuació resoldrem equacions trigonomètriques una mica més complicades amb l’ajuda d’expressions alge-
braiques.

1. Demostra la igualtat a b  (a  b) (a  ab  b ).
3 3 2 2

Nota: Utilitza la regla de Ruffini en la divisió (x b) : (x  b).


3

2. Utilitza la igualtat anterior per demostrar la identitat trigonomètrica:


sin3 x  cos3 x  (sinx  cos x )(1  sinx cos x )

3. Resol l’equació trigonomètrica:


sin3 x  cos3 x  sinx cos x  1

4. Desenvolupa l’expressió (a  b ) i comprova que a 4  b4   a2  b2   2a2b2 .


4 4 2

5. Utilitza la igualtat anterior per demostrar la identitat trigonomètrica:


sin2 2x
sin4 x  cos4 x  1 
2

6. Resol les equacions trigonomètriques següents.


a) sen4 x  cos4 x  1
b) sen4 x  cos4 x  0
x x 5
c) sen4    cos4   
5 5 8

7. Troba per a quins valors de k, l’equació sin4 x  cos4 x  k té solucions reals. Comprova que no té
arrels anteres.

1 Matemàtiques 1r Batxillerat
Activitats de diversitat Unitat 3. Trigonometria
SOLUCIONARI

AMPLIACIÓ

Fitxa: Equacions trigonomètriques més complicades

01 0 b3
1. b b b2  b3
1 b b2 0
Es dedueix que  x 3  b3    x  b   x 2  bx  b  .

Per tant, si x  a  a3  b3   a  b  a2  ba  b2 

2. sin3 x  cos3 x  (sinx  cos x)(sin2 x  sinx cos x  cos2 x)  (sinx  cos x)(1  sinx cos x )

3. sin3 x  cos3 x  sinx cos x  1  sin3 x  cos3 x  sinx cos x  1  0 


 (sinx  cos x )(1  sinx cos x )  sinx cos x  1 
 (sinx  cos x  1)(1  sinx cos x )  0
Per tant, l’equació es redueix a resoldre les equacions:
1  sinx cos x  0 i sinx  cos x  1  0
1  sin x cos x  0  sin x cos x  1  2sin x cos x  2  sin2x  2  No té solució.
sin x  cos x  1  (sin x  cos x )2  sin2 x  cos2 x  2sin x cos x 

 1 2 1  sin2x  sin2x  0  2x  k   x  k 
2
 a2  b2   a4  b4  2a2b2  a4  b4  a2  b2   2a2b2
2 2
4.

1 1 sin2 2x
5. sin4 x  cos4 x   sin2 x  cos2 x   2sin2 x cos2 x  1 2  4sin2 x cos2 x  1   2sin x cos x   1 
2 2

2 2 2

sin2 2x sin2 2x k
6. a) sin4 x  cos4 x  1  1   1   0  sin2 2x  0  sin2x  0  2x  k   x 
2 2 2
sin2 2x sin2 2x
b) sin4 x  cos4 x  0  1  0  1  sin2 2x  2
2 2
Com que 0  sin2   1 per a qualsevol angle , l’equació no té solució.
 2x   2x 
sin2   sin2  
x x 5  5 5  5   3  sin2  2x   3  sin  2x    3 
c) sin4    cos4     1     
5 5 8 2 8 2 8  5  4  5  2
 2x  2x 4  5 10
 5  3  2k  5

3
 2k   x  6  5k  x  3  5k 
 
 
 2x 2 2x 5  5 25
   2k    2k  x   5k  x   5k
5 3 5 3  3 6
sin2 2x
7. sin4 x  cos4 x  k  1   k  sin2 2x  2  2k  0  2  2k  1  2  2k  1 
2
1 1 
 2  2k  1  1  k   k   , 1
2 2 

9 Matemàtiques 1r Batxillerat
Activitats de reforç Unitat 3. Trigonometria

RESOLUCIÓ DE TRIANGLES
1. Troba el valor del costat desconegut en cada triangle.

a) b) c) d)

2. Calcula el valor de x i y en cada cas.


a) b)

Resoldre un triangle és trobar la mida de cada costat i cada angle.


1. En aquest cas coneixem un angle i dos costats. Apliquem el teorema del cosi-
nus per trobar el valor oposat i, un cop coneguem els tres costats, apliquem el
teorema del sinus per trobar els altres angles:
b2  16  49  2  7  4cos100º  74,72  b  8,64 m
8,64 4 4 sen100º
  sin Aˆ   0,456  Aˆ 27º 7 12
sin100º sen Aˆ 8,64
Cˆ  180º  (100º  27º 7 12)  52º 52 48
2. Donats tres costats de longituds a  9 cm, b  3,6 cm i c  3 cm, resol el triangle que formen.
Com que coneixem els tres costats, podem aplicar el teorema del cosinus per trobar els angles:
92  3,62  32  2  3,6  3  cos Aˆ  cos Aˆ  2,73
Com que el valor del cosinus d’un angle ha d’estar comprès entre 1 i 1, aquesta solució no es vàlida. Aquest
triangle no es pot construir ja que la mida d’un dels costats és més gran que la suma dels altres dos.

3. Resol els triangles següents.


a) c) e)

b) d) f) Tres costats:
a2m
b1m
c1m

1 Matemàtiques 1r Batxillerat
Activitats de diversitat Unitat 3. Trigonometria
SOLUCIONARI

Fitxa: Resolució de triangles


1. a) Anomenem x el catet horitzontal, i y la hipotenusa.
Com que tenim la mesura del catet vertical, per trobar la mesura del catet horitzontal utilitzem la tangent de
30º:
10 10
tg30º  x  17,32 cm
x tg30º

Per trobar la mesura de la hipotenusa, podem utilitzar el sinus de 30º:


10 10
sin30º  y   20 cm
y sin30º

b) Com que l’angle és de 45º, el triangle és rectangle i isòsceles, és a dir, els cos catets són iguals.
Anomenem x el catet vertical, i y la hipotenusa. Podem aplicar les relacions trigonomètriques per
comprovar-ho i trobar la mesura de la hipotenusa:
x 1,56 1,56
tg45º   x  1,56  1  1,56 m cos 45º  y   2,21m
1,56 y cos 45º

c) Com que tenim la mesura de la hipotenusa, per trobar la mesura dels catets, anomenem x l’horitzontal, i y
el vertical:
x y
cos71º   x  12,7cos71º  4,13 m sin71º   y  12,7sin71º  12,008 m
12,7 12,7
d) En aquest cas, el catet horitzontal, x, és l’oposat a l’angle de 38º, per la qual cosa per trobar-lo utilitzem el
sinus de 38º. Per al catet vertical, y, utilitzem el cosinus de 38º:
x y
sin38º   x  6,5sin38º  4,001m cos 38º   y  6,5cos38º  5,12 m
6,5 6,5

10 10
2. a) tg20º  y   y  27,47 cm
y tg20º
15  10 25
tg( x  20º )    0,91  x  20º 42,30º  42º 18 8  x  42º 18 8
y 27,47
1 1
b) tg6º  y   y  9,51m
y tg6º
x 1
tg46º   x  1  9,51 tg46º  9,85cm  x  8,85 cm
9,51

7 Matemàtiques 1r Batxillerat
Activitats de diversitat Unitat 3. Trigonometria
SOLUCIONARI
3. a)
a2  90,12  100,22  2  90,1 100,2  cos25º  1793,72  a  42,35 m
42,35 90,1
  sin Bˆ  0,899  Bˆ 64,04º  64º 2 42 Cˆ  180º  (25º 64º 2 42)  90º 57 18
sin25º sin Bˆ

b) Cˆ  180º (93º 75º )  12


13,1 a 13,1 sin75º 13,1 c 13,1 sin12º
 a  12,67 cm  c   2,72 cm
sin93º sin75º sin93º sin93º sin12º sin93º
10 14,3 10sin54º
c)   sinCˆ   0,56  Cˆ 34,06º  34º 3 36
sinCˆ sin54º 14,3

Bˆ  180º (54º  34º 3 36)  91º 56 24


14,3 b 14,3 sin  91º 56 24 
 b  17,66 m
sin54º sin  91º 56 24  sin54º

d) 402  60,92  922  2  60,9  92cos Aˆ  cos Aˆ  0,94  Aˆ 19,34º  19º 20 24
40 92
  Cˆ 49,61º  49 º 36 95 Bˆ  180º (19 º 20 24  49 º 36 95)  111º 2 1
sin19,34º sinCˆ

e) a2  92  11 2 2  9  11cos108º  263,19  a  16,22 m


16,22 11 11sin108º
  sin Bˆ   0,64  Bˆ 40,16 º  40 º 9 36
sin108º sin B ˆ 16,22
Cˆ  180º  (108º  40º 9 36)  31º 50 24

f) 22  12  12  2  1 1cos Aˆ  cos Aˆ  1  Aˆ  180º


No hi ha solució, ja que les tres mesures no formen un triangle.

8 Matemàtiques 1r Batxillerat
Activitats d'ampliació Unitat 3. Àlgebra

PERÍMETRE I ÀREA

A la figura es veu l’hexàgon PQRSTU inscrit en el rectangle ABCD. L’hexàgon té quatre costats PQ, RS, ST
i UP de la mateixa longitud i els altres dos QR i UT també de la mateixa longitud.
Es donen les dades següents:
• Les dimensions del rectangle són 10 cm x 5 cm.
• El punt P és el punt mitjà del costat AB.
 1
• L’angle  compleix que: 0    arctg   .
2

a) Calcula el perímetre de l’hexàgon per als diferents valors de l’angle .


  
b) Calcula el valor del perímetre per a   ,  i   , i dona els resultats arrodonits a tres xifres deci-
5 6 8
mals.
c) Calcula el valor de  si se sap que el perímetre de l’hexàgon és tres quartes parts del perímetre del rec-
tangle.
d) Calcula l’àrea de l’hexàgon per als diferents valors de l’angle .
e) Calcula el valor de  si se sap que l’àrea de l’hexàgon és tres quartes parts de l’àrea del rectangle.

1 Matemàtiques 1r Batxillerat
Activitats de diversitat Unitat 3. Trigonometria
SOLUCIONARI

Fitxa: Perímetre i àrea

PB PB 5
a) cos    PQ  
PQ cos  cos 
BQ
tg    BQ  PB tg   5 tg 
PB
BQ  QR  RC  5  QR  5  2BQ  5  2  5 tg   5  10 tg 
5 20
Perímetre: 4PQ  2QR  4   2  5  10 tg     10  20 tg 
cos  cos 

 20 
b)   P   10  20 tg    20,190 cm
5  5
cos  
5
 20 
 P   10  20 tg    21,547 cm
6  6
cos  
6
 20 
 P   10  20 tg    23,364 cm
8  8
cos  
8

20 3 20 45
c)  10  20 tg   30    10  20 tg    40  20cos   40 tg  cos   45cos  
cos  4 cos  2
 25cos   40sin   40  0  5cos   8sin   8  0 

 5 1  sin2   8sin   8  0  25 1  sin2    64  64sin2   128sin  

128  50
 89 sin2   128 sin   39  0  sin   
178
39
   arcsin  0,4536 rad  25º 59
89

d) L’àrea dels triangles que apareixen a la figura és:


5BQ 5  5 tg  25
  tg 
2 2 2
25
L’àrea de l’hexàgon serà S  5  10  4  tg   50  50 tg   50(1  tg ).
2

3 3 1  1
e) 50(1  tg )   50  1  tg    tg      arctg    0,245 rad  14º 2
4 4 4 4

10 Matemàtiques 1r Batxillerat
Avaluacions
5 TRIGONOMETRIA

Avaluació de la unitat
NOM CURS CLASSE

1. Digues si són certes o falses les afirmacions següents. En cas 2. Resol aquests triangles:
de ser falses, formula’n una de certa.
v El sinus, el cosinus i la tangent sempre prenen un valor en-
tre −1 i 1.
a = 32 cm
v Si un triangle té dos angles que mesuren 54° i 46°, alesho-
res és un triangle rectangle.
v sin A$ cos A$  1
sin A$
B = 50° A = 30°
v tg A$ 
cos A$
1 1
v Si sin A$ 
`
, aleshores, sin (180° – A ) = – . A = 60°
5 5
v sin 45° = cos 45°
c = 78 cm
v sin 100° = sin 80° b = 43 cm

v cos 100° = cos 80°


v tg 45° = 1
v El teorema del sinus s’enuncia de la manera següent:
a b c
 
sin A$ sin B$ sin C$ c = 22 cm
a = 16 cm
v sin 60° = 2 · sin 30°
v L’angle del primer quadrant que podem relacionar amb
l’angle de 150° és el de 50°.
b = 15 cm
v (sin A + cos A) 2 = 1
3. Dues persones es troben separades 153 m. Veuen un objecte
a l’altra banda del riu, un amb un angle de 22° i l’altre amb un
angle de 53°. Si volen agafar l’objecte amb una corda, quant
ha de mesurar la corda de cadascú? Fes-ne el dibuix.

156
AVALUACIÓ DE TRIGONOMETRIA

EXERCICIS(20p)______________________________

1.- Calcula el radi d’una circumferència, si sabem que un angle central de 3 radians
abraça un arc de circumferència de 15 metres. (1p)

2.- Calcula el cosinus i la tangent d’un angle agut, si sabem que el sinus val 2/3. (2p)

3.- L’ombra d’un pal és de 2m. Si el raig del sol forma un angle de 30º amb el terra,
calcula la seva alçada. (1p)

4.- El circ és a la ciutat i s’ha d’instal·lar. L’especialista a muntar-lo encara no ha arribat


i els altres no saben la quantitat de cable d’acer que necessiten. El més espavilat recorda
que, un cop tensat el cable des de l’extrem del pal principal fins a un punt determinat del
terra, amb el qual forma un angle de 60º, calen dos metres més de cable, que si forma
amb el terra un angle de 70º. En total han de posar sis cables tensats formant amb el
terra un angle de 60º. Quants metres de cable necessitem?. (3p)

5.- Quina és la superfície del cercle en el qual podem inscriure un triangle equilàter de
perímetre 60 cm?. (2p)

6.- Dibuixa i assenyala, en la circumferència unitat, els angles que tenen per: (3p)

a) cosinus (– 5/7)

b) tangent (– 1,75)

7.- Defineix les raons trigonomètriques sinus, cosinus i tangent dels nombres reals: (3p)

a) 4890º

b) – 2760º

8.- Resol en graus i radians les equacions trigonomètriques: (5p)

a) cos x = 3 2

b) tan x = – 3
AVALUACIÓ DE TRIGONOMETRIA

CONCEPTES (15p)

1.- Demostra, en la circumferència unitat, que sin2 + cos2 = 1, per un


angle qualsevol del segon quadrant. (2p)

2.- Raons trigonomètriques (sinus, cosinus i tangent) dels angles 0º, 90º,
180º, 270º i 360º. (2p)

sin 0º = sin 90º = sin 180º =


cos 0º = cos 90º = cos 180º =
tan 0º = tan 90º = tan 180º =

sin 270º = sin 360º =


cos 270º = cos 360º =
tan 270º = tan 360º =

3.- Demostra, en la circumferència unitat, la relació entre les raons


trigonomètriques sinus, cosinus i tangent de dos angles suplementaris
qualsevol. (3p)

4.- Representa gràficament, en la circumferència unitat, les raons


trigonomètriques sinus, cosinus i tangent d’un angle qualsevol del quart
quadrant. (2p)

5.- Valors màxim i mínim del sinus, cosinus i tangent d’un angle 
qualsevol. (2p)

6.- Existeix un angle agut tal que el seu sinus sigui 2/3 i a la vegada la seva
tangent sigui 2?. Raona la teva resposta. (2p)

7.- Calcula raonadament el valor real exacte d’aquesta expressió: (2p)

3 . sin250º + 3 . cos250º - 2 – sin220º - sin270º =

-1-
PROCEDIMENTS (18p)

1.- La longitud d’una circumferència és 11 cm. Calcula la longitud exacta


de l’arc que abraça un angle central de 70º. Raona la teva resposta. (2p)

2.- Troba l’altura exacta d’un gratacels, sabent que des de la finestra d’un
edifici proper es veu el punt més alt formant un angle de 60º amb la visual
horitzontal (angle d’elevació), i el punt on l’edifici toca a terra sota un
angle de 30º amb la visual horitzontal (angle de depressió). L’altura de la
finestra d’observació és de 50m sobre el terra. (3p)

3.- Si cosec a = 4/3 i /2  a  , calcula el sinus, cosinus i la tangent. Els


resultats han de donar-se simplificats i racionalitzats. (2p)

4.- Dibuixa i assenyala en la circumferència unitat (1u=8cm) tots els angles


que tenen per cosinus (–0,75). (2p)

5.- Calcula en radians els angles complementari i suplementari de 2/5 rad.


(1p)
6.- a) Calcula raonadament el valor real aproximat del sinus, cosinus i
tangent del número real: 1232º. Representa’l en el calculador
trigonomètric. (2p)

b) Calcula raonadament el valor real exacte del sinus, cosinus i tangent


del número real: -780º. (2p)

7.- Resol les equacions trigonomètriques:


a) cos x = -  3 / 2 (radians) (2p)

b) tan x =  3 / 3 (graus) (2p)

-2-
AVALUACIÓ DE TRIGONOMETRIA(12p)
=========================================================
1. Completar les taules: (2p)

RADIANS ∏/2 3∏/2 2∏

GRAUS 30º 60º 180º 45º 150º

ANGLES 0º 30º 45º 60º 90º 180º 270º 360º

SIN

COS

TAN

2. Dibuixa un triangle rectangle amb un angle agut de 55º i calcula el seu sinus,
cosinus i la seva tangent. Es tindrà en compte la precisió de les dades obtingudes,
així com la presentació. (2p)
3. Utilitza el teu calculador trigonomètric, que has d’entregar, amb el teu nom, junt
amb l’examen, per obtenir, amb la màxima precisió, el sinus, cosinus i la tangent
de l’ angle de 293º. S’ avaluarà també la presentació. (2p)
4. Els dos braços d’un compàs formen un angle de 60º. Si cada un d’ells mesura
10 cm., troba el radi “r” de la circumferència que podem dibuixar. Comença fent
una representació gràfica del problema, amb totes les dades de l’ enunciat. (2p)
5. L’ombra d’ un pal és de 2m. Els raigs del sol formen un angle de 25º amb el terra.
Dibuixa aquesta situació amb les dades, col·locant-les correctament, i desprès
calcula l’ alçada “h” del pal. (1p)
sin25º=0.42 ; cos25º=0.90 ; tan25º=0.47
6. Els dos angles aguts d’ un rombe mesuren 60º i la diagonal petita té una longitud
de 12 cm. Dibuixa el rombe amb aquestes dades i calcula el seu perímetre i la
seva àrea. (3p)
AVALUACIÓ DE TRIGONOMETRIA(14p)
=======================================================
1.- Omple les caselles buides de la taula de la pàgina (2) amb els valors “exactes”
corresponents.
Nota: per cada casella equivocada o buida es restarà una dècima a la puntuació
màxima, que és 4p., fins un mínim de 0p. (4p)

2.- Sabent que tan a = -3 i que 90º≤ a ≤ 180º, troba sin a i cos a . Dóna els
resultats indicats(sense decimals), simplificats i racionalitzats.
(2p)

3.- Dibuixa i assenyala, en la circumferència unitat, els angles que tenen per
cosinus (-5/6). Es tindrà en compte la cura en l’ execució del dibuix.
(2p)
4.- Resol, en graus, les equacions trigonomètriques:

(a) sin x = - sin 65º tan x = tan 120º (2p)

5.- Troba raonadament el valor exacte de l’ expressió:

sin2 19º + sin2 10º + cos2 40º + sin2 80º + cos2 50º + cos2 19º = (1p)

6.- Es vol trobar l’ alçada d’ una torre AB de peu inaccessible. Per fer això, des de
dos punts P i Q, alineats amb el peu de la torre, es prenen les mides següents:
a) distància de P a Q = 50 m.
b) angle de visió de la torre des de P = 34º
c) angle de visió de la torre des de Q = 72º
Representa gràficament aquesta situació i troba l’ alçada de la torre, o sigui AB.
(3p)
Dades addicionals: sin 34º = 0.56 sin 72º = 0.95
cos 34º = 0.83 tan 72º = 3.07

-Pàgina 1-
TAULA
GRAUS RADIANS SINUS COSINUS TANGENT

0º 0

∏ /6

45º

60º

90º

120º

3∏ /4

150º

180º

210º

5∏ /4

240º

270º

5∏ /3

315º

330º

360º

-Pàgina 2-
AVALUACIÓ DE TRIGONOMETRIA(6p)
====================================================
1. Esbrina, en el calculador trigonomètric, el sinus, cosinus i la tangent de
l’ angle de 145º. Es valorarà la presentació i la precisió dels resultats
obtinguts. (1p)

2. Es vol trobar l’ alçada d’ una torre AB de peu inaccessible. Per fer això, des
de dos punts P i Q, alineats amb el peu de la torre, es prenen les mides
que mostra la figura.
B

34º 72º

P 50m Q A

Troba l’ alçada de la torre, o sigui AB. (2p)


Dades addicionals: sin 34º = 0.56 sin 72º = 0.95
cos 34º = 0.83 tan 72º = 3.07

3. Troba l’ àrea del triangle de la figura adjunta.

8m

10m 27º

Dades addicionals: sin 27º = 0.45 i cos 27º = 0.89 (1p)

4. Un globus aerostàtic està lligat a terra amb dues cordes tenses, que
formen amb l’ horitzontal del sòl angles de 65º. Si la distància entre els dos
punts on són lligades les cordes és de 5 m., troba l’ alçada del globus a
terra i la longitud de les cordes. (2p)
Dades addicionals: sin 65º = 0.91 i tan 65º = 2.14
AVALUACIÓ DE TRIGONOMETRIA(10p)
========================================================
1. Si l’ angle agut “a” té per secant 3/2, emprant les relacions entre les raons
trigonomètriques, calcula el valor exacte del seu sinus, cosinus i tangent.(2p)

2. Demostra que, per un angle agut “a” qualsevol, és sempre cert que:
sin2a + cos2a = 1. (2p)

3. Sigui el triangle ABC de la figura:


A

c b=2cm.

45º 30º
B a C

Calcula el valor exacte(sense decimals) de:

a) l’ angle A. (0.25p)
b) l’ altura AH. (0.75p)
c) el costat “c”. (0,50p)
d) el costat “a”. (1p)
e) el perímetre del triangle (0.25p)
f) l’ àrea del triangle. (0.25p)

4. Al terrat d’ un edifici hi ha instal·lada una antena de telefonia mòbil. Des d’ un


punt P del carrer, l’ angle entre l’ horitzontal i la línia que va de P cap a
l’ extrem superior de l’ antena és de 34º. Ens apropem fins a un punt Q que és
15 metres més a prop de l’ edifici i ara l’ angle entre l’ horitzontal i la línia que
apunta cap a l’ extrem superior de l’ antena és de 42º, mentre que l’ angle
entre l’ horitzontal i la línia que apunta cap a l’ extrem inferior de la mateixa
antena és de 35º.

a) Feu un esquema de la situació, marcant molt clarament quins són els angles
i la distància que es donen a l’enunciat. (1p)
b) Calculeu l’altura de l’antena (x) i l’altura de l’edifici (h). (2p)

Dades: sin34º=0.56 sin35º=0.57 sin42º=0.67


cos34º=0.83 cos35º=0.82 cos42º=0.74
tan34º=0.67 tan35º=0.70 tan42º=0.90
AVALUACIÓ DE LA UNITAT UNITAT 3. TRIGONOMETRIA

Nom: ………………………........……… Curs: ....…………… Data: ....…….....

1. Sabent que sec 273° = h calcula, en funció de h, sin 267° .


2. Calcula totes les raons trigonomètriques de α sabent que verifica les dues condicions
següents:

3π 2
a) ≤ α ≤ 2π b) cotg α = −
2 2
3. Sabent que sin α = 0,75 i cos β = −0,25 i que α i β són dos angles del segon quadrant,
calcula:
a) sin ( α + β ) b) cos ( α + 2 β ) c) tg ( α − β )

4. Si α és un angle del segon quadrant i cotg α = −10, calcula:

α α
a) sin   b) cos  
2 2

tg x − sin x 1
5. Demostra la identitat trigonomètrica = .
sin3 x cos x + cos 2 x

6. Resol les equacions trigonomètriques:


3
a) 2 sin ( 2x + 45° ) = − 2 b) cos 2x = − 5 cos2 x
4

7. Resol el triangle de la figura i calcula’n l’àrea.

8. Des d’un punt veiem un núvol amb un angle d’elevació de 50º. Si tirem 2 km enrere, veiem
el núvol sota un angle de 20º, tal com mostra el dibuix.
a) Calcula l’altura del núvol.
b) Suposant que el núvol es manté fix, quant ens
haurem de desplaçar per observar-lo amb un angle
que sigui la meitat de l’angle inicial?
c) És possible que trobem dues solucions? Raona la
resposta.

3 Matemàtiques 1r Batxillerat
AVALUACIÓ DE LA UNITAT UNITAT 3. TRIGONOMETRIA
SOLUCIONARI

1 1 h2 − 1
1. h = sec 273° = sec 87° ⇒ cos 87° = ⇒ sin2 87° + cos2 87° = 1 ⇒ sin 87° = 1 − 2 = ⇒
h h h
h2 − 1
⇒ sin 273° = − sin 87° = −
h

2 1 1
2. cotg α = − ⇒ tg α = − 2 ⇒ tg2 α + 1 = ⇒ cos2 α = ⇒
2 cos2 α 3

 3
cos α = ⇒ sec α = 3
⇒ 3
sen α = − 6 ⇒ cosec α = − 6
 3 2

3. a) sin ( α + β ) = sin α cos β + cos α sin β = −0,828


b) sin2β = 2s i n β cos β = −0,4841 
 ⇒ cos ( α + 2β ) = coscos 2β − sin α sin2β = 0,942
cos 2β = cos2 β − sin2 β = −0,875 
tg α − tg β
c) tg ( α − β ) = = 0,508
1 + tg α tgβ

1
4. cotg α = −10 ⇒ 1 + tg2α = sec 2 α ⇒ sec α = − 1 + = −1,005 ⇒ cos α = −0,995
100

α 1 − cos α
a) sin   = = 0,9987
2 2

α 1 + cos α
b) cos   = = 0,05
2 2

sin x sin x (1 − cos x )


− sin x
tgx − sinx cos x cos x 1 1
5. = = = =
sin3 x sin x (1 − cos2 x ) sin x (1 − cos x )(1 + cos x ) cos x (1 + cos x ) cos x + cos2 x

2
6. a) 2 sin ( 2x + 45o ) = − 2 ⇒ sin ( 2x + 45° ) = − ⇒
2

{ {
⇒ 2x + 45° = 225° + 360°k ⇒ x = 90° + 180°k
2x + 45° = 315° + 360°k x = 135° + 180°k
3 3
b) cos 2x = − 5 cos2 x ⇒ cos2 x − sin2 x = − 5 cos2 x ⇒
4 4
3 1
⇒ cos2 x − 1 + cos2 x = − 5 cos2 x ⇒ cos2 x =
4 4

{
⇒ x = 60o + 360o k; x = 300o + 360o k
o o o

x = 60 + 360 k; x = 240 + 360 k


o

Els costats del triangle són a = 18, b = 15 i c = 10, aplicant el teorema del cosinus:

9 Matemàtiques 1r Batxillerat
AVALUACIÓ DE LA UNITAT UNITAT 3. TRIGONOMETRIA
SOLUCIONARI

= b2 + c 2 − a 2 1 1 
cos A = ⇒ Aˆ = arcos = 89° 49′
2bc 300 300 
2 2 2  ⇒ Cˆ = 33° 44′
+ − 199 199
= 56° 27′ 
= a c b
cos B = ⇒ Bˆ = arcos
2ac 360 360 

1
S= ⋅ 10 ⋅ 18 ⋅ sin 56o 27´= 75 cm2
2

8. a) tg 50o = h 
 h = 1,1918 x  x = 0,8794 km
⇒ ⇒ ⇒
x
h  h = 0,7279 + 0,364 x  h = 1,0481 km 
tg 20o =
2 + x 

El núvol està a 1048,1 m de terra.


 1,0481
 tg25° = ⇒ x1 = 1,3683 km
50°  x1 + 0,8794
b) i c) = 25° ⇒ 
2 tg25° = 1,0481 ⇒ x2 = 2,2477 km
 x2
Per observar el núvol des d’un angle meitat de l’inicial tenim dues solucions possibles,
1368,3 m si ens allunyem del núvol cap a l’esquerra, o 3127,1 m, si sobrepassem el núvol
cap a la dreta.

10 Matemàtiques 1r Batxillerat
CÁLCULO MENTAL
Trigonometría

A B C D E F

1 sen 10º = sen __ tg 210º = tg __ cos 45º = ____ cos 100º = cos__ tg 150º = tg __ tg 20º = tg __

2 cos 160º = cos__ sen 45º = ____ sen 260º = sen __ sen 210º = sen __ sen 50º = sen __ cos 360º = cos__

3 sen 330º = sen __ sen 20º = sen __ tg 225º = ____ tg 45º = ____ cos 170º = cos__ sen 225º = ____

4 tg 30º = tg __ tg 90º = ____ cos 240º = cos__ sen 40º = sen __ cos 350º = cos__ tg 100º = tg __

5 tg 350º = tg __ sen 190º = sen __ tg 220º = tg __ cos 190º = cos__ tg 240º = tg __ sen 60º = sen __

6 sen 240º = sen __ cos 140º = cos__ sen 30º = sen __ tg 50º = tg __ sen 310º = sen __ tg 250º = tg __

7 cos 270º = ____ tg 40º = tg __ cos 315º = ____ cos 160º = cos__ cos 110º = cos__ tg 270º = ____

8 tg 290º = tg __ sen 160º = sen __ cos 330º = cos__ tg 230º = tg __ tg 10º = tg __ tg 190º = tg __

9 sen 70º = sen __ cos 230º = cos__ sen 110º = sen __ sen 90º = ____ cos 290º = cos__ sen 140º = sen __

10 tg 80º = tg __ sen 280º = sen __ sen 350º = sen __ cos 20º = cos__ sen 270º = ____ cos 120º = cos__

11 sen 150º = sen __ cos 225º = ____ tg 130º = tg __ tg 320º = tg __ tg 60º = tg __ sen 320º = sen __

12 cos 210º = cos__ sen 80º = sen __ cos 90º = ____ sen 120º = sen __ sen 315º = ____ tg 340º = tg __

13 tg 260º = tg __ tg 120º = tg __ sen 170º = sen __ tg 140º = tg __ tg 330º = tg __ cos 40º = cos__

14 sen 360º = ____ cos 320º = cos__ cos 10º = cos__ sen 300º = sen __ sen 220º = sen __ sen 230º = sen __

15 tg 200º = tg __ sen 100º = sen __ sen 200º = sen __ cos 340º = cos__ cos 135º = ____ tg 160º = tg __

16 cos 50º = cos__ tg 300º = tg __ cos 150º = cos__ tg 315º = ____ cos 30º = cos__ cos 210º = cos__

17 sen 135º = ____ sen 180º = ____ sen 290º = sen __ cos 80º = cos__ tg 360º = ____ tg 135º = ____

18 tg 170º = tg __ cos 60º = cos__ cos 180º = ____ tg 180º = ____ cos 200º = cos__ tg 70º = tg __
Relaciones trigonométricas (SOLUCIONES)
SOL A B C D E F
1 170 30 √2/2 260 330 200
2 200 √2/2 280 330 130 0
3 210 160 1 1 190 315
4 210 ∞ 120 140 10 280
5 170 350 40 170 60 120
6 300 40 150 230 230 70
7 0 220 √2/2 200 250 -∞
8 110 20 30 50 190 10
9 110 310 70 1 70 40
10 260 260 190 340 -1 240
11 30 -√2/2 310 140 240 220
12 150 100 0 60 225 160
13 80 300 10 320 150 320
14 0 40 350 240 320 310
15 20 340 340 20 225 340
16 310 120 210 135 330 150
17 √2/2 0 250 280 0 -1
18 350 300 -1 0 160 250

NIVEL EDUCATIVO:

➢ 4º ESO

PUNTUACIÓN APROX.

PUNTOS 2 3 4 5 6 7 8 9 10 11
NOTA 1 2 3 4 5 6 7 8 9 10

INDICACIONES
• Importante tener claro la interpretación
Con esta hoja trabajaremos: geométrica de las razones trigonométricas.

• Razones trigonométricas de 0º,90º,180º,270º.


• Razones trigonométricas de 45º , 135º, 225º y 315º
• Ángulos con las mismas razones trigonométricas

RESULTADOS
OBSERVACIONES. Debido a su alta dificultad ,
GRUPO: ________________ tal vez sea más interesante dar a los alumnos un
minuto y medio o dos de tiempo y subir el 5 a 8-10
PUNTOS aciertos .
Media de la Clase
Máxima de la Clase

You might also like